Sei sulla pagina 1di 54

Beers Criteria -- evaluates inappropriate Rx use in elderly

Drugs to avoid:
 Nitrofurantoin  pulm & hepatic tox; peripheral neuropathy; lack of efficacy <30 mL/min (avoid if CrCl < 30 mL/min)
 Antipsychotics  inc CVA risk; inc cognitive decline; inc mortality in dementia
 Insulin (sliding scale), chlorpropamide, glyburide  hypoglycemia
 BZD  inc risk of injury, confusion
 Megestrol  risk of thrombolytic events
 PPIs  risk of C. differ, bone loss, fx
 Non-COX NSAIDs  GI bleed (can use if protect w/ PPI or misoprostol)
 Non-BZD hypnotics (“z” Rx)  inc cognitive effects, inc risk of injury
 Estrogens WWO progestin  carcinogenic potential (can use for CV dz PPX)
 Mm relaxants  antichol, falls
Syndrome Interactions
 Syncope: AchEis, peripheral alpha-blockers, TCAs, chlorpromazine, olanzapine  orthostatic hypo or bradycardia
 Insomnia: PO decongestants, stimulants, theobromines  CNS stimulant
Use Rx w/ caution
 Dabigatran  risk of bleeding (caution if ≤ 75 y/o or CrCl <30 ml/min)
 Rx linked w/ SIADH/hypoNa (SSRI, TCA, CBZ, antipsychotics)  can exacerbate SIADH, hypoNa (monitor Na+)
Avoid these Rx interactions
 ACE-Is + amiloride or triamterene  inc risk of hyperK
 Antichol + antichol  inc risk of cognitive decline
 Antidepressant OR Antipsychotics + 2+ other CNS-active Rx  inc risk of falls
 BZD + 2 other CNS-active Rx  inc risk of falls & fx
 Steroids + NSAIDs  inc risk of PUD or GI bleed
 Lithium + ACE-Is OR loop diuretics  lithium tox
 Opioids + 2+ CNS-active Rx  inc risk of falls
 Peripheral a-blockers + loop diuretics  inc risk of urinary incontinence (esp older women)
 Theophylline + Cimetidine  inc risk of theophylline tox
 Warfarin + amiodarone OR NSAIDs  inc risk of bleeding
Rx CrCl to take action Why Recommendation for reduced kidney fxn
Rivaroxaban 30 – 50 Inc risk of bleeding Reduce
<30 Avoid
Spironolactone <30 Inc K Avoid
Triamterene <30 Inc K, dec Na Avoid
Duloxetine <30 Inc GI upset (N/V) Avoid
Gabapentin <60 CNS adverse effects Reduce
Levetiracetam <80 CNS adverse effects Reduce
Pregabalin <60 CNS adverse effects Reduce
Tramadol <30 CNS adverse effects Reduce IR; Avoid ER
Cimetidine <50 AMS Reduce
Famotidine <50 AMS Reduce
Nazatidine <50 AMS Reduce
Rantidine <50 AMS Reduce
Colchicine <30 GI upset, neuromuscular, bone marrow tox Reduce + monitor for SE
Probenecid <30 Loss of effectiveness Avoid

Behavioral Disturbances in Dementia

 Apathy, poor self-care, or paranoia  usu 1st indication of dementia


 Agitation  loss of ability to modulate behavior in a socially acceptable way
o Verbal outburst, physical aggression, resistance to bathing/self-care, restless motor activity (pacing, rocking)
o Often w/ psychotic ssx (paranoia, delusions, hallucinations)
 Medical Cz’es
o New, acute, or rapidly evolving disturbances  MC d/t medical condition or Rx tox
 Rx tox can be behavioral ssx alone
o Persistent or insidious onset  MC brain deterioration
o Isolated behavioral disturbance in demented pt can be sole presenting ssx  UTI, pain, angina, constipation, uncontrolled DM
 Environmental causes
o Life stressor, Daylight savings, jetlag, New routine/caregivers/roommate, Over or under stimulation, Disruptive behavior of other pts
 Catastrophic rxn: acute behavioral, physical, or verbal rxn to environmental stressors  inability to make routine adjustments in daily life
o Anger, emotional lability, aggression
o Tx: ID & avoid precipitants, provide structured routines/activities, recognize early signs
 Behavioral ssx are categorized in 3 groups: mood ssx, psychosis, specific behavior problems
o If polyssx – target tx to prevailing feature
 Types of Dementia
o Frontotemporal Dementia (Pick’s Dz) = Prominent disinhibition, compulsive behaviors, social impairment
 Severe ssx: hyperphagia, hyperactivity, hypersexuality
o Dementia w/ lewy bodies = Visual hallucinations
 Tx:
o Manage pain, dehydration, hunger, thirst
o Consider positional discomfort or nausea 2/2 Rx
o Modify environment to improve orientation  keep calm, comfortable, & homelike  Provide regular daily activities & structure

1
o Eval & tx underlying dx  Replace hearing aids, dentures, glasses & remove offending Rx (look for antichol agents)
 Interventions
o Assess for new medical problems
o Attend to pt sleep and eating patterns & simplify bathing/dressing
o Install safety measures to prevent accidents
o Ensure caregiver is adequate; educate caregivers
o Refer to local Alzheimer’s Assoc.
o Consider antidepressants if…
 Depression > 2 wks + significant distress
 Depression > 2 mo after initiation of behavioral interventions

Antidepressants:
 If failed after 8 – 12 wks  2nd line (if failed multiple therapies  consider ect)
 Avoid TCA’s if (+) BBB or cardiac conduction disturbance
 Sometimes Stimulant (methylphenidate 2.5 – 10mg QD) is helpful but can exacerbate psychotic ssx
o Avoid stimulant + bupropion (b/c already has stimulant action)

SSRI’s: 1st line agents


Rx Daily Dose Use Precautions
Citalopram (Celexa) 10 – 20 mg (max dose) Depression, anxiety (off- GI upset, insomnia, risk of QT prolongation w/ doses >20 mg
label)
Escitalopram (Lexapro) 5 – 20 mg Depression, anxiety
Fluoxetine (Prozac) 10 – 40 mg Depression, Anxiety Long half-life, greater inhibition of cyP450
Paroxetine (Paxil) 10 – 40 mg Depression, Anxiety Greater inhibition of CYP450, some antichol effects
Sertraline (Zoloft) 25 – 100 mg Depression, Anxiety
Vilazodone (Viibyrd) 10 – 40 mg Depression, Anxiety Take w/ food, adjust dose for severe hepatic dz, reduce dose to 20 mg
if given w/ CYP3A4 inhibitors
Vortioxetine (Trintellix) 5 – 10 mg Depression Nausea, dizziness, fewer sexual SE’s than other SSRI’s

SNRI’s:
Rx Daily Dose Use Precautions
Desvenlafaxine (Pristiq) 25 – 50 mg Depression, fibro Nausea, HTN, dry mouth, dizziness, HA
Duloxetine (Cymbalta) 20 – 60 mg Depression, diabetic neuropathy Nausea, dry mouth, dizziness, HTN
Mirtazapine (Remeron) 7.5 – 30 mg Depression w/ insomnia + weight loss Sedation, hypo-tn, potential for neutropenia
Venlafaxine (Effexor) 25 – 150 mg Severe depression, Anxiety HTN, insomnia
Desipramine (Norpramin) 10 – 100 mg Severe depression, anxiety, high degree of efficacy Antichol SE’s, hypo-TN, sedation, cardiac
arrhythmias
Nortriptyline (Pamelor) 10 – 75 mg High efficacy for depression if SE tolerable; Antichol SE’s, hypo-TN, sedation, cardiac
therapeutic level: 50 – 150 ng/dL arrhythmias, caution w/ glaucoma

Other Rx:
Rx Daily Dose Use Precautions
Bupropion (Wellbutrin) 75 – 225 mg More activating, lack of cardiac effects Irritability, insomnia
Gabapentin (Neurontin) 100 – 300 mg Anxiety (off label), insomnia (off label) Sedation, falls, hypo-TN
Trazodone (Oleptro) 25 – 150 mg When sedation is desirable Sedation, falls, hypo-TN, priapism

Manic-like Behavior
 In dementia pts, they usu have a fluctuating mood (irritable or hostile as opposed to euphoric)

Rx: Approved by FDA for Bipolar, but off label for manic-like behavior in dementia
Rx Geri Dosage SE’s Comments
Carbamazepine 200 – 1000 mg/d (thera level: Nausea, fatigue, ataxia, blurred Poor tolerability in elderly; must monitor CBC, LFTs, electrolytes
(Tegretol) 4 – 12 ug/mL) vision, hypoNa+, leukopenia q 2wks for 1st 2 mo then q 3 mos
Can start @ 100 mg q12h
Lamotrigine 25 – 200 mg/d Sedation, SJS, dizziness, anemia Inc SE’s w/ divalproex, slow-dose titration req
(Lamictal)
Lithium 150 – 1000 mg/d (thera level N/V, tremor, confusion, Poor tol in elderly, tox @ low serum levels, monitor thyroid & renal
0.5 – 0.8 mEq/L) leukocytosis fxn
Divalproex Sodium 250 – 2000 mg/d (Thera Nausea, GI upset, ataxia, sedation, Monitor CBC, Plts, LFTs @ baseline & q 6 mo; better tolerated
(Depakote)* level 50 – 100 ug/mL) hypoNa+ than other mood stabilizers in elderly

Sustained release Depakote is commonly recommended – typical


starting dose: 125 mg BID then titrate up slowly w/ monitoring for
sedation, ataxia, & falls

Delusions & Hallucinations


 Clinical Dx of Alzheimer’s w/ psychosis requires delusions or hallucinations for at least 1 mo (intermittently is ok) + cz’es distress
o Delusions: fixed false beliefs
o Hallucinations: sensory experiences w/o stimuli
 Tx If pt is disturbed or lead to disruption in pt’s environment

Antipsychotic Agents
Rx Daily Dose Adverse Events Comments Forms

2
Aripiprazole (Abilify) 2 – 20 mg Mild sedation, hypo-TN Tablet, rapid dissolving tab, IM
inj, liq
Asenapine (Saphris) 5 – 10 mg Sedation Only sublingual
Clozapine (Clozaril) 12.5 – 200 Sedation, hypo-TN, antichol, agranulocytosis Weekly CBC; poorly tol by Tab, rapidly dissolving tab
mg elderly; reserve for tx of refractory
dz
Haloperidol (Haldol) 0.5 – 3 mg Sedation, EPS 1st gen agent Tab, Liq, IM, long-acting inj
Iloperidone (Fanapt) 1 – 12 mg Sedation, orthostatic hypo-TN  dose w/ CYP3A4/CYP2D6-I Tab
Lurasidone (Latuda) 40 -80 mg Sedation Don’t exceed 40mg QD w/ Tab
CYP3A4-I
Olanzapine (Zyprexa) 2.5 – 15 mg Sedation, falls, gait disturbance Weight gain, hyperglycemia Tab, rapid dissolve tab, IM inj
Perphenazine 2 – 12 mg EPS, sedation 1st gen agent Tab
(Trilafon)
Paliperidone (Invega) 1.5 – 12 mg Sedation, fatigue, GI upset, EPS Reduce dose in renal impairment ER tab, Depot IM long-acting inj
Quetiapine (Seroquel) 25 – 200 mg Sedation, hypo-TN Ophthalmologic exam q 6 mo Tab, sustained release tab
Most sedating
Risperidone 0.5 – 2 mg Sedation, hypo-TN, EPS w/ doses > 1mg/d Tab, rapid dissolving tab, liq,
(Risperdol) depot IM long-acting inj
Ziprasidone (Geodon) 40 – 160 mg Higher risk of prolonged QT Little published info in elderly Capsule, IM inj
*All have warnings about hyperglycemia, ketoacidosis, hyperosmolar states, CVA, & inc in all-cz mortality in pts w/ dementia
*All are off-label tx for psychosis in dementia
*Do not use as 1st choice for tx of behavioral & psychological ssx of dementia
*Clozapine & Quetiapine can be esp helpful for Parkinson Dementia or Dementia w/ Lewy Bodies
*Risperidone & Aripiprazole is more effective fore anger, aggression, & paranoid ideation when used for ≤ 12 wks

Cholinesterase inhibitors
 Moderate Alzheimer’s: donepezil or galantamine are better
 use in Lewy body Dementia to reduce visual hallucinations

Sleep disturbances
 Improve sleep hygiene &Tx assoc depression, suspiciousness, delusions
 Off-Label Rx:
o Trazodone 25 – 50 MG qhS
o Mirtazapine 7.5 – 15 mg qHS
o Gabapentin
o Zolpidem 5 mg at bedtime (improves sleep onset, not duration b/c of short half-life)
o Zaleplon
o Melatonin (OTC)
 Avoid BZD or Antihistamines (ex: diphenhydramine)  Risk of falls, hip fx, disinhibition, cognitive disturbance

Inappropriate sexual behaviors


 Exclude underlying txable cz’es or tx underlying syndrome
 Antiandrogens for dangerously hypersexual or aggressive
o Progesterone 5 mg/d PO (adjust dose to suppress testosterone well below nl)
 Good response  tx w/ 10 mg IM depot progesterone q week
o Leuprolide acetate 5 – 10 mg IM q mo

Intermittent aggression or agitation (once per wk or less)


 Behavioral interventions – distraction, reminiscence, validation therapy, environmental mods, caregiver edu/support, music therapy, PT, aromatherapy
 Behavioral mods w/ positive reinforcement
 Avoid physical restraints

Cases & Practice questions:


Case 1:
• An 86-year-old man has episodes of increasing psychosis and aggression over the past 2 months.
 Primary caregiver is his daughter, whom he verbally abuses and threatens; he has punched her on 3 occasions.
 Believes that his food is being poisoned
 Believes that his son, who lives 1,000 miles away, has been coming into their home and stealing
 Attempts at nonpharmacologic interventions have been unsuccessful
• History: moderate Alzheimer disease
• Which one of the following is the most appropriate pharmacologic treatment for this patient?
A. Citalopram
B. Donepezil
C. Haloperidol
D. Risperidone
E. Valproic acid

ANSWER: D
At present, no pharmacologic therapy can confidently be considered safe or effective for Tx of psychosis or physical aggression in AD, and no Rx currently has approval
from the FDA for such use. In expert consensus statements, nonpharmacologic strategies are unanimously recommended as first-line treatment. Such strategies include
provision of daily activity and structure, and educating the caregiver about verbal and nonverbal strategies to minimize the pt’s distress. Although nonpharmacologic
strategies are safer options than pharmacotherapy, the evidence for their effectiveness in treating either psychosis or aggression in AD remains limited. When
nonpharmacologic strategies have failed, or ssx cz severe distress or pose a safety risk to the pt or others, off-label trial of a pharmacologic agent is warranted.
All 1st- & 2nd-generation antipsychotic agents carry an FDA black-box warning about increased mortality and risk of cardiovascular events when used for patients with
dementia. Risks and benefits must be weighed carefully when deciding whether to prescribe an antipsychotic agent.

3
Although modest at best, evidence of efficacy for neuropsychiatric ssx in AD is strongest for 2nd-gen antipsychotic agents. The most encouraging evidence is for
risperidone. Evidence for the efficacy of haloperidol is slimmer, and mortality risk appears to be even higher.
The Citalopram for Agitation in AD Study randomly assigned participants to receive either citalopram at 30 mg/d or placebo for 9 weeks. Compared with placebo,
citalopram significantly reduced both pt agitation and caregiver distress. However, worsening of cognition and prolongation of the QTc interval were reported as adverse
effects. The FDA has since recommended that, because of concerns about QTc prolongation, the max citalopram dosage for older pts should not exceed >20 mg/d. This
dosing requirement may limit the usefulness of citalopramOL in treating neuropsychiatric ssx in AD.
Although the results from some clinical trials suggest that ACHE-I such as donepezilOL decrease neuropsychiatric symptoms in dementia, this effect is generally mild,
and AchE-I would therefore not be recommended as first-line therapy for severe neuropsychiatric symptoms, such as those described in this case. The most recent review
of randomized placebo-controlled studies of valproic acidOL confirmed previous findings of its ineffectiveness in treating agitation in dementia.

Case 2:
• A 78-year-old woman has had disrupted sleep for the past month.
 Her son says that she has difficulty falling asleep.
 Once she does sleep, she awakens after about 4 hours.
 OTC antihistamines help her sleep, but she is groggy the following day.
• History: Lewy body dementia
• Physical examination: no findings that might contribute to insomnia
• Which one of the following is the most appropriate initial treatment for this patient?
A. Mirtazapine
B. Ramelteon
C. Trazodone
D. Zolpidem
E. Melatonin

ANSWER: E
Sleep is disrupted in up to 70% of patients with dementia; the disruption appears to be worse in pts who have Lewy body dementia than other dementia subtypes. Insomnia
affects the quality of life of both patients and their caregivers, increases the risk of wandering, and may accelerate cognitive decline. When a patient with dementia has
significant sleep disruption, nonpharmacologic strategies should be tried before resorting to pharmacologic intervention. Any medical or psychiatric condition that
contributes to insomnia should be addressed. Sleep hygiene should also be discussed with patients and their families. Increasing structure and daytime activities may be
helpful and may be accomplished by enrolling the patient in a dementia-specific day program. Patients should be advised to avoid caffeine or alcohol in the evening, and
limit fluids at night to decrease awakenings to urinate. Exercise, such as a walk in the late afternoon, may also be of benefit.
If nonpharmacologic strategies are unsuccessful and insomnia remains a source of significant distress or causes significant management difficulty, a trial of melatonin
may be warranted. A meta-analysis of melatonin for the sleep disturbance of dementia found it significantly beneficial.
There is minimal evidence for the efficacy of hypnotic agents in dementia, and strong potential for adverse effects. The more sedating antidepressants trazodone and
mirtazapine are preferable to the melatonin-receptor agonist ramelteon. OTC antihistamines are strongly anticholinergic and should be avoided because they may cause
confusion, sedation, and delirium.

Case 3:
• A 72-year-old man has hallucinations that cz him severe distress. The hallucinations are of Civil War soldiers & monkeys, & he fears they will attack him.
 In previous visits, no medical or pharm (eg, antichol agents) causes of psychosis were identified. Behavioral interventions were unsuccessful.
 During a particularly stressful episode, his family took him to the ED, where he was prescribed risperidone 0.25 mg BID.
 He rapidly became more confused and markedly rigid.
 Symptoms resolved after risperidone was discontinued.
• History: early Lewy body dementia
• Which one of the following is the most appropriate treatment recommendation for this patient?
A. Clozapine
B. Haloperidol
C. Lorazepam
D. Quetiapine
E. Rivastigmine

ANSWER: E
Hallucinations and delusions often begin early in the course of Lewy body dementia; up to 75% of patients have these symptoms at initial presentation. The hallucinations
tend to be detailed, often of people or animals, and they often do not resolve over time.
Cholinesterase inhibitors are the only agents with evidence from clinical trials to support their use. In a randomized trial comparing rivastigmine ≤12 mg/d and placebo,
patients Tx w/ rivastigmine had significantly less apathy, anxiety, + fewer delusions and hallucinations. Rivastigmine was well tolerated; the MC SE were those typical
for cholinesterase inhibitors (eg, gastrointestinal upset). A 24-week open-label study of galantamine showed benefit for visual hallucinations and nighttime behaviors and
was also well tolerated. Less is understood about the effectiveness of cholinesterase inhibitors in treating psychotic symptoms severe enough to warrant a visit to an ED,
such as this pt. Still, b/c of its favorable safety and tolerability profile compared with those of antipsychotic agents, a cholinesterase inhibitor such as rivastigmine would
be the most appropriate first-line treatment for this patient.
Antipsychotic use can be hazardous for ps with Lewy body dementia, with increased confusion in 30%–50% of patients, and an exacerbation of parkinsonism that can
sometimes lead to NMS. Because haloperidol is highly a/w EPS, its use in this pt population is CI. If tx with a cholinesterase inhibitor is not effective or tolerated,
judicious use of a 2nd-gen antipsychotic w/ low potential for EPS, such as quetiapine or clozapine, may then be warranted. Because clozapine is a/w a higher SE burden
(eg, sedation, anticholinergic effects) & req freq monitoring of WBC counts b/c of risk of agranulocytosis, quetiapine would generally be considered to be the preferable
next option. The initial dose should be the lowest possible (eg, 12.5 mg nightly), and close monitoring for SE is imperative. Lorazepam has no evidence to support its use
for any neuropsychiatric ssx in any type of dementia & is not an appropriate Tx option for this pt.

Choosing Wisely

3 lists of steps physicians can take to promote more effective use of health care resources
 Don’t recommend percutaneous feeding tubes in pts w/ advanced dementia – offer PO feed instead
o Malnourished = markers of nutritional status is abnl; evidence shows benefit from additional nutrient intake/input
 Don’t use antipsychotics as 1st choice for behavioral & psychological ssx of dementia
o Limit to when Rx measures fail & pt is an imminent threat
o Rx that worsen: antichol, sedatives, narcotics, hypnotics
o Avoid polypharmacy

4
 Avoid Rx to achieve HbgA1c < 7.5% in adults > 65
o More hypoglycemia risk; increased mortality
o Moderate control is generally better
 7.0 – 7.5%  few comorbidities, no geri syndromes, extended life expectancy, newly dx DM
 8.0 -9.0%  extensive comorbidities, limited life expectance, fxn/cognitive impairments
 Don’t use BZD or other sedative hypnotics for insomnia, agitation, or delirium
o 57% elderly have chronic sleep problems
 MC Rx: alprazolam, zolpidem
 Rx  memory loss, confusion, disorientation, MVA, hip fx
o Exceptions for use: EtOH withdrawal, DT, severe GAD
 Don’t use Abx for bacteriuria unless specific UTI ssx (+)
o Increased SE > benefit  rash, candidiasis, diarrhea, swollen mouth, dizziness, inc Abx resistance
o Minimum criteria for empiric Tx for UTI
 UTI w/o catheter
 Acute dysuria or fever + 1 GU ssx
o NEW urgency, freq, suprapubic pain, gross hematuria, CVA tenderness, or UI
 UTI w/ catheter
 1 ssx of: fever, CVA tenderness, rigors, delirium
 Don’t Rx AchEIs w/o periodic assessment for cognitive benefits & GI ssx
o Improve global cognitive fxn, improve short-term global fxn for mod AD
o Does not define periodical
o Consider d/c Rx if no benefit after 12 wks
 Avoid Rx appetite stimulants or high-cal supplements for anorexia/cachexia
o Optimize social support, provide feeding assistance, clarify pt goals/expectations
o Normal weight loss: 0.1 – 0.2 kg (0.22 – 0.44 lb) / yr after 70 y/o
 Involuntary wt loss > 4% of body wt  independent predictor of inc mortality
o Do not use megestrol acetate  ¼ inc appetite, 1/12 inc weight, 1/23 will die
o Dronabinol  FDA approved for AIDS, no long term evidence
o Mirtazapine  typical antidepressant, no evidence of wt gain in absence of depression
o Cyproheptadine  on Beers Criteria – very antichol
o Eicosapentaenoic acid  no studies in elderly, little evidence
o Anabolic steroids  little evidence in elderly & not studied well
o PO liq nutrition supplements  water, sucrose, corn syrup, maltodextrin, few oils/protein, multivitamin
 Not really shown to be better than actual food
 Best: bolster feeding support, eliminate diet restrictions, make meals pleasant, work w/ caregivers to clarify goals & expectations
 Don’t Rx w/o conducting drug regimen review
o Inc risk of: receiving incorrect Rx, adverse Rx SE, nonadherence, cognitive impairment, falls, fxnal decline
o Rx review helps ID unnecessary Rs, potentially harmful Rx, underuse of Rx, opportunities to reduce Rx burden
 Avoid physical restraints to manage behavioral ssx
o Can cz: inc delirium severity/agitation, pressure ulcers, infxns, discomfort, inc risk of serious injury, death
o Exceptions: violence, disruption of therapy
 Use to ensure immediate physical safety of pt, staff member, or others… D/C ASAP!
o Alternatives: IV shields, skin sleeves, abd binders, stop sign, grip-lok

Community Based Care

 Home care organizations: home health care agencies, home care aid orgs, hospices
 Medicare only pays for orgs that are Medicare certified + some qualifications
o Homebound = condition makes leaving home a considerable & taxing effort
o Need for skilled service provided by licensed nurse or rehabilitation prof
 Outcome & Assessment Information Set  tool for setting fees for HHRG
o Completed by home-health agency
o Tracks domain of fxnal status & medical needs
 Severity of pt illness, disabilities, nursing needs, geographic info
 PCP is legally responsible for pt’s care plan
o Determine pt healthcare needs
o Dvlp, certify, & recertify care plan
o Confer regularly w/ team members to address pt care issues & handle documentation
 House calls
o Can ID additional problems not readily apparent in office
o Reduce burden of transportation for pts
o Can be done by Drs, NPs, & PAs  documentation is key – no restriction on # of visits
 Eval of pt’s fxn, caregiver issues, & medical plan are req
 Determine home safety/environment for ADLs & recommend mods to improve fxn
 ID & address caregiver’s needs for counseling, training, support, & education
o Conditions that make it inadvisable
 Caregiver not available to adequately address pt needs
 Pt requires freq lab tests, resp Tx, or IV Rx
 Caregiver stress/burnout
 Unsafe home environment
 Prohibitive expense
 Ethics
o Balance pt autonomy & safety
o Terminal  hospice referral can be indicated
o Neglect or abuse  contact APS

5
 Community based services
o No change in residence
 Adult day care
 Common for pts w/ dementia who need supervision & assistant w/ ADLs
 Form of respite for 1º caregiver
 NOT covered by Medicare; some costs covered by Medicaid or private
 Day hospitals
 Skilled nursing care  IV Abx, chemo, intensive rehab
 COVERED by Medicare (requirements similar to home-health care)
 Program of all-inclusive care for the elderly (PACE)
 Acute & long-term care to low-income, fail older people
 Must meet state-defined requirements
 Pools Medicare & Medicaid funds (optional under Medicaid)
 Managed long-term care programs (MTLC)
 “PACE w/o walls”  partners home care, nursing homes, hospitals to enable elderly to avoid nursing home placement &
dec hospital utilization
 Dually eligible pts w/ chronic conditions that impair ability to live independently
o Provides financing through various capitated payments per member per month
 Home hospital
 Provides complex care @ home to have been hospitalized for acute care
o Access to nurses & Dr on reg basis + episodic care thru on-call system
 In-home technology
 Personal emergency response system
 Devices to aid w/ admin & tracking Rx, monitoring & transmitting VS, connecting pts to care providers through audio &
visual telemedicine screens
 Fully automated system to adjust heating/lighting, remotely open/close doors
 Home robotics under dvlpmt to help w/ ADLs or IADLs
 Computers/smartphones to combat isolation and loneliness
 Telemedicine
o Change of residence
 Senior villages & senior cohousing
 Intentional communities where residents tend to be more independent, healthy, & active
 No a/w provision of medical supervision or health care services
 Assisted living
 Coordination or provision of personal care services, social activities, health-related services, & supervision in a home-like
atmosphere
 Services vary considerably
 Care is typically transitional  avg length of residency is ~2 yrs
o MC need for d/c is need for nursing home care
 NOT covered by Medicare  $800 - $4000 / mo
o Some services paid under supplemental security income & Social Services Block Grant programs
o Some states reimburse under Medicaid
 Group homes
 Houses or apts for 2+ unrelated ppl
 Serve pts w/ chronic mental illness or dementia
 Most are for profit; licensing not req in all states
 Adult foster care
 Room, board, some assistance w/ ADLs by sponsoring fam or paid caregivers
 Covered by Medicaid in some states
 Sheltered housing
 Subsidized by the Older Americans Act for seniors & ppl w/ disabilities
 Cost is sliding scale, up to 30% of income
o May be supplemented by social worker services & activities coordinators
 Includes: personal care assistance, housekeeping, meals
 Continuing-care retirement communities
 Requires entry fee + variable monthly fee
 3 common financial models
o All inclusive – total health care coverage
o Fee for service – payments match level of care
o Modified coverage – covers long-term care to predetermined level

Cases & Practice Questions:


Question 1:
• A 79-year-old woman with osteoarthritis is brought in by her son, who is concerned about a decline in her health.
• Until recently, his mother moved around their apartment independently and fixed simple meals for herself.
• One month ago, she had symptoms of a cold, with runny nose, cough, and decreased appetite.
 The runny nose improved, but the cough has persisted.
 She now has dyspnea, her appetite has declined markedly, and she is increasingly weak.
 She fell several times in the past 2 weeks.
 Her arthritis pain has worsened.
• The son works full-time and is the sole caregiver for his mother.
 He has missed work over the last few days because his mother is less able to care for herself.
 He cannot afford to hire caregivers to be present while he is at work.
• Examination

6
 The patient appears well cared for, and she is alert and oriented.
 Blood pressure 124/56 mmHg, heart rate 112 bpm, respiratory rate 24 bpm, temperature 37.3°C (99°F)
 O2 saturation 86% on room air
 Weight is 2.7 kg (6 lb) lower than at her last visit.
 Normal heart and abdominal findings
 Decreased breath sounds at the right base and bilateral scattered crackles and rhonchi
 No focal neurologic deficits
 She is unable to get up from her chair without assistance.
 She is unsteady and walks slowly, with a shuffling gait.
• Which one of the following is the most appropriate next step in caring for this patient?
A. Admit directly to a skilled-nursing facility for rehabilitation.
B. Admit to hospital to determine cause of decline and to evaluate functional status.
C. Contact Adult Protective Services about possible neglect and inadequate care.
D. Refer to home-care agency for evaluation, home safety assessment, and rehabilitation.
E. Recommend move to an assisted-living facility.

ANSWER: B
This pt’s change in condition & fxnal status + abnl VS, hypoxia, and abnl pulm findings  decline in health. She would benefit from hospitalization to determine the
etiology of her resp illness and to initiate appropriate mngnt. She is a likely candidate for d/c to a short-term nursing facility for rehab to help her regain as much of her
prior fxn as possible.
Short-term nursing care and rehab require a 3-day hospital stay to qualify for coverage under Medicare Part A, and it is not possible to obtain payment for direct admission
to a nursing facility without this hospital stay. A patient is eligible if they need services related to the hospital dx and enters the facility within 30 days of hospital d/c. The
goal of care is to improve the patient’s condition and fxn to a level that would allow him or her to resume living in the community. In contrast, admission to a long-term
nursing facility (“custodial care”) does not require a 3-day qualifying hospital stay; it is a benefit under Medicaid and some private long-term care insurance plans.
Home care is a benefit under Medicare Part A for patients who require intermittent—not daily—nursing care or rehab b/c they are homebound. Hospitalization is not a
prerequisite for the home-care benefit. This patient seems to be homebound because of her gait impairment. However, home care would not address the need for eval of
her resp illness. Home care would also not address the patient’s current need for daily supervision while her son is at work.
Drs are mandated to report suspicion of abuse/neglect to APS. This pt’s son is aware that she needs more support/oversight, & he is actively seeking care. There are no
red flags to suggest abuse or neglect.
Assisted-living facilities offer a range of services related to basic & instrumental ADLs. A move there would be premature for this pt, b/c her decline is recent, and her
potential to regain fxn has not been assessed. An assisted-living facility will also lack the capability to provide appropriate eval of her acute illness. The pt may be able to
con’t living @ home after dx+Tx, w/ adjustments in the environ & care plan.

Question 2:
 Which one of the following statements about assisted-living facilities is true?
A. They are regulated by the Centers for Medicare and Medicaid Services.
B. They offer nursing services.
C. The Green House Model is an example of an assisted-living facility.
D. Assisted-living facilities are not available within continuing-care retirement communities.
E. Long-term care insurance may cover the cost of assisted-living facilities.

ANSWER: E
Many long-term care insurance policies offer a choice between comprehensive coverage and facility-care-only coverage that applies to services or residences not covered
by traditional health insurance or Medicare. Facility-care-only policies cover costs in a residential-care setting (eg, assisted-living facility or nursing home) but not care
in the patient’s own home. As with traditional health insurance, the individual pays premiums based on his or her age, health condition, and desired benefits. As of 2015,
45 states offer home- and community-based waivers that allow some low-income adults to reside in assisted-living facilities as an alternative to long-term care nursing
homes.
Assisted-living facilities and board-and-care homes vary widely with regard to size, services provided, and cost. They are a rapidly growing long-term care option that
combines housing, meals, and support services for adults who need assistance with basic or instrumental ADLs. They are not regulated by the Centers for Medicare and
Medicaid Services. Licensing is at the state level, with regulations and requirements varying significantly from state to state.
Nationwide, 37% of adults in assisted-living fac req help w/ ≥3 ADLs; 42% = dementia, a proportion that will inc. Some assisted-living fac have units specificallly for
ppl w/ dementia, & some have a lock/ alarm system to prevent wandering.
Assisted-living facilities offer Rx management under the direction of the resident’s PCP, but they do not offer nursing services. Some have a nurse on staff to oversee
distribution of Rx and to help determine whether a patient requires urgent care. Residents who require nursing services may arrange for such through a licensed home
health care agency.
Continuing-care retirement communities offer a range of environments—independent living, assisted living, and long-term nursing—within the same location. Residents
usually pay a significant enrollment fee upfront as well as monthly fees for the convenience of staying in the same community as their care needs evolve.
The Green House Model refers to small-house nursing homes that, unlike assisted-living facilities, receive the same reimbursement and comply with the same regulations
as nursing facilities. Initial research suggests that residents spend more time with staff, have better quality of life, and maintain higher functional levels over time at a
lower cost to Medicaid than residents of traditional nursing facilities.

Question 3:
 An 89-year-old man comes to the office with his daughter to establish care.
 His wife died 10 mo ago, & he recently moved in w/ his daughter, a working single mother w/ 2 teenagers
 History: heart failure, Alzheimer disease
 He was hospitalized with HF before moving in with her. He remains weak & unsteady since hospital d/c.
 His daughter has several concerns.
 When he is home alone, he mostly sits on the couch. He cannot use the microwave to reheat meals, & he often forgets to take his Rx.
 He is more engaged when his grandchildren are home, but she worries that he is increasingly withdrawn, isolated, and depressed.
 It is difficult for her to take time from work to bring him to the office.
 He is adamant that he does not want to live in an institutional setting.
 The patient receives Social Security income and Medicare and Medicaid benefits.
 Which one of the following is the most appropriate referral for this patient?
A. Psychiatry consultation for treatment of depression
B. Community senior center for socialization
C. Long-term care nursing home

7
D. Program for All-inclusive Care for the Elderly (PACE)
E. Home hospice care

ANSWER: D
The most appropriate recn for this pt is ref to the community PACE. PACE provides integrated medical and social care based on each person’s care plan. It offers the
same services as an adult day service center, as well as 1º, specialty, acute, and home care; many programs are incorporating on-site mental health care. Transportation to
and from the center would lessen the pt’s reliance on his daughter for appts and would dec caregiver stress. Given his HF and memory lapses, he would benefit from
consistent f/u & Rx dosing facilitated by PACE. The program has demonstrated improved outcomes in hospitalization rates, mortality, and need for nursing-home
placement.
The patient = a widower with physical illness and fxnal impairment, is @ risk of depression. Before referral for a psych consult, a brief depression screen should be admin
(eg, Geriatric Depression Scale, Cornell Scale for Depression in Dementia, or the Pt Health Questionnaire [PHQ-9]) in the office w/ the pt alone and w/ further input from
his daughter.
A community-based senior center generally provides activities, socialization, links to community resources, and sometimes meals. It would provide the socialization and
structure this patient needs but not the health oversight.
Adult day service centers provide social activities and, usu, on-site health services, clinical assessment and monitoring, assistance with Rx management, and maintenance
PT. The program would provide health oversight (and possibly transportation to and from the center) for this pt, but it would not address the difficulty of getting him to
and from his PCP’s office. Still, compared with other long-term service providers, adult day service centers are among the most cost-effective programs within Medicaid
waiver or state plans and would be a good option for this patient if PACE is not available. The centers are regulated at the state level. Since 2009, cuts in Medicaid funding
have led to significant restrictions on coverage in many states.
This patient may meet criteria for admission to a long-term care facility, but he has specified that he does not want to live in an institutional setting. Despite HF and AD,
he remains ambulatory, and his current life expectancy > 6 mo. Hospice care at home would be appropriate if class III/IV heart failure or end-stage dementia develops.

Delirium

 D/o of attn & awareness that dvlps acutely & tends to fluctuate
o Acute confusional state
o Acute mental status change
o Altered mental status
o Toxic or metabolic encephalopathy
 Incidence is high in elderly
o 1/3 inpatient 70+; ½ delirious on admit
o > 75% in ICU
o Up to 85% at end of life
 Dx using Confusion Assessment Method (CAM)  bedside assessment tool
o Req features 1 + 2 and either 3 or 4
 Acute change in mental status or fluctuating course
 Inattention
 Disorganized thinking
 Altered level of consciousness
o CAM-ICU
 4 features of CAM for non-verbal pts or ICU pts
o B-CAM
 If (+) attention screen  do B-CAM
 Adapted for non-ICU settings (mostly ED) – esp good for verbal pts
o 4AT
 Non-CAM based assessment for gen med pts
 Brief series of questions + observations using tally pts
o 3D-CAM
 3 min dx assessment for CAM-defined delirium
 1 positive item triggers feature
 Spectrum  hyperactive, agitated, mixed, hypoactive ( ≥ 50%)
o CAM-S = severity scale
 Short & long forms available; excellent predictive validity for imp. Outcomes
 Neuropathophysiology
o Cholinergic def
 Antichol Rx overdose  reverse w/ physostigmine
 AchE-I have NOT been effective in preventing or treating delirium
o Inflammation
 Esp in post-op, CA, & infxn
  CRP,  IL-6,  TNF-
 Can break down blood-brain barrier  Rx & cytokines to CNS
 Neuroinflammation can damage neurons  long-term cognitive effects
 Cz’ed by sum of predisposing & precipitating factors (greater factors = fever precip req to cz delirium)
o Predisposing factors
 Advanced age, dementia, functional impairments in ADLs, medical comorbidity, h/o EtOH abuse, male sex, sensory impairment (dec
vision, hearing)
o Precipitating factors
 Acute cardiac or pulm events, bed rest, drug withdrawal (sedatives, EtOH), fecal impaction, fluid/electrolyte disturbances, indwelling
devices, infxn (esp. resp, urinary), Rx, restraints, severe anemia, uncontrolled pain, urinary retention
 Post-op delirium
o Peak onset: POD #1
o Peak prevalence: POD #2
o A/w post-op pain, anemia, use of sedatives/opioids
o Measured by declining performance on serial testing w/ neurocognitive battery
 Evaluation
o Focus hx on course of cognitive changes & Rx review
8
o PE: VS, O2 sat, gen medical eval, neuro/mental status exam
o Helpful labs/tests:
 CBC, electrolytes, renal fxn tests, UA, urine tox, LFTs, serum drug levels, ABG, CXR, ECG, culture, cerebral imaging (if trauma or
new focal deficits), EEG & CSF (if new seizure activity or ssx of meningitis)
 Treatment – failure to tx is costly, life-threatening, can cz loss of fxn
o ID & Tx reversible contributors
o Maintain behavioral control
 “Social restraints”  consider sitter or allow fam to stay
 Rx if absolutely necessary  high potency antipsychotics (haloperidol) in low doses
 CI in Parkinson Dz, Lewy-Body Dementia, or hx of neuro malignant syndrome
o Substitute 2nd gen antipsychotics (Quetiapine)
 Assess for akathisia & EPS
 Monitor for QT prolongation, torsade de pointes, NMS, withdrawal dyskinesia
 Use BZD for sedative/EtOH withdrawal & hx of NMS
o Anticipate & prevent comp  UI, immobility, fall, pressure ulcers, sleep disturbance, feeding d/o
 Prevention is key!
 Hosp Elder Life Program (HELP) Intervention: cognitive impairment, sleep deprivation, immobility, sensory impairment,
dehydration
o Restore fxn  hosp environment, cognitive reconditioning, ADL status, fam edu, d/c planning

Cases & Practice Questions


Case #1:
• An 86 y/o F is admitted to the hosp from a nursing home b/c over the past 2 d  increasingly lethargic.
 She sleeps on and off throughout the day, and is slow to respond to questions.
 She is normally alert and oriented, pleasant, and actively involved in her care.
• History: HTN, type 2 DM, paraplegia from a car accident 30 years ago, recurrent pressure ulcers
• Examination in emergency department
 Temperature 39.3°C (102.8°F)
 Blood pressure 88/40 mmHg
 Heart rate 100 bpm
 Behavior fluctuates between lethargy and mild agitation.
 She provides no useful history.
• Which one of the following is the most appropriate next step in her care?
A. Obtain computed tomography of the head with contrast.
B. Administer a high-potency, low-dose antipsychotic agent.
C. Perform physical examination and order laboratory tests.
D. Transfer to ICU for observation.
E. Obtain psychiatric consultation.

ANSWER: C
The acute onset of a change in cognition with fluctuation and AMS  pt most likely suffers from delirium. A careful H&P and appropriate labs are essential for ID’ing
and managing the underlying cz. The process requires an understanding of predisposing and precipitating RF & a sense of urgency regarding the potential long-term
impact delirium has on pt outcomes. In addition, the process requires extra patience & empathy when it involves a person who is unable to adequately interpret their
environment & express their needs.
This pt has predisposing RF for delirium (advanced age, high severity) & acute precipitating RF (possible infxn, immobility), some are modifiable. B/c most treatable
cz’eslie outside the CNS, they should be investigated 1st. Given the course of her illness, labs (CBC, electrolytes, kidney fxn, U/A), CXR, & cultures are warranted.
CT of the head would be warranted if the pt displayed any neuro findings, seizures, or further deterioration in mental status. In their absence, additional imaging is of 2º
importance to performing the H&P, and laboratory testing. Transfer to an ICU for closer observation and obtaining psych consult may be necessary if the pt’s condition
worsens, but they too are 2/2 an effective eval by the pt’s primary team.
Administering high-potency, low-dose antipsychotic agents is not the most important next step, given the risks a/w this class of Rx and the lack of compelling data
supporting their efficacy, particularly in seriously ill older adults. Pharmacologic intervention, such as administering antipsychotic medications, should generally be
reserved for key target ssx. This patient is displaying ssx more consistent with the hypoactive, rather than hyperactive, form of delirium. She is also not behaving in a way
that puts herself or others in danger, and she is not actively suffering from hallucinations or delusions.

Question #2:
 Which one of the following classes of medication is the most common cause of delirium in hospitalized older adults?
A. Angiotensin-receptor blockers
B. H2-receptor antagonists
C. Selective serotonin-reuptake inhibitors
D. H1-receptor antagonists
E. HMG-CoA reductase inhibitors

ANSWER: D
Rx are a common precipitating RF for development of delirium in hospitalized elderly. For dx of delirium, the DSM-5, requires evidence from the history, physical
examination, or labs that the delirium is a direct physiologic consequence of a general medical condition, substance intoxication or withdrawal (from a drug of abuse or
a medication), or exposure to a toxin, or has >1 cause. The exact reason for the particular susceptibility of older adults to drug-induced delirium remains unclear, but it is
believed to be due to a combination of factors, including physiologic changes a/w aging (eg, decrease in lean body mass, lower albumin levels), comorbidities (eg, HF,
dementia), pharmacokinetic parameters that change w/ age (eg, extension of drug half-life b/c of reduced metabolic capacity or dec renal elimination), and
pharmacodynamic interactions among Rx that may enhance toxic effects. Strategies for prevention and management of delirium in hospitalized elderly include careful
Rx review, prevention of drug withdrawal syndromes, avoidance of polypharmacy, d/c or avoidance of deliriogenic Rx, and individualized dosing. Pharmacists are a
particularly valuable resource on the interprofessional team responsible for caring for hospitalized older adults.
The pathophysiology of delirium remains poorly understood. It has long been hypothesized that the 1º mechanism underlying the development of both delirium and
cognitive decline w/ aging relates to central cholinergic deficiency. This theory developed, in part, from observation that overdoses of anticholinergic medication cause
delirium, from increased understanding of the role of acetylcholine in attention, and more recently from epidemiologic, neuroimaging, and biomarker studies. Alternative
hypotheses postulate the role inflammatory mediators (eg, TNF-α, IL-1, and cytokines), neurotransmitters (eg, serotonin, dopamine, norepinephrine), and genetics (eg,

9
apolipoprotein E) play in the development of delirium. Medications that have substantial anticholinergic activity (eg, H2-blocking agents, antihistamines, certain opioids,
and tricyclic antidepressants) should be avoided in older adults.
In a systematic review of the association between Rx and risk of delirium, increased risk was found with opioids, benzodiazepines, dihydropyridines, and possibly first-
generation antihistamines. The review reported uncertainty regarding H2-antagonists, tricyclic antidepressants, antiparkinsonian medications, corticosteroids, NSAIDs,
and antimuscarinic agents. The newer H1-receptor antihistamines, ARBs, and SSRIs have not been reported to be a/w delirium in any of the large cohort studies.
In recent retrospective cohort and nested case-control studies involving 374 U.S. hospitals and 225,028 participants, researchers examined the association between sedative
medications on the Beers list and delirium in hospitalized older adults with common medical conditions. In all, 38,883 (17%) participants received ≥1 sedative medications.
In the cohort study, diphenhydramine, an H1-receptor antagonist (adjusted OR 1.22; CI 1.09–1.36) and short-acting BZDs were a/w greater risk of subsequent delirium.
In the nested case-control study, diphenhydramine, short- and long-acting benzodiazepines, and promethazine were associated with delirium.
In the case of opioids, the potential for development of delirium must be weighed against the benefits of adequate pain control, recognizing that pain has also been shown
to be associated with development of delirium. The role of statins in development of delirium is unclear. Some recent studies have found statins to be a/w reduced risk of
delirium, whereas other studies report no effect of these Rx on dvlpmt of delirium.

Question #3:
 Strong evidence suggests that delirium is an important, independent predictor of all of the following EXCEPT:
A. Death
B. New institutionalization
C. Dementia
D. Functional decline
E. Delusional disorder

ANSWER: E
Although previously considered relatively time limited, there is increasing evidence that the effects of delirium often persist and extend far beyond a patient’s initial
hospitalization. In a recent meta-analysis limited to high-quality studies that controlled for age, sex, comorbid illness, illness severity, and baseline dementia, delirium in
older adults was found to be an important, independent predictor of poor long-term outcomes, including death, new institutionalization, and dementia. More recent research
seems to endorse these findings. For example, the Vantaa 85+ study, which included 553 individuals ≥85 years old, examined the relationships between delirium and
incident dementia, dementia progression, and loss of function at baseline and at 3, 5, 8, and 10 years. The researchers found that delirium increased the risk of incident
dementia, and was a/w worsening dementia severity and deterioration in global function score.
Evidence suggests that delirium is an equally important predictor of functional decline in hospitalized older adults. One study found that both delirium and lower Barthel
ADL Index score were the main risk factors for institutionalization.
In 1 study on the effect of duration of delirium on long-term patient outcome, longer duration of delirium was independently associated with worse global cognition at 3
and 12 months and worse executive function at 3 and 12 months after hospitalization. The severity of impairment in patients who had a history of delirium was similar to
that seen in patients with moderate traumatic brain injury or mild Alzheimer disease. Similarly, in critically ill patients, longer duration of delirium was independently
associated with increased odds of disability in activities of daily living and worse motor-sensory function in the year after an ICU stay. In older adults hospitalized with
hip fracture, after adjusting for covariates, each day of postoperative delirium increased the hazard of dying at 6 months by 17%.
To date, no studies have shown delirium to be an important independent predictor of delusional disorder, although both may manifest perceptual disturbances.

Dementia

 Several d/o that cz significant decline in 1+ areas of cognitive fxning severe enough to result in fxnal decline
 Progressive/disabiling – not part of nl aging
 Epidemiology
 AD  6-8% of >65 y/o
 Prevalence doubles q5y after age 60
 Nearly 45%+ of those 85+ y/o have AD
 Vascular dementia  15-20% (often coexists w/ AD  “mixed dementia”
 Dementia w/ Lewy Body (DLB)  2nd MCC in ppl > 65
 Etiology
 AD  amyloid plaques/oligomers; tau neurofibrillary tangles
 LBD & Parkinson  cytoplasmic alpha-synuclein inclusion bodies
 Frontotemporal dementia  tau or ubiquitin proteins
 Risk Factors
 Possible protective factors: NSAIDs, antioxidants, intellectual activity, physical activity, statin
 Definite RF: Age*, Fhx*, APOE4 allele, Down Syndrome (*greatest)
 Possible RF: Head trauma, fewer yrs of formal education, hx of depression, CV RF (HTN, DM, HLD, obesity)
 Genetics
 Early onset (<60 y/o)  amyloid precursor protein (APP) & pressenilin proteins (PS1 & PS2)  1% of AD
 Late onset
 Apolipoprotein E gene (APOE 2/3/4 on xsome 19)
o APOE4  2 alleles = greater risk (decreasing age of onset in dose-related fashion)
o APOE2  protective

Screening Instruments
Mini-Cog 2 items; Score = 5 Visuospatial, executive fxn, recall
SLUMS 11 items; Score = Orientation, recall, calculation, naming, attention, executive, fxn
30
MoCA 12 items; Score = Orientation, recall, attention, naming, repetition, verbal, fluency, abstraction, executive fxn, visuospatial
30
Folstein; MMSE 19 items; Score = Orientation, registration, attention, recall, naming, repetition, 3-step command, language, visuospatial
30
Functional Activities 10 items; Score = Informant based, executive fxn, ADL, attention, concentration, memory, home safety
Questionnaire 30

 Consider brain imaging (noncontrast CT, MRI, or PET) in work up when:


o Onset < 65 y/o

10
o Ssx are sudden or progress rapidly
o Evidence of asymmetric or focal neuro deficits
o Clinical pic suggests normal pressure hydrocephalus  wet, wobbly, wacky
o Pt had recent fall or other head trauma

DDX:
Normal Aging -Mild decline in memory that DOES NOT affect fxning
-Req more effort & time for recall new info
-New learning is slower, but occurs
Mild neurocognitive d/o -Subjective complaint of decline in at least 1 cognitive domain  noticeable & measurable
(mild NCD) / mild -No impairment in independent living
cognitive impairment -9.4-14.3/1000 person-years convert to AD
(MCI) -50% amnestic MCI = stable or return to normal in 3-5 yrs
DDX Onset Cognitive Motor Progression Labs Imaging
AD Gradual Memory impairment w/ Rare early, apraxia Gradual over 8 – Normal Possible global
difficulty learning new info, later 10 yrs on avg atrophy, small
language, visuospatial (core in hippocampal
early stages) volumes
Vascular dementia Sudden, stepwise Depend on anatomy of Correlates w/ Gradual or Normal Cortical or
ischemia, dysexecutive deficits ischemia stepwise w/ subcortical changes
& slowing common further ischemia on MRI
NCD w/ Lewy Bodies** Gradual Memory, visuospatial, Parkinsonism Gradual, but Normal Possible global
hallucinations, fluctuating ssx faster than AD atrophy
FTD Gradual, usu age Executive, disinhibition None (rare genetic Gradual, but Normal Atrophy in frontal
<60 (hyperorality), apathy, forms a/w ALS) faster than AD & temporal lobes
language, +/- memory
**Parkinson dz + dementia = PD ssx > 1 yr before cognitive ssx
**PD ssx + cognitive ssx at the same time  consider dementia w/ Lewy Bodies

 Depression vs. Dementia


o Primary depression have
 Dec motivation during cognitive testing
 Express cognitive complaints that exceed measured deficits
 Maintain intact language & motor skills
o 50% w/ reversible dementia & depression progress to dementia w/in 5 yrs

 Progression of dementia
o Stage 1: No cognitive impairment
 No memory problems; not evident to PCP during interview
o Stage 2: Very mild cognitive decline
 Pts feel they have memory lapses, esp forgetting words/names/locations of everyday objects
 Problems not evident during PE or to friends/family
o Stage 3: Mild cognitive decline
 Early-stage AD dx in some, but not all
 Friends/family/coworkers start to notice deficiencies
 Problems w/ memory or concentration measurable in clinic or discernible during H&P
o Stage 4: Moderate cognitive decline (mild or early-stage AD)
 PCP can detect clear-cut deficiencies
 Pt may seem subdued & withdrawn, esp in social or mentally challenging situations
o Stage 5: Moderately severe cognitive decline (moderate or mid-stage AD)
 Major gaps in memory & deficits in cognitive fxn emerge
 Some assistance w/ day-to-day activities become essential
o Stage 6: Severe cognitive decline (moderately severe or mid-stage AD)
 Memory difficulties worsen
 Significant personality changes may emerge
 Need extensive help w/ customary daily activities
o Stage 7: Very severe cognitive decline (severe or late-stage AD)
 Final stage of dz  indvs lose ability to respond to environment, speak, &, ultimately, to control movement

Treatment
 1º goal  enhance quality of life & maximize fxnal performance by improving or stabilizing cognition, mood, & behavior
 Nonpharm: cognitive rehab, supportive indv/group therapy, physical/mental activity, reg appts q3-6mo, family/caregiver education/support, attn to safety,
environment mods
 Pharm:
o AchE-I  slows down breakdown of Ach
 Modest delay in cognitive decline
 GI SE = common
 Max dose of donepezil 23mg QD  lots SE w/o improving global fxn
 Use in other dementias
 Widespread use in vascular dementia NOT recommended
 Attn & behavior disturb in LBD can BENEFIT from tx
 Rivastigmine is FDA approved for mild to mod Parkinson dementia
 FTD may WORSEN agitation
o Memantine  decrease glutamate-mediated excitotoxicity
 Modest benefit on cognition, ADLs, & behavior in AD

11
 FDA approved for moderate to severe AD
 Limited effect on cognition & no evidence for widespread use in vascular dementia
 Common SE: constipation, dizziness, HA
o VitE (-tocopherol) – may lower rate of fxnal decline, no evidence of cognitive improvement in AD
o Selegiline – may lower rate of fxnal decline, no evidence of cognitive improvement in AD
o Ginkgo biloba  no benefit in slowing decline in MCI
 Ssx mgmt.
o 1st/2nd gen antipsychotics – use w/ caution to target delusions, hallucinations, paranoia, irritability
o Valproic acid & carbamazepine – limited evidence w/ increased risk of mortality
o BZD & antichol Rx should be avoided

Cases/Practice Questions
Case #1
• A 65-year-old man comes to the office with his wife because she is concerned about his memory. He is a retired engineer.
• His wife offers examples of recent uncharacteristic mistakes that he has made in their finances.
 He forgot to pay the mortgage several months ago.
 In the grocery store, his credit card was denied for missed payments.
 The patient describes his experience.
 Balancing accounts feels more effortful and takes longer than it had in the past.
 He feels overwhelmed by distractions.
 He is frequently unable to find keys and other objects.
 He is often unable to recall names of acquaintances until minutes or hours later.
• Which one of the following is most likely to indicate pathologic neurologic decline?
A. Taking longer to complete routine tasks
B. Forgetting to pay mortgage and credit card bills
C. Having a complaint about memory
D. Experiencing difficulty retrieving names

ANSWER: B
Memory difficulties are considered pathologic if they exceed an individual’s baseline to the point of significantly interfering with daily life. This patient’s forgetting to
pay bills, which is validated by an informant, highlights a memory problem that interferes with daily life.
Taking longer to complete tasks represents a decline in the speed of processing that may occur in normal aging. Having a complaint about memory does not highlight the
hallmark of pathologic aging in relation to disruption in daily living. The patient’s difficulty recalling the names of acquaintances relates to delayed retrieval (“tip of the
tongue” phenomenon), not a problem with learning or encoding. Impaired encoding is commonly associated with abnormal aging.
Changes in cognition occur over a life span and do not necessarily represent pathology. Some facets of memory, such as the ability to learn a list of words, decline even
in the absence of dementia, whereas vocabulary and knowledge of general facts remain robust.
Some of the normal decline in memory with aging seems to be because of difficulty with recall. The encoding of new information into memory may remain intact, yet
difficulty in retrieval may hinder task performance. With a cue to assist memory retrieval, either explicitly provided by an examiner or serendipitously observed, the
person may later successfully retrieve the memory. Persons with dementia and pathologic memory impairment often have problems with encoding or storing new
memories (or both), and may not benefit from cues. A “tip of the tongue” experience is a common failure of retrieval, in which objects are recognized but their name
cannot be accessed from memory at the time. The person knows that the information is stored in memory yet cannot retrieve it. The phenomenon seems to be category
nonspecific, and it has been described with various types of stimuli, including word, person, and place recognition. The incidence of “tip of the tongue” experiences
increases with age, but its frequency is not associated with overall memory decline.
Some complaints about age-related memory = more suggestive of underlying path than others. A general CC of change in ability to remember is only weakly a/w cognitive
impairment; difficulty following conversations or understanding instructions & getting lost in familiar places have much stronger assoc. The likelihood of underlying
cognitive dysfxn inc as the range of tasks affected by memory increases.
Executive function and processing speed, fine motor control, and visuospatial skills also may decline in normal aging, perhaps even more so than memory. As with
memory, declines in these areas become pathologic when they preclude independent completion of activities of daily living.

Case #2
• A 73-year-old man has memory problems that are increasingly evident to his wife, and he has had several unexplained falls.
• Neuropsychiatric history
 Seven years ago he began to have impaired smell, altered taste, and fitful sleep with recurrent dream enactment.
o Symptoms have progressed.
o The dream enactments wake his wife at night.
 Fluctuating cognitive dysfunction and bilateral arm tremors developed.
 For several years, he has had hallucinations of children in the room. The hallucinations do not frighten him.
o The hallucinations worsened when he was hospitalized for a UTI last year.
o Neuroleptic agents were administered, but his condition deteriorated and he required restraints for several hours.
 Behavioral interventions have been unsuccessful.
• History: hypertension, hyperlipidemia, diabetes, urinary incontinence, constipation
• MRI of the brain shows mild white matter changes.
• Which one of the following medications should NOT be considered for this patient?
A. Clonazepam
B. Donepezil
C. Fludrocortisone
D. Haloperidol
E. Rivastigmine

ANSWER: D
Lewy body dementia is the 2nd MC dementia (after AD) + is likely underdx’ed. It is notoriously difficult to dx before the onset of memory complaints. Subacute
constipation, anosmia/hyposmia, & parasomnia are all a/w a higher risk of LBD. Cardinal features: dementia, visual hallucinations, & EPS. From 60%–80% pts have
fluctuating cognition & alertness + prominent daytime drowsiness, extended naps, & staring spells. The dementia cz’es more prominent impairment in attn, executive
fxn, & visuospatial skills than other dementias. Well-formed or more abstract visual hallucinations occur in 2/3 pts & vary in type. EPS (bradykinesia, akinesia, rigidity,
tremor, and abnormal gait) are seen in 70%–90% of patients.

12
Patients have prominent ANS dysfxn, w/ urinary retention or incontinence, constipation, & impotence. Falls = common, likely due to a combination of OH, EPS, &
cognitive dysfxn.
Tx of LBD is mostly symptomatic. Patients have severe neuroleptic sensitivity, and first-generation antipsychotics, such as haloperidol, should be avoided altogether. B/c
Rx are poorly tolerated, behavior mod, PT, and other nonpharmacologic therapies are preferred first-line approaches and may work for a limited time. Cholinesterase
inhibitors can be tried if cognitive ssx are present. Use of memantine is currently controversial. BZD and melatonin have been used to mitigate parasomnias; benefits of
BZD should be weighed against underlying risk of falls and confusion. Hallucinations and delusions should be treated with the lowest possible dosage of 2nd-gen (atypical)
antipsychotic. Carbidopa/levodopa can be used for extrapyramidal symptoms. A high-salt diet, fludrocortisone, or midodrine may be effective for frequent falls due to
orthostatic hypotension.

Case #3
Which one of the following is true regarding the risk of prescribing psychotropic medications to patients with dementia?
A. First- and second-generation antipsychotics increase both morbidity and all-cause mortality.
B. Second-generation antipsychotics do not increase morbidity and all-cause mortality.
C. First-generation antipsychotics do not increase morbidity and all-cause mortality
D. First- and second-generation antipsychotics increase morbidity but not all-cause mortality.

ANSWER: A
Neuropsychiatric ssx affect nearly all pts w/ dementia at some point during the course of the illness. The decision to tx psychosis in pts with dementia is a common clinical
conundrum, because there is a 1.5- to 1.7-fold increased risk of mortality with use of first- and second-gen antipsychotics. Further, in pts with dementia, use of
antipsychotics increases the risk of mortality and stroke and is a/w adverse cardiovascular and metabolic effects, EPS, worsening cognition, infections, and falls. The
2012 American Geriatrics Society (AGS) Beers Criteria therefore recommends against use of antipsychotics to treat neuropsychiatric ssx of dementia unless
nonpharmacologic options are ineffective and the pt is a threat to himself or others.

Falls

 Fall = coming to rest inadvertently on the ground or at a lower level


o One of the MC events threatening independence of older adults
o Excludes falls a/w LOC
 Sequelae: decline in fxnal status, nursing home placement, increased use of medical services, fear of falling
o ½ who fall are unable to get up w/o help  “long lie”
 Intrinsic RF: older age, cognitive impairment, female gender, past hx of a fall, leg weakness or gait problems, foot d/o, balance problems, hypovitaminD, pain,
PD, stroke, arthritis
o Increased risk of fall w/ increased number of RF
 Single fall  check for balance or gait disturbance
 Recurrent falls or gait/balance disturbance  pursue multifactorial falls risk assessment
 PE Testing
o Functional Reach Test
 Level yardstick secured to wall at height of acromion
 Person stands comfortably w/o shoes & socks so that shoulders are perpendicular to yardstick
 Make fist + extend arm forward as far as possible along wall w/o taking step or losing balance
 Total reach = measured along yardstick
 Inability to reach ≥ 6”  concern + further evaluation
o Up and Go Test (WWO Timing)
 Indv stand up from chair w/o using arms to push against the chair  walk across room (about 3 m)  turn around  walk back  sit
down w/o using arms
 Grades mm weakness, balance problems, gait abnormalities on scale 1 – 5
 5 = severe
 If timed, > 15 sec  increased risk of falls
o Berg Balance Test
 14 test items of balance  timed tandem stance, semitandem stance, ability of pt to retrieve object from floor
 Scores <40  increased risk of falls
o Performance-Oriented Mobility Assessment (POMA)
 Tests balance + gait  ability to sit/stand from armless chair, ability to maintain standing balance when pulled by dr, ability to talk
normally & maneuver obstacles
 Abnl if 1+ pt deduction from 2 or more items

Cases/Practice Questions:
Case #1
• An 85-year-old woman comes to the office for routine evaluation.
 Occasionally incontinent of urine
 Sometimes gets dizzy when she stands
 Fell 6 months ago, without injury
• History: hypertension, declining memory (over past 3 years)
• Medications: hydrochlorothiazide, donepezil, quetiapine
• Physical examination:
 Heart rate, 60 bpm; blood pressure, 150/80 mmHg seated and 130/70 mmHg standing
 She is thin and mildly cachectic.
• Which one of the following is the single most predictive risk factor for this patient’s future fall risk?
A. Dementia
B. Hydrochlorothiazide
C. Prior fall
D. Orthostatic hypotension
E. Quetiapine

ANSWER: C

13
The most predictive risk factor for falls is a history of a fall within the past year. Other RF: mmweakness, gait or balance problems, postural hypo-TN, psychotropic Rc,
dementia, depression, impairment in ADL, incontinence, and polypharmacy.

Case #2:
• A 77-year-old man is evaluated because he fell in the nursing home. He did not sustain any significant injuries.
• History: hypertension, atrial fibrillation, insomnia
• PE: blood pressure, 140/80 mmHg seated and 120/70 mmHg standing; heart rate, 68 bpm and irregularly irregular
• Medications: amlodipine, vitamin D, warfarin, amiodarone, lorazepam
• Which one of his medications is most strongly associated with falls?
A. Amlodipine
B. Vitamin D
C. Warfarin
D. Amiodarone
E. Lorazepam

ANSWER: E
Psychotropic agents are the drugs most strongly a/w falls, increasing the risk of falls by roughly 70%. Nighttime falls among older adults taking sleeping pills
(sedative/anxiolytic lorazepam), are a serious problem. Even short-acting and very-short-acting BZD increase the risk of falls by 80%–90% among hospitalized elderly.
Rx a/w falls to a lesser extent include antiHTN, anticholinergic, & diuretic agents.
Vit D has been shown in a #of studies to be a/w a reduced risk of falls. Warfarin may increase risk of bleeding and serious injury in the case of a fall, but it does not
increase fall risk per se. Amlodipine and amiodarone have a dose-related increase in fall risk by lowering blood pressure and through other CV mech, but this risk is not
nearly as large as that from psychoactive medications.

Case #3:
• An 85-year-old woman has fallen twice in the last 6 months.
 Most recent fall resulted in a hip contusion, from which she has now recovered.
 A personal trainer works with her 3 days per week doing gait and balance training.
• History: diabetes (controlled by diet), osteoarthritis
• Surgical history: cholecystectomy, removal of a cataract in her right eye
• Medication: acetaminophen as needed
• Physical examination:
 Blood pressure, 148/80 mmHg (seated and standing)
 Mild bilateral hearing loss
 Visual acuity: 20/30 on the right and 20/80 with correction on the left
• Based on the results of controlled clinical trials, which one of the following interventions is most likely to help prevent future falls in this patient?
A. Removing the second cataract
B. Wearing hip protectors
C. Taking a vitamin D supplement
D. Wearing non-slide shoes
E. Using a personal alarm system

ANSWER: C
Vit D supplementation was a/w significantly reduced rates of falls. These analyses further note that the effect of vit D on fall reduction was in studies that used both
calcium and vitamin D supplementation and was strongest among indvls w/ very low levels of 25(OH)D.
In persons with bilateral mature cataracts, first cataract surgery was a/w significantly reduced fall rates in the year after surgery. A subsequent controlled trial with the
same group on the second cataract did not produce a significant change in fall rates. Hip protectors are highly effective in dec rates of hip fx, particularly among elderly
in nursing homes. Hip protectors, however, do not reduce falls per se.
Neither non-slide shoes nor personal alarm systems have been shown to reduce the rate of falls. However, a small study of younger adults using Yaktrax®—a non-slip
“tire-chain” device that is secured to the shoe sole—has been shown to be helpful when used in icy conditions.

Feeding and Swallowing

 3 phases of swallowing
 Preparatory or oral phase = voluntary
 Complex activities of mastication & propelling the food bolus to the back of the mouth toward the pharynx
 Pharyngeal phase = involuntary
 Involuntary initiation of the swallow reflex
 Food propelled into the esophagus
 Esophageal phase = involuntary
 Food propelled down the esophagus by the action of sk.mm proximally & sm.mm distally
 Regulated by intrinsic innervation
 Normal aging  taste sensation decreased, but not discrimination, decreased olfactory fxn  will add more salt + sugar
 Types of dysphagia
 Oral dysphagia: difficulty w/ voluntary transfer of food from mouth  pharynx
 Pharyngeal dysphagia: problem w/ reflexive transfer of food bolus from pharynx  initiate the involuntary esophageal phase while simultaneously
protecting the airway
 Esophageal dysphagia: sensation of food being “stuck” after swallow
 Aspiration: misdirection of oral or gastric contents into the airway
 Aspiration PNA = inoculum of bacteria large enough to overcome host defenses arrives in the lung
 Prevention…
 Feeding tube = RF!
 Hand feeding might be better, but only shown in one nonrandomized prospective comparison
 Assessment of dysphagia
 Video fluoroscopic deglutition exam (VDE) by speech-lang pathologist
 Nasopharyngeal laryngoscopy by an ENT
 Interventions
14
 Swallow therapy
 Compensatory (eg: turn head toward weaker side while swallowing)
 Indirect (eg: exercises to improve the strength of the involved mm)
 Direct (eg: exercises to perform while swallowing)
 Diet mods
 Percutaneous endoscopic gastrostomy (PEG) tubes
 Low complications, but substantial mortality
 Medial survival after PEG < 1 yr
 Complications: increased risk of aspiration PNA, metabolic disturbances, local cellulitis, diarrhea
 Oral feedings are better even tho it’s harder and more time consuming
 Avoid Rx appetite stimulants or high-cal supplements for tx of anorexia/cachexia

Cases/Questions
Case #1:
• A 75-year old man has gradually lost 4.5 kg (10 lb) over the last 4 mos; he was unaware that he had been losing weight
• History
 Ketosis-prone diabetes
 Esophageal rupture >1 year ago that required creation of a spit fistula (esophagus to chest wall). (Anastomosis of esophagus to stomach was not
possible.) Hospital stay included prolonged immobility
• D/c to nursing home & has lived there since. Can now propel himself in a wheelchair and ambulate 100 ft with 1 assistant
• Nutrition
 Bolus of commercially available solution via gastrostomy tube QID (unchanged since admission to nursing home)
 With each tube feeding: has uncomfortable sensation of fullness & fecal urgency, & passes loose, non-bloody stool
 Never skips a feeding, but often ends it before completion because of fecal urgency
• He reports no palpitations, night sweats, heat or cold intolerance, anxiety, or depression.
• Physical examination: unremarkable, beyond weight loss
• Lab findings: mild anemia, mild renal insufficiency, hemoglobin A1c is 7.6%. No blood in stool.
• Which one of the following is the most likely cause of the patient’s weight loss?
A. Inadequate caloric intake
B. Malabsorption
C. Occult malignancy
D. Inadequate control of diabetes

ANSWER: A
Inadequate caloric intake is a common cause of wt loss in older adults. Most likely, this patient’s caloric needs were calculated after his prolonged immobility & have not
been adjusted for his improved functional status. Additionally, by d/c the feeding he is not receiving the full amount prescribed.
GI adverse effects, MC diarrhea, occur in 15%–30% of tube-fed patients. Possible causes of diarrhea in this setting include C. difficile colitis and use of Abx and other
Rx. Liquid formulations of Rx admin via gastrostomy tube often contain sorbitol, which can cz diarrhea. Fat malabsorption in the small intestine has also been implicated.
However, malabsorption is unlikely to have caused this patient’s weight loss, given that he had maintained his weight for the previous 8 mo, with the same formula.
Occult malignancy is plausible, but it is unlikely to be the cause of weight loss without other clinical or laboratory findings.
The first step in evaluating for a change in weight is verifying the accuracy of the measurement. In this instance, there has been a steady decline in recorded values over
4 months, making measurement error unlikely.

Case #2:
• A 79-year-old woman has lost approximately 10% of body weight over last 3 months.
• History: Alzheimer disease (diagnosed 6 years ago), bowel and bladder incontinence
• Medications: senna 2 tablets at bedtime
• Lived in nursing fac for 12 mos; wheelchair bound but can transfer w/ assistance; req assistance w/ bathing & dressing
• Speech is dysarthric & limited to single words. Demeanor is typically pleasant. Often observed to be humming or singing.
• Nutrition
• Food intake has declined significantly despite modifications to consistency and content of her diet.
• Family has provided foods that she previously enjoyed.
• Staff have set up her tray to facilitate her ability to eat.
• Which one of the following is the most appropriate next step in managing the patient’s weight loss?
A. Program of careful hand feeding
B. Referral for placement of gastrostomy tube
C. Addition of oral nutritional supplements
D. Evaluation for occult malignancy

ANSWER: A
This patient has end-stage dementia. The most appropriate next step would be instituting a program of careful hand feeding.
Cohort studies have shown that tube feeding of patients with advanced dementia does not prolong life. Further, there is no evidence of benefit in functional performance
or quality of life, no evidence of improved nutrition, and no reduction in the consequences of malnutrition (eg, pressure ulcers). Tube feeding is a/w lower family
satisfaction in end-of-life care for persons with advanced dementia. Risks a/w tube feeding include aspiration, occlusion of or leakage from the tube, and local infection.
Additionally, patients may require restraints to stop them from removing the tube.
PO nutritional supplements are unlikely to be harmful, but there is no clear evidence of benefit from their use in this setting. A systematic review found that high-calorie
supplements may aid in weight gain but are unlikely to improve other outcomes, such as mortality. In this patient’s case, the supplement would still need to be hand fed.
Because the most likely cause of this patient’s weight loss is progression of dementia, evaluation for an occult malignancy is not indicated. Given her poor functional
status, her life expectancy is limited by Alzheimer disease.

Case #3:
• An 81-year-old man vomited overnight and has since had a nonproductive cough, rapid breathing, and fever
• History: severe intellectual disability and schizophrenia; gastrostomy tube because of severe dysphagia
• Lives in a nursing home and is dependent for all activities of daily living. He does not speak.
• Physical examination:
 Appears thin and chronically ill; no sign of respiratory distress
15
 Temperature: 38.5°C (101.3°F)
 Heart rate: 85 bpm
 Blood pressure: 115/70 mmHg
 Respiratory rate: 24 bpm
 O2 saturation 91% on room air
 Cardiac auscultation: regular rhythm and rate, no murmur. Lung sounds are coarse throughout.
• Laboratory findings:
 WBC 19.2/µL, with 77% neutrophils and 9% bands
 Hemoglobin 10.1 g/dL
 Hematocrit31%
 Platelets 258,000/µL
 Sodium 139 mEq/L
 Potassium 3.9 mEq/L
 Chloride 100 mEq/L
 Carbon dioxide 26 mEq/L
 BUN 29 mg/dL
 Creatinine 0.44 mg/dL
• Chest radiography shows diffuse mild interstitial prominence, unchanged from prior examinations.
• Which one of the following is the most appropriate first-line treatment?
A. Ceftriaxone 1 g IM and azithromycin 500 mg via gastrostomy tube
B. Clindamycin 450 mg three times daily via gastrostomy tube
C. Prednisone 60 mg via gastrostomy tube
D. Supplemental oxygen

Answer: D
In this case, initial tx hinges on whether the pt is displaying ssx of chemical pneumonitis or aspiration PNA w/ a bacterial infxn. The dysphagia, reduced consciousness,
and presence of a gastrostomy tube make pneumonitis the more likely diagnosis. Given the temporal relationship between the episode of emesis & dvlpmt of resp ssx,
aspiration of gastric contents is likely. The low-grade fever & increased WBC count w/o major changes on CXR are consistent with pneumonitis. The best course of Tx
for aspiration pneumonitis is respiratory support. Supplemental oxygen would be the first-line therapy for this patient with tachypnea and low oxygen saturation. Positive-
pressure support or mechanical ventilation may be necessary in more severe cases. The patient should be followed clinically, because there is a significant risk of infxn
during recovery from aspiration-induced lung injury. SSX that persist or increase >48 hrs after aspiration suggest aspiration PNA.
Combination ceftriaxone & azithromycin is appropriate Tx for community-acquired PNA  no 1º role in aspiration PNA, (usu anaerobic bacteria that are normal flora
of the mouth).
Clindamycin is a reasonable choice for bacterial infection related to aspiration pneumonia. If this pt were more critically ill, he would require hospitalization, & empiric
IV Abx would be warranted.
There is no proven benefit for corticosteroids in management of aspiration PNA or pneumonitis. Some evidence suggests an increased incidence of gram-negative bacterial
pneumonia a/w use of corticosteroids.

Fragility

 Fragility = chronic, progressive condition w/ spectrum of severity


o Distinct Physiologic syndrome d/t dysregulation of multiple systems
o Most severely frail  irreversible, pre-death phase w/ high mortality over 6-12 mo
o Earlier phases may be responsible to tx
o Fraility predicts fxnal decline & onset & progression of dependency @ end of life
 1º frailty  results from intrinsic aging processes
o Sarcopenia (loss of lean body mass) = central component & key predictor of other clinical manifestations
 Predictors of sarcopenia & loss of strength w/ aging
 Decreased anabolic factors (testosterone & IGF-1)
 Diminished physical activity
 Reduced nutritional intake
 Older age
 2º frailty  a/w end stages of several chronic dz’es
 Frailty = outcome of # of likely unrelated abnl health conditions in an indv
o Multimorbid state  increased risk of mortality & disability
o # of conditions predicts this vulnerability
 Clinical manifestations seen in: strength, balance, motor processing, nutrition, endurance, physical activity, mobility, cognition (possibly)

Diagnosis ≥ 3 of the below:


Characteristic Criteria for frailty
Weight loss Lost > 10 lbs unintentionally last year
Exhaustion Felt last week that “everything I did was an effort” or “I could not get going”
Slowness Time to walk 15 ft (cutoff depends on sex & height)
Low activity level Expends < 270 kcal/wk (calc from activity scale incorporating episodes of walking, household chores, yard wark, etc.)
Weakness Grip strength measured using hand dynamometer (cutoff depends on sex & BMI)

 1st manifestations: weakness, slowed walking speed, decreased physical activity


o Most pts present w/o full syndromic def of fraility – be adept at constructing a ddx
 Intermediate process is dysregulation of: (# of abnl systems = strong predictor of syndrome)
o Inflammation (increased IL-6 & CRP)
o Decreased immune fxn
o Anemia
o Increased glu intolerance, insulin resistance
o Low levels of DHEA-S & IGF-1
o Increased cortisol, low testosterone

16
o Decreased heart rate variability
o Nutritional derangements
 Screen pts to ID risk for adverse outcomes, gauge severity of risks, PPX or Tx??, Track changes in status over time, determine eligibility for palliative care
o Cumulative ssx, illnesses, & disabilities present is useful to characterize aggregate morbidity burden
 Ways to assess frailty:
o Assess # of phenotypic frailty criteria present (out of 5)
 Severe frailty (score 4 -5) + metabolic abnls of low chol + albumin  high short term mortality rates
 Consider palliative approaches b/c poor response to Tx
o Rapid screening/assessment that have been validated (ex: interview-based FRAIL scale)
o Clinical Global Impression of Change in Physical Frailty (CGIC-PF) as clinical composite or “gestalt”
 Draws on clinical judgement & incorporates intrinsic domains (mobility, balance, strength, endurance, nutrition, neuromotor
performance) & outcome/consequence domains (medical complexity, healthcare utilization, appearance, self-perceived health, ADLs,
emotional status, social status)
o Walking speed or grip strength (early manifestations of frailty syndrome)
 Focus of care:
o Exclude any modifiable precipitating cz’es of fragilty
o Improve clinical manifestations
o Minimize consequences
 Prevailing strategies:
o Carefully choose goals
o Optimize abilities needed to reach goals
o Compensate for diminished competencies by increased reliance on other fxns or by replacement
 Prevention of frailty
o Decrease mobility (critical), depression, hospitalization/Sx
o Maintain physical activity & mm mass  critical
 Resistance / strengthening exercise is effective (increases mm mass, strength & walking speed)
 Other forms of exercise is helpful (Tai Chi, aerobic exercise)
o Consumption of Mediterranean diet  lowers risk over 6 yrs in adults ≥ 65
 Attn to preventing nutritional inadequacy is important (protein, cals, micronutrients)
 Some studies say Nutritional supplementation is only effective when added to exercise
 Overlap btwn “failure to thrive” and very severe or end-stage frailty
o Historically used as blanket dx @ admission  old pt w/ range of severe ssx
 Fatigue, poor nutritional intake, wt loss, social withdrawal, decline in cognitive/physical fxn, depression
 A/w poor response to Tx or rehab, inc pressure sores, infxn, diminished cell-mediated immunity, high sx & short-term mortality rates

Cases/Practice Questions
Case #1:
• A 78-year-old man comes to the office because he is bothered by arthritis in his hands, shoulders, and, to a lesser extent, knees. He is concerned that the
osteoarthritis will lead to functional decline.
• Discussion ensues regarding better pain management, and the potential contribution of osteoarthritis to the development of frailty.
• What aspect of the frailty phenotype predicts reduced activity?
A. Weight loss
B. Exhaustion or fatigue
C. Decreased strength
D. Diminished walking speed

ANSWER: B
Fatigue is common among older adults, esp if (+) chronic dz’es; prevalence reports range from 5%–50%. OA has an impact on several components of the frailty phenotype,
& fatigue is one of the earliest affected. It is a significant contributor to decreased fxn in adults with OA. Many factors contribute to fatigue, as outlined in an AGS
“Bedside-to-Bench” conference. In a study of 172 adults with knee or hip OA and clinically relevant fatigue, fatigue was the strongest predictor of subsequent reduced
activity. The relationship between fatigue & activity was strongest for those with high fxnl mobility. A recent study compared two scores, one obtained by subject report
and the second measured as a ratio of change in walking speed to total distance walked. Both fatigability severity measures were significantly correlated w/ physical
activity level, frailty, & gait speed. The methods used to quantify the severity of fatigue should be useful in the design + eval of interventions to inc physical activity in
elderly @ risk of fxnl decline.

Hearing Impairment

 4th MC chronic dz; MC benign but disabling


o 50-100% of nursing home residents have hearing loss
 Nl hearing: sound energy  external ear  TM + auditory ossicles  malleus, incus, stapes  oval window of cochlea  fluid waves stimulate outer hair
cells  stimulate inner hair cells  impulses sent via cochlear neurons to cochlear nuclei  auditory pathways  brain
 Age-related changes a/w hearing loss
o External ear canal walls become thin
o Cerumen is drier & more tenacious  increases likelihood of impaction
o Eardrum thickens and appears duller
o Cochlea hair cells & fibrocytes in the organ or Corti are lost, basilar membrane stiffens, auditory structures calcify, cochlear neurons are lost
o Stria vascularis capillaries thicken, endolymph production decreases, Na+ K+ ATPase activity decreases
 Audiograph = graph representing hearing thresholds
o Horizontal axis: frequency or pitch in cycles/s (Hertz)
o Vertical axis: loudness of sound in decibels (dB)
o Lower tones = bottom of graph; nl hearing (threshold of 0 dB) = top of graph
 Types of hearing loss
o Conductive hearing loss = AC > BC
 External ear path: cerumen impaction, foreign body
 Middle ear path (MC): otosclerosis, cholesteatoma, TM perf, middle ear effusion
o Sensorineural hearing loss = MC from age, noise damage or ototoxicity = AC = BC
17
 Cochlear dz (MC): MC noise damage
 Other cz’es: ototoxic Rx, genotype, vascular dz, occupational/environmental chemical exposures (rare), autoimmune dz (rare), auditory
nn tumors (rare)
o Mixed hearing loss
 Presbycusis = sensorineural hearing loss (usu symmetrical)
o Classified by cochlear path (sensory, neural, strial, cochlear conductive, combined, or indeterminant
o Tinnitus can be an early sign of hearing loss
Sensory Presbycusis -Loss of sensory hair cells in basal end of cochlea
-Slowly progressive loss, beginning w/ higher frequencies
-Difficulty hearing in presence of background noise
-Tx: amplification
Strial Presbycusis -Results from atrophy of stria vascularis
-Onset in 20s – 60s
-Mild – mod hearing loss in most frequencies
-Usu good speech discrimination
-Tx: amplification
Neural Presbycusis -Cochlear neuronal loss ≥50%
-Poor speech discrimination
-Amplification usu not successful
Cochlear Conductive Presbycusis -Cz/ed by changes in cochlear mechanics produced by mass or stiffness changes, or spiral
ligament atrophy
-Audiogram descends gradually over at least 5 octaves
-Speech discrimination impaired

 Hearing loss detection


o Hearing Handicap Inventory for Elderly (screening version)
o Whisper Test
o Handhold otoscope w/ tone generator
o Apps + online hearing tests
 Referrals
o ENT if…
 Asymmetrical hearing loss  tumor of posterior pharynx blocking eustachian tube or auditory nn tumor
 Cerumen impaction not responsive to cerumenolytics, irritation, or manual extraction or those w/ h/o TM Sx or Perf
o Audiologist if…
 Consult to determine the presence & type of hearing loss, recommendation & fit hearing aids, provide auditory rehab
 Assessment includes eval of pure-tone thresholds (both AC & BC), speech recognition thresholds, speech discrimination, middle-ear
fxn
 Medical Tx:
o Bisphosphonates  Paget Dz, otosclerosis
o Corticosteroids, immunosuppressants  sudden hearing loss
 Surgery  Otosclerosis, tympanosclerosis
 Strategies to improve communication
o Ask the listener what is the best way to communicate with him or her
o Obtain the listener’s attention before speaking
o Eliminate background noise as much as possible
o Be sure the listener can see the speaker’s lips:
 Speak face-to-face in the same room
 Do not obscure the lips with hands or other objects
 Make certain that light shines directly on the speaker’s face, not from behind the speaker
o Speak slowly and clearly towards the better ear, but avoid shouting
o Change phrasing if the listener does not understand at first
o Spell words out, use gestures, or write them down
o Have the listener repeat back what he or she heard
 Active Listening Devices
o Pocket-sized, personal amplifiers
o Telephone equipment: amplifiers, vibrating and flashing ringer alert devices, text telephones (TTY)
o Television listening devices
o Vibrating and flashing devices for alarm clocks and timers, smoke alarms, doorbells, and motion sensors
o FM loops, and FM or infrared transmitter-receiver systems for groups

Effects & Rehab of Hearing


Degree of Loss Loss in dB Sounds difficult to hear Effect on communication Amplification or other assistance
needed
Mild 25 – 40 Whisper Difficulty understanding soft Hearing aid needed in specific
speech or normal speech in situations
presence of background noise
Moderate 41 – 55 Conversational speech Difficulty understanding any but Frequent need for hearing aids
loud speech
Severe 56 – 80 Shouting, vacuum cleaner Can understand only amplified Amplification needed for all
speech communication
Profound ≥ 81 Hair dryer, heavy traffic, Difficulty understanding amplified May need to supplement hearing aid w/
telephone ringer speech; may miss telephone calls lip-reading, assistive listening devices,
sign language

Hearing Aids

18
Type Degree hearing Advantages Disadvantages
loss
Completely in the canal Mild to mod -Almost invisible -User’s dexterity may be a problem
-Less occlusion of pinna  more natural sound -Small size may limit features
-Easier to use w/ headphones, telephone -May cost > canal or in ear aids
-Shorter battery life
Canal Mild to mod -More cosmetically appealing -User’s dexterity may be a problem
-Telecoil avail for some mods -Small size may limit features
-May use w/ headphones
In-The-Ear Mild to severe -Ease of handling -More conspicuous than completely-in-the-canal or
-Comfortable fit canal aids
-Options: telecoil, directional microphone -May be difficult to use w/ headphones
-More power than completely-in-the-ear or canal
aids
Behind-The-Ear Mild to profound -Greatest power -More conspicuous
-Options: telecoil, directional microphone, direct -May be more difficult to insert than in-the-ear aids
audio input -Difficult to use w/ headphones
-Earmold can be changed separately
Body Severe to profound -Greatest separation of microphone from receiver -Most conspicuous
reduces feedback -Picks up noise from rubbing on clothing
-Microphone @ chest or on waist, but speech directed
@ ear
Bone Conduction Mild to severe -Bypasses middle ear -Receiver cz’es pressure on scalp, which can be
-Used if ear canal is unable to tolerate aid or uncomfortable
earmold or unilateral hearing loss -Does not correct sensorineural loss
-Being replaced by bone-anchored hearing aids

Cost of devices**
Type of Tech Cost Comments
Assistive Listening Devices $100+ Useful for specific situations
Hearing Aids 1 aid / 2 aids
Digital, economy $1,200 / $2,200 Slower signal processing, fewer options, useful for quiet
lifestyle
Digital, value $1,500 / $2,800 Multiple programs, telecoil usu available
Digital, mid-range $2,100 / $4,000 Multiple programs, telecoil, Bluetooth input
Digital, premium $2,800 / $5,400 Multiple programs, telecoil, Bluetooth input, more rapid
signal processing
**Typically NOT covered by Medicare
**Store w/ battery case open (can drain battery), clean w/ dry cloth, ask dr before cleaning wax from sound input

 Cochlear implant = electronic device that bypasses the fxn of damaged or absent cochlear hair cells by providing electrical stimulation to cochlear nn fibers
o Implantation requires extensive pre-implant testing, post-implant testing, & gen anesthe
o Covered by most Medicare carriers & insurance companies (prior-auth gen req)
o Does not restore normal hearing, but can sense environmental sounds & understand speech easier
o Characteristics of older candidates for cochlear implants:
 Severe to profound sensorineural hearing loss in both ears
 Functional auditory nn
 Short duration of sever hearing loss
 Good speech, language, & communication skills
 Not benefiting enough from other kinds of hearing aids
 No medical CI to Sx
 Realistic expectations about results
 Appropriate support services available for aural rehab after cochlear implant
 Adequate motivation & cognition to participate in aural rehab

Cases/Questions
Case #1:

• An 80-year-old woman comes to the office with her daughter, who is concerned that her mother has recently begun to withdraw from former activities
(attending book clubs, lectures).
 Patient’s self-reported health status has changed from “excellent” last year to “average” today.
• During the appointment she misunderstands some questions, and she appears to struggle to understand some of what is said. Her memory does not seem as
keen as on previous visits.
• In response to direct questions, she states that over the past 2 weeks she has felt a little down.
 She says that if people would include her in conversation, she would probably feel better and be more inclined to engage in the activities she
previously enjoyed.
• Medications: daily multivitamin, daily aspirin
• Physical examination:
 Normal blood pressure and EKG
 MMSE score is 26 of 30.
 Findings from routine laboratory tests are normal.
• Which one of the following is the most appropriate next step?
A. Begin antidepressant therapy.
B. Refer for psychiatry consultation.
C. Refer for neurologic evaluation.

19
D. Administer in-office hearing screen.

ANSWER: D
Hearing loss affects more than 2/3 of older adults in the US, yet it tends to be underrecognized and underreported. Its prevalence increases with age; it is now the third
most prevalent chronic condition affecting older adults. The severity and nature of hearing loss are measured by objective tests and self-report. Deficit in physiologic
hearing function (pure-tone hearing loss) has a weak association with self-reported hearing handicap. Hearing loss is a/w many mortality risk markers, including falls,
smoking, CVD, dementia, and cognitive impairment. It is further a/w poor self-rated health, functional limitations, and excess hospitalizations. Self-reported hearing loss
is also associated with increased health care expenditures, increased risk of emotional distress, and restricted social engagement. Self-reported hearing loss is a key element
in seeking help and is predictive of outcomes with hearing interventions.
Because age-related hearing loss progresses slowly, patients tend not to bring it to their physician’s attention. Screening should be undertaken when a patient expresses
concern or exhibits behavior consistent with hearing loss, such as misunderstanding questions or not responding when addressed by someone outside his or her visual
field. Screening can be done via a handheld audiometer/otoscope (AudioscopeTM), which includes a pure-tone generator that screens for hearing loss at 4 selected
frequencies (0.5, 1, 2, and 4 kHz) and 2 loudness levels (25 and 40 dB HL). In a quiet environment, when the level is set at 40 dB HL (preferred level for screening), the
test is 94% sensitive and between 82% and 90% specific for detecting hearing loss at 1 and 2 kHz. Additionally, self-administered questionnaires, such as the screening
version of the Hearing Handicap Inventory for the Elderly short form (HHIE-S), are reliable and valid. Although likelihood ratios (LRs) for the HHIE-S range from 0.03
to 0.52, its strong a/w self-rated health, subjective social isolation and loneliness, and successful use of hearing aids underscores the value of this instrument for screening.
The gold standard for need of a hearing aid is hearing loss ≥40 dB HL or HHIE-S score ≥10. Diagnostic confirmation of a (+) screening result is typically done by an
audiologist who administers pure-tone and speech tests. The decision for intervention is based on results of this assessment, in combination with self-reported information,
patient motivation, manual dexterity, and cost.
Using an objective measure of hearing loss (whisper test or AudioscopeTM) and a measure of perceived hearing disability (HHIE) is recommended for screening older
adults who describe difficulty hearing (SOE=B), because the measured degree of hearing loss and the patient’s perception of hearing loss correlate with different markers
and outcomes. Persons at high risk, especially those with multiple morbidities, are ideal candidates for routine screening. Measured hearing loss significantly increases
the risk of eventual moderate or severe hearing handicap, further underscoring the value of a two-pronged screening approach (SOE=B). Conditions that place persons at
risk of hearing loss include mild cognitive impairment, depressive disorder, difficulty with basic and instrumental activities of daily living, falls, and poor self-rated health
(SOE=B). Self-reported hearing handicap is associated with increased likelihood of low self-rated health. The relationship holds after adjustment for age, sex, walking
disability, depressive symptoms, and history of arthritis, diabetes, or cardiovascular disease (SOE=B). One randomized controlled trial has demonstrated improvement in
social, physical, affective, and cognitive quality-of-life domains with use of hearing aids. Cross-sectional studies have also demonstrated that hearing aids reduce feelings
of loneliness and moderate the emotional and social consequences of hearing loss. For patients unable to afford hearing aids, there are less expensive options of proven
value. These include hearing assistive technologies and communication strategies that can be taught to the patient and family (SOE=B).
The incidence of depressive symptoms is higher (19.6%) among persons with a self-reported hearing loss than among those with no loss (6%). When compared w/ persons
w/o self-reported hearing loss as indicated by the HHIE-S, persons with a hearing loss have a increased odds ratio of dvping depressive ssx after adjustment for age, sex,
walking disability, use of community support services, living alone, cognitive impairment, and h/o arthritis or stroke. Given the presenting ssx in this case, screening for
hearing loss is more cost-effective than ref to a neurologist or psychiatrist.

Case #2:
• A 75-year-old man has age-related hearing loss, diagnosed by an audiologist. The patient requests advice regarding hearing interventions.
• He has difficulty understanding speech at meetings and in restaurants; he describes “hearing people talk but not making out the words.”
• Score on the Hearing Handicap Inventory for the Elderly (short form) is 25, consistent with significant self-reported handicap. (HHIE-S is a self-reported
questionnaire that quantifies the handicapping effects of hearing loss.)
• Pure-tone test indicates a mild sloping to moderate sensorineural hearing loss most pronounced in high frequencies.
• Speech understanding in quiet environment is excellent, but decreases to fair in a noisy environment.
• Which one of the following is the most suitable intervention for this patient?
A. Cochlear implant
B. Bone-anchored hearing aid
C. Environmental modifications
D. Hearing aid and counseling on communication strategies

ANSWER: D
Presbycusis, or age-related hearing loss, is a multifaceted, degenerative condition that affects >10 million people globally. The primary sites of degenerative changes
include the stria vascularis, sensory hair cells, and spiral ganglion neurons. Degenerative changes also occur throughout the brainstem into the temporal lobe of the
auditory cortex. Modifiable risk factors associated with age-related hearing loss include noise exposure, diet, ototoxic medications, and smoking. Nonmodifiable risk
factors include increasing age, sex, and race. Comorbidities include HTN and CVD.
The age-related hearing loss affecting this patient is not amenable to medical or Sx Tx; hence, hearing aids are the TOC. However, in the US, only 14% of persons with
hearing loss actually use hearing aids. Although use increases with age (from 4.3% among adults 50–59 years old, to 22% among persons ≥80 years old), most older
adults do not use hearing aids.
Recent hearing aid models are digital and include feedback suppression and dual microphones to enhance the level of the speech signal relative to the background noise.
There is growing evidence that hearing aids improve hearing, ability to function, and health-related quality of life (SOE=B). One randomized controlled trial of hearing
aid use found positive effects on functional outcomes, depression, and cognition.
The U.S. FDA recommends that patients have a medical evaluation by a physician, preferably one specializing in diseases of the ear, before they purchase a hearing aid.
Federal law restricts the sale to persons >18 years old who have obtained a medical evaluation from a licensed physician. A fully informed adult may sign a waiver
declining medical evaluation for religious or personal beliefs that preclude consultation with a physician. Persons choosing this option must be informed that the waiver
is not in their best health interest. Hearing aid dispensers and audiologists are required to advise prospective patients of the following medical contraindications to hearing
aid use: unilateral hearing loss of sudden or recent onset (within the previous 90 days); active drainage from the ears within the previous 90 days; acute or chronic
dizziness; air–bone gap ≥15 dB at 500, 1000, and 2000 Hz; visible congenital or traumatic deformity of the ear; and pain or discomfort in the ear. These contraindications
require referral to a physician.
Nonmedical barriers to hearing aid use include sociodemographic factors such as cost and stigma, and personality variables such as motivation and readiness. Severity of
hearing loss is rarely a contraindication for hearing aids. New studies suggest a greater role for cochlear implants in persons w/ moderate to severe hearing loss. Implants
appear to improve speech understanding to a greater degree than traditional hearing aids. The most significant predictors of incident hearing aid ownership include self-
report of hearing handicap (eg, HHIE-S >8) and the hearing loss itself (SOE=C). Difficulty hearing whispered words was highly predictive of incident hearing aid use, as
was difficulty hearing in a restaurant. Given the low usage of hearing aids, communication strategies should be adopted during discussion with older adults who have
hearing loss, such as obtaining the listener’s attention before speaking, facing the listener while speaking, and eliminating background noise.
Cochlear implants are more suitable for persons with severe to profound sensorineural hearing loss who derive minimal benefit from hearing aids. Age-related degenerative
changes in the ear are rarely associated with severe to profound sensorineural hearing loss.
A bone-anchored hearing aid is a device that is surgically implanted in the mastoid bone. The implanted post connects to a speech processor that transforms sound waves
into vibrations that are transmitted to the inner ear via the skull, thereby bypassing the middle ear. Bone-anchored hearing aids are helpful for people who have unilateral
hearing loss, conductive or mixed hearing loss involving the middle ear, or a malformation that precludes use of a hearing aid.

20
Environmental modifications, such as a modified smoke alarm or doorbell ringer, are helpful for persons with more significant hearing loss as an adjunct to hearing aids.
These do not address the primary complaint of persons with age-related hearing loss.

Mistreatment

 Definitions
o Intentional actions that cz harm or create a serious risk of harm (whether or not intended) to a vulnerable elder by a caregiver or other person who
stands in a trust relationship to the elder
o Failure by a caregiver to satisfy the elder’s basic needs or to protect the elder from harm
 Forms of misTx: physical abuse, psychological abuse, neglect, self-neglect, financial exploitation, abandonment
 Impact of screening is not studied, but routine screening in PCP office is warranted
 RF: poverty, dependency for caregiver needs, age, race, fxnal disability, frailty, cognitive impairment
 Caregiver stress
o Level of care req may exceed ability
o Assessment of stress may ID opportunities to prevent misTx
o Indvs w/ dementia + aggressive behaviors can pose excessive caregiving demands & trigger abusive response
o Document caregiver level of skill & understanding of situation
o RF for abuse or inadequate caregiving
 Cognitive impairment in patient and/or caregiver
 Functional decline in patient and/or caregiver
 Excessive dependency of the caregiver on the elder or of the elder on the caregiver
 Family conflict or FHx of:
 Abusive behavior
 EtOH or drug misuse
 Mental illness/Behavioral health issues
 Intellectual disability
 Financial stress or lack of funds to meet new health care demands
 Isolation of the patient and/or caregiver
 Living arrangements inadequate for the needs of the patient
 Stressful events in the family, such as:
 Death of a loved one
 Loss of employment
 Signs of possible misTx:
o Clothing: inappropriate, soiled, or in disrepair
o Poor hygiene
o Deficient nutritional status
o Compromised skin integrity
 Possible ssx of physical abuse
o Anxiety or nervousness in presence of caregiver
o Excessive deference to the caregiver
o Bruises or welts in various healing stages, especially bilateral or on inner arms or thighs
o Fractures, especially in various healing stages
o Cigarette, rope, chain, or chemical burns
o Lacerations and abrasions on face, neck, lips, or eyes
o Repeated, unexplained, or inconsistently explained falls or injuries requiring emergency room visits
o Head injuries, hair loss, or hemorrhages beneath the scalp as a consequence of hair pulling
o Unusual discharges, bruising, bleeding, or trauma around the genitalia or rectum
o Delays in seeking treatment, inconsistent follow-up, or constant switching among doctors
o Statements about abuse by the patient
 Possible ssx of psychological abuse
o Impatience, irritability, or demeaning statements of caregiver
o Ambivalence of patient to caregiver, or high levels of anxiety, fearfulness, or anger
o Unexpected depression or withdrawal
o Lack of adherence to treatment regimen or frequently canceled appointments
o Frequent requests for sedating medication
o Frequently canceled appointments
o Failing to provide social / emotional stim, or restricting or preventing the pt’s nl activities  total social isolation
 Possible ssx of neglect
o Frequent requests for sedating medication
o Frequently canceled appointments
o Failing to provide social/emotional stim, or restricting or preventing the pt’s nl activities  total social isolation
o Poor hygiene
o Pressure ulcers
o Repeated falls
o Repeated hospital admissions
o Urine burns
o Statements of neglect by the patient
 Possible signs of abandonment
o Evidence that patient is left alone without adequate arrangements for care
o Evidence of sudden withdrawal of care by caregiver
o Statements about abandonment by the patient
 Financial Assessment
o Financial exploitation = unauthorized use of funds, possessions, or property
o Prevalence estimate 5.2%

21
o Fiscal neglect = failure to use the funds & resources to provide for their needs
o Possible ssx of financial exploitation
 Recent marked disparity between the older adult’s living conditions or appearance and his or her assets
 Sudden inability to pay for health care or basic needs
 Unusual interest on the part of caregivers in the older adult’s assets
 Sudden acquisition of expensive possessions by a caregiver who has apparently limited financial assets
 Unwillingness of a caregiver to allow access to the home of an older adult
 Self-neglect
o RF: lower levels of social networking & engagement
o A/w increased rate of 30-day hospital readmissions
o Critical to assess pt capacity to understand risk/benefits/consequences of accepting or enduring misTx
o Dr must honor pt’s right to autonomy & self-determination
o If pt decisionally capable, intervention contrary to his/her choice is gen inappropriate
o Home assessments are crucial along w/ meticulous fxnal assessment
 Institutional misTx
o Aggravated by: poor working conditions, low salaries, inadequate staff training/supervision, prejudiced attitudes, disruptive/insulting behavior by
elder
o State departments are usu responsible for investigations
 Intervention
o Questions to consider:
 How safe is the pt if sent back to the current setting? Do they need to be removed to a safe environment?
 What services or resources are available locally to support the care of this person?
 Do any caregivers have health problems of their own that need attention?
 Does this situation need the expertise of others (eg, medicine, nursing, social work), and if so, who would best serve this person’s needs?
o Imp. to understand exact nature & degree of misTx + whether pt &/or caregiver can/will cooperate w/ eval & Tx
o Consult social work, nursing home, or local health department can be a useful early step
o Report to APS (guidelines vary by state)
 Required in most states by Drs

Cases/Questions:
Case #1:
• An 88-year-old woman with advanced Alzheimer disease has an unexplained fracture of the right zygomatic arch.
 There is no history of witnessed or unwitnessed falls.
 She has no medical conditions that would lead to spontaneous or pathologic fractures.
• The nursing home where she lives conducts a thorough evaluation and determines that the injury is a result of physical abuse.
• Who is the most likely perpetrator?
A. Her assigned primary certified nursing assistant
B. The registered charge nurse assigned to the floor
C. Another resident
D. A family member visiting another resident
E. Maintenance staff

ANSWER: C
Resident-to-resident abuse in long-term care has only recently been systematically studied; it is likely the most prevalent form of abuse of older adults in nursing homes.
The high prevalence of dementia and associated behavioral problems, along with the shared community spaces, likely contributes to the phenomenon. This patient resides
on a dementia unit; such units often include dementia patients with particularly aggressive behavioral problems. The facility appropriately conducted an investigation of
the unexplained injury, and although details are not provided as to how abuse was ascertained, this was the finding. Given the circumstances, another resident is the most
likely perpetrator.
Physical abuse of residents by staff does occur, but it is probably rarer than portrayed in the lay media, and less common than verbal and physical resident-to-resident
abuse. Unlike in previous eras, nursing homes are highly regulated, and personnel undergo both intensive training and CBC as a matter of law. Although this does not
eliminate staff-to-resident abuse, it decreases it.
Visiting family and maintenance staff have far less direct contact with residents and are not plausible choices.

Case #2:
Which one of the following forms of abuse is most common according to epidemiologic surveys that query older adults directly?
A. Physical abuse
B. Abandonment
C. Neglect
D. Sexual abuse
E. Financial exploitation

ANSWER: B
Unlike young victims of domestic violence, older adults have a higher burden of chronic illness, leading to false-positive and false-negative determinations about whether
an injury is caused by abuse. Examples of FN determinations: fx incorrectly ascribed to osteoporosis, or wt loss attributed to chronic illness rather than to the withholding
of food.
Examples of FP: delusional accusations of theft by patients with AD. B/c of the higher burden of chronic illnes, dx criteria for determination of elder abuse have not been
identified.
Spontaneous long-bone fx have been described in bedbound residents in the absence of physical abuse. Case reports describe accusations of physical abuse by staff from
family, when the fx actually results from severe osteoporosis in pts who have been appropriately turned & repositioned.

Case #3:
• A 77-year-old woman is brought in by her daughter @ the insistence of the director of the elder housing project where she lives.
 Neighbors have complained that an odor is emanating from her apartment and that it is filthy.
 She does not bathe, and other tenants cannot ride in the elevator with her.
• History: diabetes, high cholesterol levels
 She has not seen a physician in 7 years.

22
• Physical examination
 She is disheveled, with dirty clothing, matted hair, and long fingernails and toenails.
 BMI and cardiovascular findings are normal.
• Which one of the following is true of this presentation in older adults?
A. It occurs almost exclusively in patients with dementia.
B. It is the most common reason older people are referred to adult protective services.
C. Executive dysfunction is rarely the pathophysiologic mechanism.
D. It is known as Kapgras syndrome.
E. TheDSM-V includes distinct criteria related to this presentation.

ANSWER: B
Self-neglect in older adults is a common syndrome in which the person demonstrates severe neglect of hygiene, medical conditions, and home environment. Hoarding is
a common manifestation. Pts are brought to medical attention when their condition poses a public health problem, such as hoarding that leads to fire hazards or spoiled
food that leads to rodent infestations.
Dementia is a common cz of self-neglect, but it is not the only cz; many pts who self-neglect have normal cognition. Other psychiatric dx’es have been implicated,
including OCD, depression, & PTSD.
Executive function—the neuropsychologic ability to organize and plan one’s daily activities and responsibilities—is commonly impaired in pt who self-neglect.
Imp. b/c executive dysfxn is common in dementia & depression, which are implicated in self-neglect.
Self-neglect is also sometimes called Diogenes syndrome, after the unclothed and disheveled philosopher who wandered the streets in ancient Greece. Kapgras syndrome
is a delusional disorder in which the patient believes family members or other close confidantes have been replaced with an imposter.
Although hoarding and other manifestations of self-neglect have DSM diagnostic criteria, self-neglect itself does not.

Nursing Home Care

 Demographics
o >1.4 mil Americans in nursing home
o 2.8% Americans > 65 y/o; 15% of nursing home residents are < 65 y/o
o Avg age = 82.6 yrs
 Availability
o Ancillary services provided variable
o Many offer on-site mobile radiography & infusion services
 Postacute care
o Response to decline length of hosp stays & higher care needs of elder
o Integrates acute medical care, long-term care nursing, & rehab
o Services vary (dialysis, orthopedics, ventilators, post-op, rehab, wound care)
 Funding: Medicaid (64%), Medicare (14%), other
o Medicare for SNF
 Predictated on pt fxnal needs & rehab potential to help recovery from acute illness or injury
 Gains in fxn must be carefully documented to ensure payment
 Req 3 day hosp stay
 Prospective payment system (PPS) for Medicare
 First 20 d in SNIF = pay in full
 Days 21 – 100 require copay
 100+ d Medicare will NOT cover
 Staffing patterns
o Nursing  high turnover rate
 Higher quality care = higher nursing hrs, higher ratio RNs:other nursing staff
o Physicians
 Perception: excessive regulations & paperwork + limited reimbursement + undesirable
 Usu internist or FM who devotes < 2 hrs/wk
o Reality: challenging & fulfilling work req excellent clinical skills & sensitivity
o Issues
 Closed-staff model may improve care by facilitating interdisciplinary communication and treatment
 Some evidence suggests lower hospitalization rates in nursing homes that employ a limited # of committed doctors
 In 1 study, quality of Rx use in nursing homes correlated w/ enhanced nurse-Dr communication & regular IPE discussions
 RF for nursing home placement
o Increasing age
o Low income
o Low social activity
o Poor family supports (esp lack of spouse & children)
o Cognitive & fxnal impairment
 Acute to long-term care
o Most residents are admitted from acute-care hosp
o High rates of hosp (MC d/t infxn)
o NPs & Pas work in concert w/ PCP  reduces hosp rates while maintaining cost of neutrality
o Suboptimal info transfer is common
 Missing or illegible transfer summaries
 Omission of Rx meds
 Advance directives not documented
 Psychosocial issues & behavior prob not reported
 INTERACT = interventions to reduce acute care transfer
o Dvlped w/ CMS to improve early ID, assessment, documentation & communication
o Goal: reduce freq of hosp transfers
o INTERACT II reduced acute hosp admission by 17%
 Omnibus Budget Reconciliation Act (OBRA)
o Passed 1987 to set training guidelines & minimum staffing req
23
o Mandates that each indv receive & be provided necessary care + services to achieve & maintain “the highest practicable physical, medical, &
psychological well-being” can be obtained
o Bolstered resident’s rights (limited use of restraints, limited psychoactive Rx)
o Initiated Minimum Data Set (MDS)
 Periodic comprehensive clinical assessment of all residents
 Used to compile nursing facility quality measures such as pain, pressure ulcers, weight loss, depression, rates of vaccination, restraint
use, and UTI
 Identification of current or potential problem triggers review of diagnostic and therapeutic protocols
 In 2010, CMS updated the MDS to version 3.0
 MDS 3.0 includes five-star quality ratings of nursing homes
 Included as part of the publicly reported quality measures
 Helps consumers/families/caregivers make comparisons about nursing homes & areas of strength/concern
 Based on 3 sources of data: 1) Facility’s health inspection survey results, 2) Staffing levels, 3)Quality measures
 In 2015, nursing home ratings included the percentage of a facility’s residents who are prescribed antipsychotic drugs
o Req documentation of the need for all Rx, particularly psychoactive Rx
o Other info:
 Use of restraints in nursing homes has decreased significantly, registered nurse staffing has increased, and training requirements for
certified nursing assistants have been est. Assisted-living facilities do not operate under such all-inclusive mandates, which some believe
contribute to the significant variability of care practices & quality of care.
 The facility must ensure that the resident optimally improves or deteriorates only within the limits of that resident’s right to refuse
treatments and within the influence of their illnesses and normal aging. When a resident declines (or does not improve), a survey team
may investigate whether the decline was avoidable.
 The Minimum Data Set (MDS), which surveys a host of clinical issues thought to directly relate to the quality of resident care and thus
considered pertinent to effective care planning. A resident’s medical regimen must be consistent with the assessment compiled in the
MDS. CMS also uses the MDS for individual facilities to compile nursing-facility quality measures data, which are reported publicly
on the CMS Web site
 OBRA requires that a state agency must screen and preapprove the admission of individuals with intellectual disability or serious mental
illness to a nursing facility. This screening is done to ensure that the facility can provide appropriate programs and services to meet the
individual’s needs.
 Legislation in the Nursing Home
o Each federal regulation is given a tag number  “F-tag”
o Adherence is assessed by mandatory site visits q 15 mo
o Failure = “deficiency”
o Penalties imposed based on nature & severity
 Medical director role (must be licensed Dr)
o Influences the quality of the physician practice by:
 Coordinating medical care that meets current standards for care in the nursing home
 Setting quality standards & specific policies & procedures in concert w/ med staff
 Ensuring compliance w/ govt guidelines
 Working w/ admin + director of nursing to foster effective team care & appropriate CME
 Medication Regulation
o OBRA req monthly evals of Rx by PharmD
 Req clin documentation demonstrating indication for all Rx
 Psychoactive Rx: gradual dose reductions mandated unless clinical CI exists & is documented
o Unnecessary Rx are:
 Rx given in excessive doses
 For excessive periods of time
 W/o adequate monitoring
 W/o adequate indications for use
 In the presence of adverse consequences indicating the need for dose reduction or d/c
 Challenges
o Heterogeneity of residents (req individualized approaches)
o Atypical & subtle presentation of illness
o Limited access to biotech
o Dependence on non-Dr for pt eval
o High prevalence of cognitive impairment
o The need to involve families in care plans & providing them education and psychosocial support
o Ethics & legal concerns
o Intense regulatory oversight
 Models of nursing home care
o Eden Alternative
 Focuses on dvlpmt of collaborative partnerships btwn caregivers & elder
 Dvlpmt of human habitat w/ continued contact w/ plants, animals, & kids
o Wellspring Model
 Dvlpd learning collaborative alliances btwn nursing homes to share management, training, & data systems
 Goal of implementing best practices through empowerment of frontline workers
o Green House Model
 Small groups (6-10) of elder residing in small non-institutional homes in residential neighborhoods
 Care provided by empowered direct-care staff (total care + meal prep)
 Dr Responsibilities
o Comprehensive admission assessment
 NPs can provide initial H&P for long-term ppl who do not req a skilled level of care
o Dvlpmt of care plan
 Subsequent visits may be done by mid-level
 Face-to-face visits req q 30d for 1st 90 d after admit then q 60d after
o Periodic monitoring of chronic health problems
24
o Prompt & thorough assessment of acute medical problems or change in fxn
o Communication w/ interdisciplinary team, resident, family for new dx & Tx plans
o Periodic review of all Rx
o Optimization of quality of life & fxn
o Determination of each resident’s decision-making capacity & assistance in est advance directives
o Physical attendance to each resident + documentation

Cases/Questions
Case #1:
• An 82-year-old woman is preparing for discharge to a skilled-nursing facility after hospitalization for pneumonia.
 She became deconditioned in the hospital and now needs assistance when she walks.
 She still requires oxygen.
• Before admission she lived independently, with minimal support from her daughter.
• The patient asks about the likelihood of discharge from the nursing facility back to her own home.
 Her daughter is unable to care for her at home.
• What is the percentage of residents discharged back to the community within 3 months of admission to a skilled-nursing facility?
A. 10%
B. 30%
C. 50%
D. 70%

ANSWER: C
One-half of all persons admitted to a nursing facility are discharged within 3 months. Most are admitted for restorative therapies, either rehabilitation and convalescence
after an acute illness, or wound and ostomy care after a surgical procedure. Several factors make the 3-month time frame important for distinguishing short-stay from
long-stay residents.
Short-stay residents most often extend their convalescence after an acute illness until they are better able to return to the community. These residents require more services
from all disciplines, and while they are in the facility, their medical regimen is adjusted, they receive therapies to help them adapt to current capabilities, families prepare
for the changed circumstances, and additional services are arranged and equipment is procured. DR, NP, or PA visits are required at least Q mo by regulation, but the
residents are often seen at least weekly. Medicare supports this need by funding postacute care at 100% for the first 20 days, after which a co-payment is required for up
to 100 days or until the maximum benefit is reached. Few residents require the full 100 days to reach a plateau; the usual length of stay is approximately 1–2 months.
Care beyond 100 days is considered long-term and is not supported by Medicare. The costs—an average of approximately $90,000 per year—are assumed by the patient
on a private-pay basis until the patient has depleted personal resources and becomes eligible for Medicaid. Because of the financial incentive to preserve the patient’s
assets, when the determination is made that no further progress is expected and the Medicare subsidy will cease, patients and families generally seek discharge to home
if possible.
Of the remaining 50% of patients admitted to a SNF, approximately 25% are transferred back to the hospital b/c of clinical decompensation, <10% die w/in the first 3
mos, and approximately 20% do not recover sufficiently to return home and decide to remain in the institution for long-term care.
Long-stay residents require mostly custodial care, with monitoring of their medical regimen and progression of their underlying disease. The expectation is that these
residents will remain in the nursing home. Indeed, their average length of stay is >800 days, and the likelihood of community discharge is <10%. Therefore, in a cross-
section of the nursing home population, approximately 80% of residents are there for long stay.
Recent observations suggest that this dynamic will soon change: the average occupancy rate (82%) has declined over the past few years. The use of postacute services
has been shown to be the major determinant of Medicare cost variations among hospital referral regions, and there is an increasing focus on utilization in high-cost sectors.
At the state level, health policy analysts are recognizing that long-stay residents often improve over time, and Medicaid programs are exploring custodial care in assisted-
living facilities at a substantial savings. However, based on the 2013 Jimmo v Sebelius settlement agreement, facilities may be able to maintain payments for skilled-level
services as long as the patient’s condition warrants this level of care, regardless of the potential for improvement. The burgeoning movement to expand health care in the
home will also affect the nature of the institutionalized population over the next few years.

Case #2:
Which one of the following is the most common syndrome requiring medical care in the nursing home population?
A. Arthritis
B. Chronic obstructive pulmonary disease
C. Heart failure
D. Dementia
E. Metabolic syndrome

ANSWER: D
1/2 of patients admitted to a nursing home have a dx of dementia; estimates from population studies show that 70%–80% of residents are affected. The slow progression
of these conditions and the paucity of modifying therapies define the course of decline and outcome in this population. The disease spectrum includes Alzheimer disease,
vascular dementia, and related disorders (including Lewy body dementia, frontotemporal dementia, traumatic and anoxic brain injuries, dementia related to toxicity from
alcohol and other illicit drugs, and dementia a/w longstanding psychiatric disease & developmental disorders). Each greatly affects cognitive function, behavioral
aberrancy, & functional dependency.
Although arthritis is ubiquitous in frail older adults, issues related to arthritis decline with diminished functional ability (approximately 60% of long-stay residents are
“chairfast”) and greater responsiveness to non-narcotic analgesics. The effort to decrease the use of opiates in older adults is reinforced by the effectiveness of
acetaminophen, the benefits of exercise (including passive range of motion), and the response to lower-level schedule II drugs. The principles of palliative care have also
influenced prescribing patterns, from as-needed dosing to standing regimens, which tend to require lower doses and have greater efficacy.
Nearly 40% of long-stay residents have heart failure or coronary artery disease. However, because neither chronic obstructive pulmonary disease nor heart failure affects
cognition, institutionalization is usually sought late in the course of these conditions, when advanced physiologic compromise is challenging. Survival is not usually
prolonged after admission. If these diagnoses are comorbidities to a diagnosis of dementia, compliance is better assured for both medications and diet in the nursing home,
and the prognosis is determined more by the patient’s condition on admission.
Because metabolic syndrome is amenable to medication and diet, the structured life in the institutional setting makes control of metabolic disorders much easier than in
the community. Few long-stay residents have physiologic parameters outside acceptable ranges. Once monitoring targets are reached and the regimen is stabilized, there
is usually little need for adjustment until the patient loses weight as part of a terminal decline.

Case #3:
Which one of the following is most associated with decreased transfer rates from skilled-nursing facilities to hospitals?
A. Increased use of hospice services and advance care planning directives
B. Financial incentives to decrease transfer rates

25
C. Early identification of illness and increased communication between providers
D. Increased diagnostic and therapeutic capacities in skilled-nursing facilities
E. Increased staffing ratios

Answer: C
Nursing-home residents account for 2.2 million emergency department visits per year, or 1.6 visits per patient per year. One-half of these visits lead to admission, costing
Medicare approximately $14.3 billion dollars in 2011 (the latest year for which such data are available). Of the several initiatives to address this issue, the Interventions
to Reduce Acute Care Transfers (INTERACT II) program has been the most successful. Several tools are available to identify changes in condition while they are still
manageable in the nursing home and to help empower the staff to take responsibility for implementing the care plan. The tools include the Stop and Watch checklist,
SBAR (Situation Background Assessment Recommendation), and guidelines specific to the skilled-nursing facility population. Educating families about the capabilities
within the facility and the limits of hospitalization helps to create a sense of trust and realism in their expectations.
INTERACT II originated as a quality-improvement intervention sponsored by the Centers for Medicare and Medicaid Services to assist nursing-home staff in reducing
the use of emergency departments, with their attendant hospitalization, and in reducing the rate of readmissions. INTERACT II was evaluated in an open 6-month trial
among 25 nursing homes in 3 states; hospitalization rates were compared with rates from the same 6-month period in the previous year. Seventeen nursing homes were
rated as engaged with the intervention and had a 24% reduction in hospitalizations; there was a 6% reduction in hospitalizations for the 8 nursing homes rated as not
engaged, and a 3% reduction in a comparison group of 11 nursing homes. The average cost of the 6-month implementation was $7,700 per nursing home; the projected
savings to Medicare in a 100-bed nursing home was >$100,000 per year. The authors suggest that INTERACT II should be further evaluated in randomized controlled
trials, with a mechanism for the nursing homes to share in the savings to offset the costs of implementation.
INTERACT focuses on continuous quality improvement implemented by a team with strong leadership, early evaluation of changes in a resident’s condition, with
management in the nursing home when safe and feasible, improved advance-care planning and use of palliative or hospice care, and improved communication and
documentation within the nursing home and with families and the hospital.

Nutrition & Weight

 Age related nutritional changes: body comp, energy req, macro needs, micro req, fluid needs
 Changes in body comp w/ age
o  Bone mass, lean mass, water content
o  Total body fat, commonly with  intra-abdominal fat stores
o Volume of distribution of many medications changes as a result of these shifts
o Creatinine-based determinations can overestimate renal clearance in older adults
o Cannot generalize well-standardized nutrient requirements of young or middle-aged adults to older adults
 Reduced BMR reflects loss of lean body mass
o Principal determinant of total energy expenditure
o Est based on body wt: 25 – 30 kcal/kg/day
o Avoid overfeeding, while still meeting basal req
 Macronutrient needs
o Protein: 0.8 g/kg/day (may need to restrict w/ renal or hepatic dz)
o Fat: 20-35% of total energy intake per day (w/ reduced chol, sat fats, & trans-fatty acids)
o Carbs: 45-65% of total energy intake / day (w/ complex carbs as preferred fiber source)
o Fiber: 30 g/day (men) ; 21 g/day (women)
 Micronutrient Needs
Nutrient Men Women
Ca++ 1,000 mg 1,000 mg
Mg+ 350 mg 265 mg
Vit D 10 mcg 10 mcg
Thiamine 1.0 mg 0.9 mg
Riboflavin 1.1 mg 0.9 mg
Niacin 12 mg 11 mg
Vit B6 1.4 mg 1.3 mg
Folate 320 mcg 320 mcg
Vit B12 2.0 mcg 1.0 mcg
Pantothenic Acid 5 mg 5 mg
Vit A 625 mcg 500 mcg
Vit K 90 mcg 90 mcg
Iron 6 mg 5 mg
Zinc 9.4 mg 6.8 mg
Vit C 75 mg 60 mg
Alpha-Tocopherol 12 mg 12 mg
Selenium 45 mcg 45 mcg
Potassium 4,700 mg 4,700 mg

 Fluid Needs
o Dec perception of thirst
o Dec response to serum osmolarity
o Reduced ability to concentrate urine following fluid deprivation
o Fluid needs can be met w/ 30 ml/kg/day or 1 ml/kcal ingested
 Dehydration = MC fluid or electrolyte disturbance in older adults
o Common signs: decreased urine output, constipation, mucosal dryness, confusion
 Nutrition screening & Assessment:
o Antropometrics = measures of wt & height
 BMI = wt in kg / height in m2
 Low BMI threshold = 18.5
 Wt loss of 5% in 1 mo or 10% in 6 mo  nutritional risk & morbidity

26
 Predicts: functional limitations, health care charges, need for hospitalization
o Nutritional intake
 Inadequate intake = avg intake of food groups, nutrients, or energy 25 – 50% below a threshold level of the RDI
 Poor intake often indicates illness
 Problems w/ obtaining food commonly contribute to inadequate nutritional intakes among older adults
 Imp to ascertain what the limitation is (resources, transportation, fxnality)
o Lab tests
 Albumin  risk indicator for M&M
 Hypo  lacks sensitivity & specificity as nutritional indicator
o Marker for injury, dz, or inflammation
 Prealbumin  better reflect of short-term changes in protein status (b/c of shorter t ½)
o Largely same limitations as albumin
 Serum Cholestrol
 Low (<160)  in indvs w/ serious underlying dz
o Acquired hypo  nonspecific feature of poor health status that is independent of nutrient or energy intake
o May = inflammation
o Hypoalbumin + low chol = Increased M&M > hypoalbumin or low chol alone
o drug-nutrient interactions
 Rx can modify nutrient needs & metabolism of elderly
 Some Rx can cz anorexia in older adults (even at therapeutic levels)
 Digoxin, phenytoin, SSRIs, CCB (ex: dihydropyridines), H2 receptor antagonists, PPIs, narcotics, NSAIDs, furosemide,
potassium supplements, ipratropium bromide, theophylline
 Some Rx reduce intake by czing inattention, constipation, dysphagia, dysgeusia, or xerostomia
 Some drugs reduce nutrient availability
EtOH Zinc, Vit A, Vit B1, Vit B2, Vit B6, Folate,
Vit B12
Antacids Vit B12, Folate, Iron, Total Kcal
Abx, broad spectrum Vit K
Colchicine Vit B12
Digoxin Zinc, Total Kcal
Diuretics Zinc, Mg, Vit B6, K+, Copper
Isoniazid Vit B6, Niacin
Laxatives Ca++, Vit A, Vit B2, Vit B12, Vit D E K
Levodopa Vit B6
Lipid-binding resins Vit A, D, E, K
Metformin Vit B12, total Kcal
Mineral Oil Vit A, D, E, K
Phenytoin Vit D, Folate
Salicylates Vit C, Folate
SSRIs Total Kcal (Via Anorexia)
Theophylline Total Kcal (Via Anorexia)
Trimethoprim Folate

o Multi-item tools for nutrition screening


 Nutrition screening initiative = 3 interdisciplinary tools to screen for nutrition risk & help eval nutritional status of older adults
 DETERMINE checklist:
 10 item, self-report questionnaire
 ID Risk, but does not dx malnutrition
 Level I Screen = intended for use by health care prof
o Incorporates additional assessment items regarding dietary habits, fxnal status, living environment, wt change,
measures of ht/wt
 Level II Screen = highly trained medical / nutritional prof
o Suggested for use in dx of malnutrition
o Also measures depression and mental status
 Mini-Nutritional Assessment
 18 items on short form of 6 items
 Required admin by trained professional
 Evals risk of malnutrition among frail elderly & ID those who may benefit from early intervention
 Simplified Nutrition Assessment Questionnaire
 Answered by mail or while sitting in waiting room
 RF for poor nutritional status: ETOH/substance abuse, cognitive dysfxn, decreased exercise, depression/poor mental health, fxnal limitations, limited mobility,
transportation limitation, inadequate funds, limited education, medical problems/chronic dz, medications, poor dentition, restricted diet/poor eating habits,
social isolation

Nutritional Syndromes:
Involuntary -50% organ related (CHF, CODP, renal -Wt loss 10 lbs (4.5kg) or >5% of usual body wt -Eval: documentation of wt over time, detailed
Wt Loss failure, etc.) over period of 6 – 12 mo H&P, PE, tx of underlying cz, appropriate f/u
-20% neoplastic -BMI < 17 -Therapy: dietary education, removing restrictions,
-20% idiopathic (includes sarcopenia w/ -Excess loss of lean body mass  poor wound nutritional supplements,
aging) healing, infxns, pressure sores, depressed fxnal multivitamin/multimineral
-10% psychosocial conditions ability, mortality
Obesity -Prevalence increased in all age groups, -BMI > 30 -Goals: achieve more healthful wt to promote
genders, racial, and ethnic backgrounds -A/w HTN, DM, CVD, OSA, OA improved health, fxn, & quality of life
-Combo of diet, behavior mods, & exercise

27
-Adverse outcomes: impaired fxnal status, -Frail  emphasize preservation of strength &
increased health care resource use, increased flexibility
mortality

 Oral Nutrition
o Preventing undernutrition is much easier than treating it
o Enhance food intake by:
 Catering to food preferences as much as possible
 Avoiding therapeutic diets unless their clinical value is certain
 Placing two or more patients together for meals can increase sociability and food intake
 Preparing foods with appropriate consistency, attention to color, texture, temperature, and arrangement.
 Using herbs, spices, and hot foods helps to compensate for loss of the sense of taste and smell and avoiding the excessive use of salt and
sugar
 Avoiding hard-to-open individual packages
 Giving adequate time for leisurely meals
 Nutritional Supplements
o Often decrease food intake, but overall nutritional intake increases due to nutrient quality and supplement density
o Contain macro- and micronutrients
o Available in liquid and bar forms
o Chosen based on patient preferences, chewing ability, or product cost
o Most formulas provide 1–1.5 calories/mL, and many are lactose- and gluten-free
o Use of micronutrient supplements is growing, approximately 60% of older adults take self-prescribed dietary supplements
o Many vitamin & mineral supplements are commonly available at stores and are generally safe except for excessive intake of some such as vitamins
A, D, and iron
o There is poor to insufficient data that many nutritional supplements are beneficial
o Imperative that the clinician obtain information about the patient’s use of all supplements
 Vit D Def  30% of indvs > 70 y/o
o A/w impaired Ca++ absorption & reduced activity levels
o Screen w/ total Vit D
o Supplements up to 4,000 IU (100 mcg) QD are safe
o Repletion is a/w improved physical performance, reduced falls, improved bone healing, & response to bisphosphonates
 Drug Tx for Undernutrition Syndromes
o No reports demonstrating improvement in long-term survival, may cause serious adverse effects
o Adverse effects associated with medications (most used off-label) used for treatment:
o Mirtazipine: Caution with doses >15 mg/d due to hepatic and renal insufficiency
o Cyproheptadine: Can cause confusion
o Megestrol: On Beers List. Increased risk of DVT, fluid retention, edema, and CHF exacerbation. May negate effects of exercise on strength and
function
o Dronabinol: Somnolence and dysphoria
o Human Growth Hormone: Contraindicated in cancer states. Hyperglycemia and fluid retention
o Anabolic Steroids: no significant improvement in strength, function, or a reduction in fractures
 Ethics / Legal shit
o Food and fluids should always be offered to all patients
o The decision to start or to discontinue artificial nutrition or hydration must be considered very carefully
o Competent adults may choose to forgo artificial feeding, just as they have the right to decline any invasive procedure
o Advance directives should be honored unless there is compelling evidence that the individual would have changed his or her mind in the current
o In incompetent adults without advance directives the decision to start or to discontinue artificial feeding should be considered carefully with the
surrogate
o Take into account the risks and burdens of such an action, the risks and burdens of alternative actions, and the evidence to support likely benefits
of the various actions
o Current evidence does not support the use of feeding tubes in patients with end-stage cancer, dementia, or COPD
o Palliative care, including emotional support, is extremely important at the end of life

Cases / Questions
Case #1
• An 81-year-old woman had surgery for a ruptured appendix 6 days ago.
 She responded well to IV fluids and antibiotics and has been afebrile for 4 days.
 She is now on a regular diet.
 Per nurses’ notes, she consumes 50%–75% of her meals.
• Past medical history: unremarkable
• Physical examination
 Alert, oriented, in no acute distress
 Weight: 79.5 kg (175 lb)
 5 kg (11 lb) above weight on admission
 Body mass index: 28 kg/m2
 Lungs are clear.
 Jugular venous pressure: <7 cm
 3–4+ pretibial and presacral edema (not present at admission)
 Surgical wound in right lower quadrant of abdomen
• Laboratory findings: normal for electrolytes, BUN, creatinine, WBC
• Which one of the following would be most helpful for assessing the adequacy of the patient’s nutrient intake?
A. Serum albumin level
B. Serum prealbumin level
C. Daily weighing
D. Resting metabolic rate measured by indirect calorimetry
E. Complete calorie counts for the next 3 days

28
ANSWER: E
Complete calorie counts for the next 3 days would be the most effective way to assess the patient’s nutrient intake. Although her condition is improving, she remains at
significant nutritional risk. Given the seriousness of a ruptured appendix, the emergency sx, & the time it has taken for her to recover gut fxn, she has most likely been
in negative energy and nitrogen balance for most of her hospital stay. Adequate nutrient intake promotes complete wound healing and helps to avert setbacks during
recovery. Any of several formulas can be used to estimate energy and protein requirements. Once the requirements are est the most effective way of determining whether
they are being met is to perform careful calorie counts. The routine daily assessments completed by nursing staff may not be accurate enough to guide clinical decision-
making, especially for a high-risk patient whose nutrient intake is inconsistent.
Neither serum albumin nor prealbumin levels are sufficiently S&S for determining the adequacy of nutrient intake, esp in a pt recovering from an acute inflammatory
insult, Ex: ruptured appendix.
Measuring weight daily would not help in assessing the adequacy of this patient’s nutrient intake, at least in the short term. During recovery, her weight is likely to decline
initially as she begins to mobilize the excess subcutaneous fluid that has accumulated since surgery. It will be important to document her postsurgery dry weight when
and if her edema resolves to document her subsequent weight trajectory after hospital discharge. The trajectory can be used to assess her nutritional health at outpatient
follow-up visits.
Although indirect calorimetry could provide an accurate assessment of the patient’s resting metabolic rate, the test is not generally available, and the formula-derived
estimates of protein and energy requirements are usually adequate to guide clinical decision-making.

Case #2
Which one of the following micronutrient supplements increases healing of pressure ulcers in older adults?
A. Zinc sulfate
B. Selenium
C. Vitamin C
D. Daily multivitamin that contains zinc, selenium, iron, and vitamins A, C, and E
E. No supplement is necessary if vitamin deficiency is not present.

ANSWER: E
There is little evidence that any form of micronutrient supplement promotes healing of pressure ulcers. Because zinc, copper, iron, selenium, and vitamins A, C, and E
are vital to good health and wound healing, it is prudent to ensure that patients with pressure ulcers meet the U.S. recommended daily intake for these micronutrients. If
this cannot be accomplished by a balanced diet or complete nutritional supplement, a multivitamin and mineral supplement can be prescribed until overall nutrient intake
is adequate to meet the patient’s nutritional needs. The most recent nutrition guidelines from the National Pressure Ulcer Advisory Panel (NPUAP) and European Pressure
Ulcer Advisory Panel (EPUAP) endorse this approach.
There is little evidence to support intense zinc supplementation for wound healing in zinc-deficient patients. Indeed, it is difficult to establish a diagnosis of zinc deficiency,
because interpretation of blood and tissue zinc levels is controversial. Risk factors for zinc deficiency include profuse wound drainage, chronic malabsorption, and
uncontrolled diarrhea. If zinc supplements are administered, it should be for a limited time, and tolerance should be monitored. SSX of zinc toxicity include N/V, diarrhea,
headaches, and cramps. Excess dietary zinc can lead to copper deficiency, which may manifest as impaired wound healing, anemia, and neurologic impairment. There is
no evidence that any other micronutrient supplements benefit healing of pressure ulcers.

Case #3
• An 81-year-old woman is admitted to a nursing home after 6 wks in hospital for PNA & acute respiratory failure. She is deconditioned and needs assistance
with all ADLs.
• Hospital course
• Intubation and ventilatory support for 3 days
• Complications: acute kidney injury requiring dialysis, prolonged ileus, pressure ulcers, UTI, C. difficile–induced colitis, mild right-hemisphere
stroke
• At discharge, all catheters, IV lines, and antibiotics were discontinued.
• She can ambulate short distances & tolerates a regular diet.
• No difficulty chewing or swallowing, per recent assessments by dietitian and speech therapist
• Physical examination (at nursing home)
• Vital signs, chest, cardiac, and abdominal findings: unremarkable
• Weight 82 kg (181 lb) (pre-hospitalization: 81.5 kg)
• Pretibial and presacral edema 3–4+
• Unstageable 5 cm × 4 cm sacral pressure ulcer surrounded by non-inflamed, healthy skin
• Muscle strength: mild weakness of left upper extremity
• Laboratory findings
• Hemoglobin 10.5 g/dL
• WBC 7.8/µL
• Creatinine 1.8 mg/dL
• BUN 12 mg/dL
• Albumin 2.7 g/dL

Which one of the following is most appropriate for optimizing the patient’s nutritional status?
A. Start megestrol acetate, 400 mg orally twice each day.
B. Order a complete swallowing assessment.
C. Place a feeding tube and start enteral feedings with a high-protein polymeric (nutritionally complete) formula at a rate sufficient to provide 30 kcal/kg daily.
D. Provide feeding assistance and monitor nutrient intake with complete calorie counts for the next 3 days.
E. Start a high-protein oral nutritional supplement, 240 mL three times daily between meals.

ANSWER: D
This pt has likely been in a profound catabolic state throughout most of her hosp stay and is now @ high nutritional risk. Although her wt has dropped only 1 kg (2.2 lb)
since admin, she has extensive edema that is masking a much more profound loss of body mass. Improving her nutritional status is of prime importance. Her medical
condition appears to be stabilizing & she is tolerating a regular diet, with no apparent chewing or swallowing problems. No information is given as to the total amount of
nutrients or the protein, energy, or micronutrient composition of the food she is consuming. This info is needed to determine whether her current nutrient intake is optimal,
& before a decision as to whether other forms of nutrition support are advisable. The pt’s nutritional needs & preferences should be assessed carefully, her diet optimized,
and any need for feeding assistance addressed. An interprofessional team is required to provide optimal nutritional care.
The use of PO nutrition supplements is controversial and should not be the first recourse. The supplements are effective in increasing wt in older patients, yet there is less
certainty as to whether supplements improve clinical outcomes, and no prospective studies have compared supplements or tube feeding with enhanced bedside care. This

29
is an important issue, because studies have shown that focusing on the quality, timing, and frequency of meals can significantly improve an older patient’s total nutrient
intake. The focus can include working with a dietitian to ensure that the patient’s dietary preferences are addressed, food consistency is modified to minimize chewing or
swallowing problems, energy- and protein-dense foods are provided, and benefits of between-meal snacks are assessed. Adding variety to the menu and attention to
packaging, food color, texture, temperature, and flavor enhance food intake. Restrictive therapeutic diets should not be used when oral intake is inadequate unless there
is a strong clinical indication. Studies have shown that simple measures can lead to a 35%–40% increase in older patients’ energy intake. For example, adding between-
meal snacks has been shown to increase daily intake by 600 kcal and 12 g protein and was associated with shorter lengths of stay among hospitalized patients.
Megestrol acetate would not be a good choice for this pt. Although its use is a/w an increase in wt & appetite in some pt populations, there is considerable controversy as
to the composition of the wt gain, no evidence of benefit regarding other clinical outcomes, & a high risk of serious SE’s when given to frail older adults. Probably as a
result of its strong corticosteroid agonist properties, megestrol acetate, @ a dose of 800 mg/d, negates the benefits of exercise on strength and function in older patients
in recuperative care. Its use in nursing homes is a/w a higher risk of DVT, fluid retention, edema, and exacerbation of CHF. Use of megestrol acetate should be considered
only after all other nutritional interventions have failed.
Because recent assessments by a dietitian & a speech therapist suggested no problems, a formal swallowing study is not indicated. Should repeat bedside assessment or
subsequent monitoring suggest a swallowing problem, formal assessment can be reconsidered. Placement of a feeding tube should be considered if there is clear
documentation that volitional PO intake is inadequate despite optimal nutritional care. There is no evidence that tube feeding improves clinical outcomes of recuperating
patients with a normally functioning gastrointestinal tract, and there is considerable evidence of potential harm.

Pain Management

 1º tenant of geri medicine = relief of pain & suffering + promotion of fxnal status & quality of life
o Common in adults ≥ 65 y/o + ability to tolerate severe pain decreases w/ age
 Common cz’es: OA, DJD, post-sx pain, nocturnal leg pain, pain a/w various chronic illnesses
o Shingles & post-herpetic neuralgia are MC
 Commonly undertreated
o Pt’s minimize ssx, don’t/can’t report, think it’s normal
o Drs may inadequately assess, be reluctant to manage b/c of lack of adequate knowledge, undertx pain
 Pain = unpleasant sensory/emotional experience a/w actual or potential tissue dmg or desc in terms of such damage
o Subjective, idiosyncratic, beyond objective measure  intensity & character are what the pt says
 Acute pain = sudden in onset & expected to last a short time
o Clearly linked to specific bodily insult or injury
 Chronic/persistent pain = pain w/o apparent bio purpose that has persisted beyond the nl tissue healing time (3-6 mo)
 RF for acute  chronic pain: lower SES, vivid memory of childhood trauma, obesity, low level of physical fitness, overuse of joint/mm, chronic illness, lack
of social support, abuse
 Assessment
o International Association for the Study of Pain (IASP)  taxonomy to classify
 Axis I: anatomic regions
 Axis II: organ systems
 Axis III: temporal characteristics, patter for occurrence
 Axis IV: intensity, time since onset of pain
 Axis V: etiology
 Pain intensity scales  use the same scale in the hosp and during f/u
o Unidimensional scales
 Numeric Rating Scale
 0 = no pain; 10 = worst pain imaginable
 Faces Pain Scale = pt chooses facial expression that corresponds to pain
 Wong-Baker FACES pain rating scale w/ foreign translations useful for non-English pts
 Verbal Descriptor Scale
 “no pain” to “pain as bad as it could be”
o Multidimensional scales  McGill Pain Questionnaire; Pain Disability Scale
o Pain Map used to indicate locations of pain on a drawing of a human figure
 Consider referral if pain is erratic, diffuse, or does not conform to an anatomic distribution
 Fibromyalgia  multiple tender points, sleep disturbance, fatigue, generalized pain, morning stiffness
o Myofascial pain is dz’ed by presence of taut band of mm & trigger points
 Pain Syndromes
o Nociceptive  pain d/t activation of nociceptive sensory Rc; often adequately tx’ed w/ common analgesics
 Somatic  well localized in skin, soft tissue, bone  throbbing, aching, stabbing
 Visceral  d/t cardiac, GI, or lung injury
 Not localized, difficult to describe  crampy, tearing, dull, aching
o Neuropathic  irritation of components of the CNS or PNS
 May respond well to nonopioid therapies; responds unpredictably to opioids
 Burning, numbness, shooting pains
o Mixed or unspecified  characteristics of both nociceptive & neuropathic pain
 Common in older adults
 Tx w/ trials of different Rx or w/ combos of Rx along w/ interprofessional collaboration is often beneficial
 Pain in cognitively impaired people
o Observe for possible pain-related behaviors & ask caregivers
o Consider trial of analgesics
o Validated scales req trained evaluators
 Hurley Discomfort Scale, Checklist of Nonverbal Pain Indicators
 Pre-medicate w/ analgesics for procedures & conditions known to be painful
 Remember PO pain Rx take 30 – 60 min to act  give 1 hr before procedure
 IV Rx can be given 5 – 10 min before procedure
o Common pain behaviors
Facial expressions -Slight frown; sad, frightened face
-Grimacing, wrinkled forehead, closed/tightened eyes
-Any distorted expression

30
-Rapid blinking
Verbalizations, vocalizations -Sighing, moaning, groaning
-Grunting, chanting, calling out
-Noisy breathing
-Asking for help
-Verbal abusiveness
Body movements -Rigid, tense body posture, guarding
-Fidgeting
-Increased pacing, rocking
-Restricted movement
-Gait or mobility changes
Changes in interpersonal interactions -Aggressive, combative, resists care
-Decreased social interactions
-Socially inappropriate, disruptive
-Withdrawn
Changes in activity patterns or routines -Refusing food, appetite changes
-Increase in rest periods
-Sleep, rest pattern changes
-Sudden cessation of common routines
-Increased wandering
Mental status change -Crying or tears
-Increased confusion
-Irritability or distress
 Nonpharm therapies
o Pt education & involvement in decisions
o CBT
o Regular physical activity
o Referral to interprofessional pain clinic
 Pharm therapy
o Principles
 Besides pain relief, goals are improved fxn & enhanced adherence w/ rehab
 Risks/benefits considered & individualized to pt
 Try nonsystemic or nonpharm tx first
 Individualize initial dose & rate of titration
 Monitor closely for adverse effects
 Start opioids at lowest dose & titrate slowly
 Do not withhold Rx in pain crisis!
o WHO pain ladder
 Step 1: start nonopioid Rx WWO adjuvants
 Step 2: if pain persists/increases, start weak opioid (Norco) + adjuvants
 Step 3: if pain persists/  strong opioid (morphine, hydromorphone) WWO nonopioid Rx + adjuvants
o Acetaminophen
 Particularly for musculoskeletal pain from OA
 Recommended as first-line therapy for persistent pain
 Preferred maximal dose in older adults is approximately 3 grams every 24 hours
 Lower the dose by 50%, or avoid entirely, in patients @ risk of liver dysfxn, esp w/ h/o heavy EtOH intake
 Admin q 6 hrs for pts w/ a Cr clearance 10–50 mL/min & q 8 hrs for pts w/ a Cr clearance <10 mL/min
 Know all Rx the pt is taking, as acetaminophen is a common ingredient in Rx & OTC drugs to avoid tox
o NSAIDs
 SE: renal dysfxn, GI bleed, plt dysfxn, fluid retention, exacerbation of HTN or CHF, precip of delirium
 FDA cautions use w/ ASA d/t interaction that blocks anti-plt effect of ASA
 Use COX-2 inhibitors w/ great caution d/t class effect of increased CVD risk
 Short term use of topical NSAIDs is safe & effective
o Opioids
 Tx Mod-Severe Pain
 Tx continuous pain w/ 24-hr opioids in long-acting or sustained release formations
o Cover breakthrough pain w/ fast-onset Rx that have short half-lives
o Breakthrough pain typically requires 5-15% of the daily dose (offered q2 to qh4 PO)
 Opioids in renal failure
 Avoid morphine b/c metabolites accumulate (increase dose interval & reduce dose)
 Oxycodone hasn’t been studied well but safer than morphine
 Hydromorphone is acceptable choice for older adults  start slow and go slow
 Barriers to using opioids
 Older adults have concerns  fear that using will  effectiveness in the future, fear of addiction
 Pharmacies may not have it in stock
 Culture & religion & language
 Combat fear of tolerance & addiction
 Avoid withdrawal symptoms by tapering carefully over days to weeks
 If rapid upward titration is required to reduce pain, evaluate the cause of pain:
o Search for new pathologies, exacerbation of known sources
o Consider nonphysical causes of pain
 There is partial cross-tolerance between opioids
o When switching Rx,  dose of the new drug by 50%–65% of the equianalgesic dose
 Patient and caregiver education is vital in successful opioid management

31
 Physical dependence = expected change in pt physiology that occurs while pt is receiving Rx. If stop Rx, will have withdrawal ssx
(restlessness, tachycardia, HTN, fever, tremors, lacrimation)
 Tolerance = change in physiology  need to increase opioid dose over time to achieve adequate analgesic effect
 Psychological dependence (addiction) = psychiatric state = compulsive drug seeking & using w/ disregard of adverse social, physical &
economic consequences
 SE of opioids: constipation, N/V, sedation/fatigue/mild cognitive impairment, respiratory distress (use naloxone sparingly & if < 8
breaths/min or O2 sat < 90%)
o Nonopioid Adjuvant Analgesics
 TCAs (off-label)
 Avoid amitriptyline in older adults, but can use imipramine, desipramine, or nortryptamine
 Optimal analgesia req tx of depression
 SSRIs (better tolerated than TCAs)
 Duloxetine for both antidepressant & diabetic neuropathy
 Venlafaxine off-label for pain & depression
 Anticonvulsants  carbamazepine, gabapentin, pregabalin, clonazepam
 Corticosteroids for assoc swelling, inflammation, tissue infiltration
 IV bisphosphonates for pain from malignant bone mets
 A/w rare cases of osteonecrosis of jaw (particularly w/ dental Sx)
 Tramadol
 Use with caution b/c lowers seizure threshold
 Caution w/ serotonergic Rx
 Tapentadol for mod-severe acute & chronic pain
 Avoid in pts w/ renal & hepatic impairment
 Interventional pain management
o Trigger point injections
 Acute or chronic mm pain a/w bone/mm path
 Q 1 – 2 w until pain relief is attained
o Pulsed Radiofrequency (PRF)
 Various types of pain (mostly nerve) & can last for months

Cases / Questions
Case #1
• A 74-year-old woman has severe bilateral hip pain related to osteoarthritis.
 Over the last mo, the pain has inc such that she can only ambulate w/ a walker and now needs help with toileting.
 She is not a candidate for hip replacement surgery.
• History: OA, mild dementia, CKD (baseline creatinine 2.2 mg/dL), seizure disorder, heart failure (ejection fraction 25%)
• Rx: acetaminophen 650 mg TID, furosemide 40 mg/d, carbamazepine 200 mg TID, donepezil 10 mg/d, lisinopril 10 mg/d, metoprolol succinate 100 mg/d
• Which one of the following is the most appropriate pharmacologic treatment for the patient’s pain?
A. Carbamazepine titrated to 400 mg three times daily
B. Ibuprofen 400 mg three times daily
C. Immediate-release oral morphine 7.5 mg every 8 hours, held for sedation
D. Oxycodone 2.5 mg every 8 hours, held for sedation
E. Tramadol 50 mg every 6 hours as needed for pain

ANSWER: D
Despite the dosage of acetaminophen, this patient continues to have severe noncancer pain that is causing functional impairment and diminished quality of life. Treatment
with opioids should be considered. Because her renal function is impaired, opioid half-life may be extended in this patient, and thus dosing every 8 hours should be
considered. Oxycodone at a low starting dose (2.5 mg) would be appropriate, carefully titrated upward if needed. It has been shown to have efficacy in treating pain from
OA. Given that the patient has continual, daily pain, around-the-clock dosing withhold parameters for sedation is appropriate. Because of her mild dementia, “as needed”
dosing could result in undertreatment of the pain if she does not readily call nursing staff. A bowel regimen should also be started to avoid opioid-related constipation.
The patient should be monitored closely for SE and efficacy.
PO morphine has similar efficacy to oxycodone but is not appropriate for this patient, because her renal impairment may cause metabolites to accumulate and cause
neuroexcitatory adverse effects.
Tramadol is a centrally acting analgesic that possesses weak opioid agonist activity and can be helpful in patients with chronic pain. However, tramadol can lower the
seizure threshold, and is CI in patients with a h/o seizures. It should also be used cautiously in patients who take other serotonergic Rx. Further, using pain Rx on a PRN
basis risks undertreating this patient’s pain.
Anticonvulsant agents (carbamazepine) can be helpful in Tx of neuropathic pain, but carbamazepine is unlikely to provide significant pain relief for this patient’s somatic
nociceptive pain related to osteoarthritis.
Ibuprofen and other NSAIDs are effective in Tx chronic inflammatory pain, including OA. However, in this patient with CKD and an EF of 25%, NSAIDs would pose
significant risk of renal dysfunction, fluid retention, and precipitation of HF. NSAIDs should be used with great caution in all older adults because of the risk of PUD,
platelet dysfunction, GI bleeding, and exacerbation of HTN.

Case #2
• An 86-year-old woman is admitted to an acute rehabilitation facility 4 days after surgery for a broken hip. Her daughter describes the hospital stay:
 The patient seemed fine on days 1 and 2 after Sx: she worked with the PT, and her pain seemed well controlled.
 For the last 2 days she has refused meals and seems more confused.
 She vomited twice that morning before transfer to the facility.
• Medications on discharge from the hospital:
 Diltiazem 360 mg/d, alendronate 70 mg/wk, calcium carbonate 600 mg with vitamin D 400 IU BID, oxycodone 5 mg q 6 hours (held for sedation)
 Ondansetron, 4 mg by mouth three times daily as needed, was added on the day of discharge.
• At the rehabilitation facility, she frequently groans and calls out for help, and she seems tense.
• History: moderate dementia, hypertension
• Which one of the following is the most likely explanation for these findings?
A. Abdominal pain due to constipation
B. Delirium due to initiation of ondansetron
C. Delirium due to anesthesia and change of environment
32
D. Nausea due to opioid therapy
E. Worsening postsurgical hip pain

ANSWER: A
This patient with moderate dementia is displaying several common pain behaviors: she is groaning and calling out for help, seems tense, refuses food, & demonstrates
increasing confusion. Presumably oxycodone was started 4 days ago after Sx, and it appears that her pain was initially well controlled. There is no record that a bowel
regimen was started, which is imperative to consider when initiating opioids. Constipation is the most common adverse effect of opioid treatment, and tolerance does not
develop.
The patient is actually taking several other Rx that can cause constipation: diltiazem, calcium supplementation, and ondansetron. The development of nausea (as evidenced
by the recent addition of ondansetron) and her refusal of meals further support the likelihood that she is constipated.
Delirium alone would not explain the patient’s nausea and her tense body posture. The delirium would most likely be caused by the constipation and unTx abd pain a/w
it. Ondansetron, which is not a particularly deliriogenic antiemetic, was started that day, and therefore does not account for the ssx that began earlier. Anesthesia can
contribute to delirium, although it would be less common for the onset to be delayed by 2 days. The change of environment from home to hospital, and now from hospital
to rehab facility, could contribute to confusion, although the contribution from abd pain related to constipation is likely much greater.
Nausea is a common SE of opioid therapy and would account for her refusing meals. However, it does not explain her calls for help, her tense posture, and her apparent
increase in confusion. In this case, the nausea is likely related to her constipation.
Although possible, it would be unexpected for the patient’s hip pain to worsen as more time passes after surgery. If her pain had been well controlled on the current
regimen of oxycodone, an increase in pain level could signify a complication. It would be prudent to exclude DVT, surgical site infection, or other issue, and to conduct
a careful physical examination and laboratory evaluation to exclude other causes of change in status, such as urinary tract infection, dehydration, or pneumonia.

Case #3
• An 82-year-old man is admitted to hospice after persistent functional decline.
• History: diabetes mellitus, heart failure, chronic renal insufficiency, osteoarthritis
• Oxycodone controls his pain, but he suffers from opioid-induced constipation.
• Which one of the following is the most appropriate next step in the management of his constipation?
A. Start senna.
B. Start docusate and senna.
C. Start methylnaltrexone.
D. Increase fiber in his diet.

ANSWER: A
Opioid-induced constipation (OIC) is common and should be anticipated, especially in hospice and palliative care settings. OIC contributes to symptom burden and
decreased quality of life. Unlike constipation associated with other conditions or causes, OIC is unlikely to respond to increased dietary fiber, fluid intake, or physical
activity levels. Pharmacologic treatment can include surfactants (docusate), stimulants (senna, bisacodyl), osmotic laxatives (magnesium sulfate, lactulose, polyethylene
glycol), opioid-receptor antagonists (naloxone, methylnaltrexone), and enemas (soapsuds, mineral oil, sodium phosphate). Stimulant laxatives, including senna and
bisacodyl, act on the colon to increase intestinal motility by local irritation of the mucosa or on nerves and smooth muscle. Previously, there was concern that chronic use
of stimulant laxatives would harm the colon, damaging the enteric nervous system and leading to physical dependence on laxatives. However, there is no evidence that
this occurs when laxatives are given in appropriate doses. Common SE of stimulants and other laxatives include abd pain and cramping and fluid and electrolyte imbalance.
A double-blind, placebo-controlled randomized trial showed that docusate in combination with senna did not offer additional benefit to senna alone in alleviating OIC.
Methylynaltrexone is often used for intractable OIC that is unresponsive to regular laxatives and is not the most appropriate next step for this patient.

Palliative Care

 Palliative care = interdisciplinary care that aims to relieve suffering, improve quality of life, optimize fxn, & assist w/ decision making for pts w/ advanced
illness & their families
o Offered simultaneously w/ all other appropriate medical Tx
o Drs need to talk w/ pts early about pref & to provide better ssx control & palliative measures @ end of life
o Available prognostic tools
 Palliative Performance Scale  embraces self-care fxn, mobility, activity level, PO intake, LOC
 Eprognosis
 “Hospice in a Minute”
 Hospice = specialized palliative care limited to pts who meet 2 criteria
o Life expectancy is < 6 mo
o They have elected to focus on comfort measures & forgo curative Tx
 Medicare benefits
o Pt must be recertified hospice-eligible every few mo
o Face-to-face visit for all pts entering 3rd certification period (at 6 mo’s of hospice) & q 60 d certification period thereafter to reeval the pt’s prognosis
 National consensus guidelines
Domain Guideline Comments
1 Structure & Process Describes the interprofessional team engagement w/ pts & families w/ emphasis on indv pref
2 Physical Aspects Describes the assessment & management of physical ssx w/ validated tools & a multidimensional approach to
management
3 Psychological & Psychiatric Aspects Describes the collaborative assessment of psychological conerns & psychiatric dx to enhance care; outlines req
for a bereavement program for pts, families, & staff
4 Social Aspects ID’s the essential elements of palliative care social assessments & emphasizes ID’ing & supporting family
strengths
5 Spiritual, Religious, & Existential Describes the assessment of these concerns through the dz course & the importance of eval by the
Aspects interprofessional team
6 Cultural Aspects Defines cultural competence for the interprofessional team
7 Care of the Pt @ end of life Describes communication & documentation of ssx of the dying process w/ the fam
8 Ethical & Legal Aspects Describes advance care planning, ethics of palliative care, & legal issues
 Steps in communicating serious news
o Prepare for meeting
 Plan what will be discussed
 Ensure that all medical facts and confirmations are available

33
 Choose an appropriate, comfortable setting
 Deliver the news in person, privately
 Allow time for discussion
 Minimize interruptions
o Est. pt’s understanding
 Ask questions such as:
 “What do you understand about your illness?”
 “When you first had symptom X, what did you think it might be?”
 “What have other doctors told you about your condition or procedures that you have had?”
 “What are you hoping for or worried about?”
o Determine how much the pt wants to know
 Important to:
 Discuss the patient’s preferences and goals of care to help guide treatment recommendations
 Understand what is important to the patient
 Determine if and how they want information give to them
 Determine how they want to participate in the decision making about their care
 Patients with serious illness and their families also may have different communication needs from each other, and it is important to
determine this early
 Asking, “Are you the type of person who is comforted by details and test data?” or “Would it be helpful to discuss prognosis now?”
both allows the patient to remain in control as well as determines the amount of information to which they are exposed
o Tell pt
 Deliver information in a sensitive, straightforward manner
 Avoid technical language or euphemisms
 Check for understanding
 Phrasing the includes a warning helps prepare patients for bad news.
 “The report is back, and it is not as we had hoped.”
 Based on these discussions, clinicians may offer to make a recommendation for care that is seen as consistent with the goals and values
a patient has expressed
o Respond to feelings & emotions
o Organize a plan & f/u visit
 Address patient’s concerns in an immediate plan
 Set an appointment for a follow-up visit
 Discuss additional tests, referrals, sources of support
 Provide information on how you can be reached with additional questions
 After patients’ preferences for end-of-life care are elicited (perhaps through surrogates), they must be faithfully communicated. Often,
these prefs are documented in AD or written orders about CPR
 Advanced Directives
o Physician Orders for Life Sustaining Tx (POLST)  medical orders reflecting pref for CPR, medical interventions, Abx, artificial hydration,
transfer across care settings
 Decreases rate of unwanted hospitalizations & better documentation of preferences
 Palliative-Focused Ssx assessment
o Determine the patients goals of care
o Assess the patient’s functional status
o Assess the patient’s physical ability to tolerate different treatment modalities and routes of Rx administration
o Determine the patient’s overall prognosis
Constipation  Common for terminally ill pts  PPX laxatives: stool softener, bowel stimulant (docusate sodium,
 Cz’ed by opioids, immobility, poor fluid intake senna, Bisacodyl)
 Can add osmotic laxative (sorbitol, lactulose, polyethylene glycol)
 If no BM in 4 days  consider enema
 If impaction  disimpact manually or w/ enemas before starting
laxative Tx
 If nothing works, consider methylnaltrexone bromide or
lubiprostone (both CI in bowel obstruction)
N/V  40 – 70% of pts w/ advanced CA • Keys to successful management involves:
 May be cz’ed by dz or its Tx • Identifying the likely cause of the nausea
• Selecting a Rx that works on the cause
• Giving around-the-clock Rx if constant
Diarrhea  7-10% of pts w/ CA admitted to hospice • Radiotherapy to abd/pelvis can cz diarrhea & responds to
 Passage of urine > 3 unformed BM w/in 24hrs cholestyramine
 Consider fecal impaction presenting as watery diarrhea in • Fat malabsorption responds to pancreatic enzymes (ex: pancreatin)
immobile older pts on opioids
 Review Rx for excessive laxative Tx
GI Obstruction  Cz’es: direct intraluminal obstruction by tumor, malignant • Surgical: limited evidence for benefit in terms of quality @ end of
infiltration, external compression, dysmotility, fecal life
impaction, SE of radiation, volvulus, adhesions • Endoscopic: stents = best for single pt of obstruction
 50% of pts w/ ovarian & GI CA • Medical: mainstay of Tx (combo opioids, antispasmodics,
 Pts w/ malignant BO have median surv of 3 mo anetiemetics, antisecretory agents, corticosteroids)
 High ssx burden: hypersalivation, N/V, colicky abd pain, • NG tube: temporary when awaiting further Tx
anorexia, wt loss
Anorexia &  Loss of appetite is almost universal among terminally ill • Encourage patients to eat whatever is most appealing, without
Cachexia patients dietary restrictions
 Anorexia in actively dying patients who do not wish to eat • In pts w/ advanced dementia, offer PO-assisted feeding
should not be treated • Provide pt & fam education regarding the normalcy of anorexia as
 Ssx of dry mouth should be treated a part of end-of-life & dying process

34
 Appetite stimulants (eg, megastrol acetate & corticosteroids • Enteral feeding is not recommended @ end of life as it does not
(off label)) may benefit pts in early stages; doesn’t prolong improve quality of life or survival and is a/w inc freq of aspiration
survival or improve fxn or Tx tolerance of CA Tx, & are a/w & other complications
SE
Delirium  Common and distressing for both terminally ill patients and • Nonpharm approaches: minimizing noise, using an orientation
their families board, mounting a visible clock, using simple communication &
 Identify potentially reversible causes (infection, impaction, minimizing disruptions
uncontrolled pain, urinary retention, medications, • Pharm approaches: Rx are indicated either to ensure the pt’s safety
dehydration, and hypoxia) or if the delirium is cz’ing distress
 Use low doses of nonsedating antipsychotic • Use low doses of nonsedating antipsychotic Rx
 Actively dying, nonambulatory patients may benefit from • Actively dying, nonambulatory pts may benefit from sedating
sedating antipsychotic antipsychotic Rx
 Avoid benzodiazepines • Avoid BZD due to their a/w paradoxical agitation and worsening of
delirium in older adults
Dyspnea  Patient self-report is the only reliable measure • Treat underlying cz, but don’t delay ssx management
 Respiratory rates, pulmonary congestion, and lab tests often • Opioids = most effective Tx agent. Act by dec the perception of
do not correlate dyspnea & don’t affect resp drive. Both PO & parenteral
 Goal of Tx is the subjective improvement of breathlessness, formulations are effective
rather than lowering the respiratory rate to normal • O2 if saturation < 90% (caution w/ pts who retain CO2)
 Often patients will report improvement in breathlessness yet • Cool air across the face by fan or an open window to stimulate 5 th
still breathe rapidly cranial nn & reduce dyspnea
• BZDs control anxiety but not dyspnea
Cough  Production of excess fluids • Treat underlying cause
 Inhalation of foreign material • Add opioids if underlying disease not resolvable
 Stimulation of irritant receptors in the airway • Dextromethorphan: few sedative effects
• Codeine, hydrocodone elixirs
• Methadone syrup for longer duration of action
• Nebulized anesthetic for irritated pharynx of local infxn or
malignancy
Loud Respiration  The inability to clear secretions from the oropharynx  loud • Family education prior to its occurrence
or “rattling” respirations at the end of life • Anticholinergics to reduce secretions
 Caused by secretions oscillating up and down during • Hyoscine (scopolamine patch) [off label], +/-sedating
inspiration and expiration • Hyoscyamine
• Glycopyrrolate
• Atropine eye drops (off label)
• Careful monitoring is necessary when using antichol Rx as they can
contribute to dry mouth, constipation, delirium, & mucous plugging
Depression  Under-recognized & underTx in terminally ill • Be alert for mood , loss of interest, suicidal ideation
 Diminishes quality of life, amplify pain & other ssx, and • Tx aggressively: Antidepressants, psychiatric consult, CBT are
impair a patient’s ability to deal with the emotions involved appropriate
in saying goodbye • Consider ECT for those who are severely depressed and when a
 Major RF for suicide & for requests to clinicians to hasten rapid response is needed
death • Involvement of clergy or pastoral care may be helpful to help
 Vegetative ssx (insomnia, anorexia, weight change) may not patients & families to work toward closure & resolution
be reliable b/c of underlying dz

 Death certificate
o Death certificates are needed for personal, legal, and public health purposes
o Accurate documentation of cz of death & communication w/ the family may help them w/ closure & peace of mind
o Physicians are responsible for completing the medical portion of the death certificate, the “Cause of Death” section and usually the pronouncement
of death
 Cause of Death Section has 2 parts:
 1) Immediate cause of death
 2) Other conditions that contributed to the death
o Pts to remember when completing a death certificate
 Do not delay completion of the certificate. The burial or disposition of the remains cannot proceed until a correctly completed death
certificate is accepted by the state.
 Print clearly or type using black ink.
 Do not use abbreviations -- spell out the month.
 Use a 24-hour clock.
 Complete all items, do not leave blanks. If necessary, use “unknown.”
 Do not complete the medical information if another available Dr has more knowledge of the circumstances.
 Do not alter the document or erase any part of it.
 Physician burnout
o Ssx: irritability, insomnia, forgetfulness, resentment, mental/physical fatigue, social withdrawal, increased EtOH use, apathy, chronic sadness
o Affects both personal & professional consequences
 Choosing wisely
o Do not use topical lorazepam, diphenhydramine, or haloperidol gel for nausea.
o Don’t recommend percutaneous feeding tubes in pts with advanced dementia; instead offer oral assisted feeding.
o Avoid using prescription appetite stimulants or high-calorie supplements for treatment of anorexia or cachexia in older adults; instead, optimize
social supports, provide feeding assistance and clarify patient goals and expectations.

Cases / Questions
Case #1
 A 75-year-old woman comes to the emergency department because of new severe lower back pain.

35
o She reports no falls or trauma.
o She has had no new weakness or bowel or bladder incontinence.
 History: CAD, diabetes mellitus, metastatic breast cancer
o The breast cancer was treated with chemotherapy and radiation 6 years ago.
o Three months ago she had right shoulder pain, and a scapular metastatic lesion was diagnosed.
 Radiography of the lumbar sacral spine demonstrates met dz involving L1–L4, with severe pathologic compression of L3.
 The patient is admitted for pain control.
o The pain (aggravated by movement) is controlled at rest with around-the-clock morphine.
o B/c of its severity, she is unable to walk, spends all of her time in bed, and is dependent in all ADLs except feeding.
o The patient acknowledges that her CA is incurable & states that she does not want further hosp or interventions.
 The medical team recommends hospice. The patient’s husband asks for information on hospice services.
 Which one of the following is covered under the Medicare hospice benefit?
A. Room and board in a long-term care facility
B. Nursing care in a postacute facility
C. Hospital bed and bedside commode for home
D. Private-duty caregiver at home
E. Medication for diabetes mellitus and coronary artery disease

ANSWER: C
Patients with Medicare Part A are eligible for the Medicare hospice benefit if they have a chronic or terminal illness with a life expectancy ≤6 months as determined by
2 physicians. Once a patient decides to pursue hospice care, he or she signs off from Medicare Part A and signs up for the hospice benefit. The benefit covers a variety of
services, including provision of durable medical equipment, such as a hospital bed and bedside commode, if needed because of the terminal diagnosis.
The Medicare hospice benefit does not cover room and board in a long-term care facility. Patients who utilize the benefit may live in a house, an adult congregate-living
facility (eg, assisted-living or board-and-care facility), or a long-term care facility. Room and board is typically paid for from personal funds or Medicaid.
The Medicare hospice benefit does not cover skilled-nursing care. There are 4 levels of hospice care: routine home care, continuous home care, inpatient respite care, and
general inpatient care. Most patients receive routine home care. Patients with pain or other symptoms that cannot be managed at home may require general inpatient care,
which may be provided in a freestanding hospice unit or in a dedicated bed at a hospital or long-term care facility, until the patient’s symptoms are under control.
The Medicare hospice benefit includes services from a variety of health professionals, but it does not cover private-duty caregivers. The benefit does cover home health-
care aides and homemaker services. Other covered services include nursing and physician visits, social services, counseling, and physical, occupational, and speech
therapy, if indicated. The Medicare hospice benefit covers medications related to comfort and palliation, not for cure of the terminal illness or other medical conditions.

Case #2
• An 88-year-old woman is admitted to the hospital because she has worsening abdominal pain.
• History: osteoarthritis, hypertension, hypothyroidism, metastatic pancreatic cancer
• Pain management
• At home, she was taking oxycodone 5 mg every 6 hours, with little pain relief.
 Because of pain and fatigue, she has spent most of the last month in bed.
• In the hospital, the pain was controlled with IV hydromorphone, with subsequent transition to long-acting oxycodone 20 mg twice daily and short-
acting oxycodone for breakthrough pain.
• Before discharge, the medical team meets with the patient and her family. The patient’s husband asks the team to address her prognosis.
• Which one of the following is most predictive of a poor prognosis?
A. Advanced age
B. Uncontrolled pain
C. Low performance status
D. Multiple comorbidities
E. Opioid dependence

ANSWER: C
Predicting life expectancy is difficult, and physicians tend to overestimate survival in patients with advanced cancer. Hospice referrals require a Dr-predicted prognosis
of ≤6 months. A # of pt characteristics have been investigated & several tools developed to help physicians predict prognosis. Low performance status is the factor that
best predicts poor prognosis in a patient with advanced CA.
Performance status is a global measure of a patient’s functional capacity, or ability to maintain independence in daily life. A number of different tools are available to
assess performance status, including the Karnofsky Performance Score and the Palliative Performance Scale, Version 2 (PPSv2). The PPSv2 assesses 5 domains:
ambulation, activity level and evidence of disease, self-care, nutritional intake, and level of consciousness. Advanced age, uncontrolled pain, polymorbidity, and opioid
dependence have been shown in some studies to influence prognosis, but the findings are not consistent.

Case #3
• An 84-year-old man is admitted to the hospital for pain control related to a pathologic pelvic fracture after a fall at home.
• History: Alzheimer disease, metastatic prostate cancer
• Hospital course
• Oxycodone is started, 5 mg every 6 hours around the clock, and his symptoms improve.
• On hospital day 2, there is swelling in his left lower extremity.
• Ultrasonography reveals acute deep-vein thrombosis, and anticoagulation therapy is started.
• Over the next few days, his oral intake decreases, and he requires IV hydration.
• A bedside meeting is held with the patient’s wife, who has durable power of attorney.
• She says that his appetite at home has been poor and that he has lost 6.8 kg (15 lb) in the last 3 months.
• She asks what can be done to prevent further weight loss.
• Which one of the following is the most appropriate recommendation for this patient?
A. Oral dronabinol 2.5 mg twice daily
B. Oral megestrol 400 mg/d
C. Placement of gastrostomy tube for enteral feedings
D. Total parenteral nutrition
E. Small meals of the patient’s favorite foods

ANSWER: E

36
Loss of appetite is almost universal at the end of life, and it is usually more distressing to the fam than to the pt. This pt has advanced CA with a poor prognosis. An inc
in appetite & wt is unlikely to improve his quality of life or clinical outcome. Pt & fam education on the effects of terminal CA on appetite & wt is essential. Next, address
any uncontrolled ssx that may contribute to anorexia, such as constipation & nausea. Finally, the patient’s diet should be liberalized to include calorically dense food,
particularly his favorites.
Other interventions for anorexia have limited benefit and potential risks in patients with advanced CA. Total parenteral nutrition and enteral nutrition via gastrostomy
tube have not been shown to improve life expectancy or quality of life for pts with end-stage CA and are not recommended for this pt. Although appetite stimulants such
as dronabinol and megestrol acetate have been shown to increase appetite and even weight in some studies, their use does not improve survival or clinical outcome.
Dronabinol can affect mental status and should be used with caution in this patient with dementia. Megestrol acetate is a/w an increased risk of thromboembolism and
would be inappropriate for this patient, given his recent h/o DVT.

Pressure Ulcers & Wound Care

 Pressure ulcers = localized injury to skin &/or underlying tissue (usu over boney prominence that results from pressure or pressure + shear
o Associated with
 Decreased quality of life
 Longer hospital stays
 Increased chance of 30-day readmission
 Increased chance of admission to a long-term care facility
 Increased risk of death
o Estimated cost of $3,500 to >$60,000 per patient
o Affect 1.3-3 million adults in the U.S.
o Are a designated quality measure and major risk management issue
 Unavoidable pressure ulcers
o One that develops despite the provider having
 1) evaluated the individual’s clinical condition and pressure ulcer risk factors
 2) defined and implemented interventions consistent with individual needs, goals, and recognized standards of practice, and
 3) monitored and evaluated the impact of the interventions, revising the approaches as appropriate
o Patients at high risk are those with immobility and multiple chronic conditions
o These pressure ulcers may be a marker for disease severity or impending death
 Normal wound healing: hemostasis  inflammation  proliferation  remodeling

Staging of pressure ulcers


Stage I: nonblanchable erythema • Intact skin with nonblanchable redness of a localized area, usually over a bony prominence.
• Darkly pigmented skin may not have visible blanching; its color may differ from the surrounding area.
• The area may be painful, firm, soft, or warmer or cooler than adjacent tissue.
• May be difficult to detect in those with dark skin tones.
• May indicate “at risk” patients.
Stage II: partial thickness • Partial-thickness loss of dermis presenting as a shallow open ulcer with a red pink wound bed, without slough.
• May also present as an intact or open/ruptured blister filled with serum or serosanguinous fluid.
• Presents as a shiny or dry shallow ulcer without slough or bruising
Stage III: full thickness tissue loss • Full-thickness tissue loss
• Subcutaneous fat can be visible but bone, tendon, or muscle are not exposed
• Slough may be present but does not obscure the depth of tissue loss
• May include undermining and tunneling
• Depth varies by anatomical location
Stage IV: full thickness tissue loss • Full-thickness tissue loss with exposed bone, tendon, or muscle
• Slough or eschar may be present
• Often includes undermining and tunneling
• Depth varies by anatomic location
Stage V: unstageable; full thickness skin • Full-thickness tissue loss in which actual depth of the ulcer is completely obscured by slough (yellow, tan, gray,
or tissue loss, depth unknown green, or brown) and/or eschar (tan, brown, or black) in the wound bed
• Until enough slough and/or eschar is removed to expose the base of the wound, the true depth (and therefore stage)
cannot be determined
• Stable (dry, adherent, intact without erythema or fluctuance) eschar on the heels serves as “the body's natural
(biological) cover” and should not be removed
Suspected deep tissue injury, depth • Purple or maroon localized area of discolored intact skin or blood-filled blister due to damage of underlying soft
unknown tissue from pressure and/or shear
• May be preceded by tissue that is painful, firm, mushy, boggy, warmer, or cooler than adjacent tissue
• May be difficult to detect in individuals with dark skin tones
• Evolution may include a thin blister over a dark wound bed
• The wound may further evolve and become covered by thin eschar
• Evolution may be rapid and expose additional layers of tissue, even with optimal treatment

 Risk assessment
o Braden Scale = most widely used
 Combines sensory perception, moisture, activity, mobility, friction, shear
 Validated in home, SNF, hospital settings (not ICUs)
 RF for pressure ulceration
Intrinsic RF Extrinsic RF
Dermatitis Friction
Edema Immobility
Hypoperfusion Medical devices
Immobility Moisture
Long-term corticosteroid use Pressure
Circulatory impairment Shear forces

37
Nutritional compromise

 Moisture-assoc. skin damage (MASD)


o Occurs from perspiration, urine, diarrhea, fistulas, or wound exudate
o Increases susceptibility to pressure ulcers
o Is a common occurrence in patients with constant loose stools from tube feeding or Clostridium difficile colitis
o Strategies to avoid or treat MASD: moisture barrier creams, absorbable undergarments, continence care, and fecal/urinary diversion devices
 PPX Strategies
o Strategies to minimize pressure, friction, and shear
 Pillows and foam wedges to keep pressure off bony prominences
 Lifting devices and draw sheets to minimize friction
o Industry standard for turning and repositioning is every 2 hours, however, individualize for each patient
o Pts who sit for long periods of time in chairs should be assessed for proper posture/alignment & provided pressure relief schedules & cushioning
o Prevent heel wounds with local skin care, cushioning, “floating the heels”
 Pressure redistribution surfaces
o Group 1 support surfaces are nonpowered devices made of gel, foam, or water
o Group 2 support surfaces include alternating-pressure air mattresses and pressure-reducing air mattresses of the low air loss type.
o Group 3 support surfaces are complete bed systems using air-fluidized technology using pressurized silicone-coated beads that promote a flotation
environment
 Principles of pressure ulcer Tx
o Should be a holistic, patient-centered approach
o Assess overall health status of the patient
o Address psychosocial needs
o Treat underlying comorbidities
o Assess and correct causes of tissue damage
o Assess and monitor the wound
o Understand patient’s functional and cognitive status, home environment, family support, and other factors such as presence of depression
o Elements of pressure ulcer treatment
 Offloading and pressure redistribution strategies
 Removing debris and necrotic tissue
 Addressing moisture balance
o Clinicians must also:
 Address pain
 Acknowledge advance directives
 Recognize wounds appropriate for a palliative approach and alter the plan accordingly
 Wound Tx Modalities
Type Content Rationale Best Use

Gauze Cotton, polyester, or other fabrics Versatile, can be absorptive, Secondary dressing, wet to moist, or wet to dry, or as a
protective, 1º or 2º dressing protective to the wound and surrounding skin

Hydrocolloid Adhesive pad w/ moisture-activated, Moisture retention Superficial, clean pressure ulcers w/o necrosis or infxn
gel-forming material; gelatin & pectin

Semi-permeable Transparent polymer + acrylic adhesive Moisture retention Superficial, clean pressure ulcers w/o necrosis or infxn
films

Hydrogel Water in a delivery vehicle such as Promote moist wound healing & Dry wounds; wounds with some necrosis
glycerin or cross-linked polymer sheets autolytic debridement

Foam Polyurethane WWO adhesive borders Absorb exudate, cushioning, 2º Control exudate, protect the wound
dressing

Alginate Seaweed derivative; can be in different Absorptive dressing Control of exudate


forms, including sheet or rope, and
combined with other materials such as
silver or charcoal

Collagen Animal-derived collagen formulated Deactivates matrix metallo- Partial- or full-thickness wounds with minimal necrosis
into gel, powder, paste, or sheet proteases  inhibit wound healing

Silver-containing Silver can be impregnated into multiple Silver has broad-spectrum Wounds requiring control of bacterial balance
dressings types of dressings antimicrobial activity

Enzymatic Enzyme in a petrolatum vehicle Selected degradation of denatured Wounds with necrosis and slough
debriding agent collagen

Cadexomer Iodophor in a polysaccharide polymer Absorbent, antimicrobial Wounds with slough, infected wounds
iodine dressing

Silicone Inert silicone polymer; sometimes has Contact layer that can be removed When a nonadherent dressing is required, protects the
dressings pores that allow passage of exudate without causing trauma to wound or wound and surrounding skin
surrounding skin

Activated Combined with silver or other vehicles Control odor Palliative wounds with odor
charcoal

38
Honey Medicinal grade honey can be used as a Antimicrobial properties, anti- Autolytic debriding agent on noninfected wounds
gel or impregnated into other dressing inflammatory
types

Topical Includes hydrogen peroxide, Dakin’s Reduce bacterial burden of wounds Can be cytotoxic to healing wounds; for limited use in
antiseptics solution (hypochlorite), povidone-iodine heavily contaminated or nonhealable wounds

Petrolatum- Woven mesh; medical petrolatum and Bacteriostatic, nonadherent, retains Use with larger wounds with minimal necrosis and slough
impregnated 3% bismuth tribromophenate moisture
gauze

 Wound bed preparation


o Moisture balance
o Bacterial balance
 Facilitated by cleansing, topical antibiotics, disinfectants, and debridement
 Methods of debridement: autolytic, enzymatic, mechanical, biologic, surgical
o Management of eschar
 Alternative Tx: electrical stimulation, therapeutic US, light therapy, negative pressure wound therapy, hyperbaric O2
 Nutrition
o 30 – 35 kcal/kg/day for calories
o 1 – 1.5 g/kg/day for protein
 Infxn
o All chronic wounds are contaminated or colonized with bacteria but may not be infected
o Infectious complications  cellulitis, abscess, osteomyelitis, pyarthrosis, necrotizing fasciitis, systemic infectious
o Signs and symptoms of infection
 Fever, increased drainage, pain, warmth, edema, erythema, slough, odor, cessation of healing, worsening of the wound
o Swab cultures are best reserved for wounds with purulent drainage in the setting of high suspicion for infection
o Treatment must involve
 Managing underlying conditions & protection from urine and feces
o Wound infections can be treated locally, systemically, or both depending on the clinical situation
 Palliative care for chronic wounds
o SPECIAL
 Stabilize wound
 Prevent new wounds
 Eliminate odor
 Control pain
 Infxn PPX
 Absorbent wound dressings
 Lessen/reduce dressing changes

Case/Questions
Case #1
• A 65-year-old man being admitted for rehabilitation after surgical repair of his left hip, fractured in a fall 1 week earlier
• History: type 2 diabetes mellitus, coronary heart disease, hypertension, hyperlipidemia
• 50-pack-year smoking history
• Physical examination:
• Diminished sensation in both feet (by monofilament testing)
• Diminished pulses in bilateral dorsalis pedis and posterior tibialis
• Crusted skin, thick toenails
• Nonfluctuating, dry, black eschar on lateral left heel. Surrounding skin is intact, with no evidence of cellulitis.
Which one of the following is the most appropriate initial management of the wound?
A. Sharply debride the eschar.
B. Leave the eschar intact and elevate both heels off the bed surface.
C. Cover the wound with a hydrocolloid sheet; change every 3–5 days and as needed.
D. Apply a silver alginate dressing plus a secondary dressing over the wound; change weekly.

ANSWER: B
When external surface pressures, friction, and shearing forces exceed the nl arterial capillary pressure of 32 mmHg, perfusion is impaired and ulceration can occur. Placing
a pillow vertically under the patient’s legs will support or “float” the heels off the bed surface, thereby eliminating pressure, friction, and shearing.
Stable, dry eschar can act as a physiologic barrier to infection. The European Pressure Ulcer Advisory Panel and the National Pressure Ulcer Advisory Panel do not
recommend its debridement, especially if there is actual or suspected vascular disease. Instead, the wound should be inspected daily for signs of infection (≥1 of the
following: erythema, tenderness, edema, purulence, fluctuance, crepitus, and odor). If infection is present, the ulcer should be debrided immediately.
Hydrocolloid sheets provide a protective barrier and facilitate autolytic debridement. However, a stable dry eschar should not be debrided. When hydrocolloid sheets are
indicated, they should be used with caution on patients with diabetes, because removal of the sheets may cause tissue trauma.
Silver-impregnated dressings are used to reduce wound bioburden if infection is evident or suspected. This patient’s wound displays no clinical signs of infection. A
calcium alginate dressing, used for exudate management, is contraindicated because the wound has stable, dry eschar and no exudate.
B/c this pt has a sign of and several RF for PAD (diminished pulses, DM, CAD, HTN, hyperlipidemia, and tobacco use), he is at greater risk of poor healing and infxn of
open wounds. His arterial circulation should be evaluated by an ABI; if indicated, he should be referred to a vascular surgeon as soon as possible.

Case #2
• An 85-year-old man with areas of redness and skin breakdown on his buttocks
• History: dementia, Parkinson disease
• Spends much time sitting in a recliner, wears a diaper because of urinary and occasionally fecal incontinence. Appetite is good, weight is stable.
• Physical examination
• Several areas on the buttocks have partial-thickness erosions with sanguineous drainage.

39
• Tender to touch
• No induration, satellite lesion, or surrounding erythema
• Diaper is saturated with urine and there are traces of stool adherent to the buttocks and perirectal area.
Which one of the following is the most appropriate skin-care recommendation?
A. Apply a petrolatum-based barrier ointment.
B. Apply a barrier film.
C. Discontinue use of diapers and put the patient on bed rest until the areas are healed.
D. Apply a mild antifungal cream to the affected areas.

ANSWER: B
This patient has incontinence-associated dermatitis, which results from the effects of urine and stool on skin. A flexible barrier film that coats the affected area and allows
vapor transmission for up to 72 hours is an appropriate, cost-effective method of protecting the skin from the effects on incontinence. Petrolatum-based ointments, such
as zinc oxide plus petrolatum, form an occlusive layer on the skin surface to support skin barrier repair, promote skin hydration, and establish a physical barrier. However,
the ointment needs to be reapplied after each incontinence episode, requiring an effective toileting protocol. A petrolatum-based ointment provides a barrier to moisture
but does not adhere well to desquamated tissue.
Extended exposure to urine can macerate the stratum corneum and increase its susceptibility to friction, leading to further skin breakdown, infection, and activation of an
inflammatory response, all of which increase the likelihood that a pressure ulcer will develop. Incontinence-associated dermatitis is exacerbated by the use of containment
devices, such as diapers, that reduce airflow and trap heat and moisture. Removing the diaper would be optimal and allow appropriate airflow without entrapping moisture
and heat. However, limiting an older adult to bed rest, even briefly, will lead to unnecessary functional decline.
Antifungal ointments are appropriate for a fungal infection related to incontinence-associated dermatitis. An antifungal cream is not indicated at this time, because there
is no evidence of a fungal infection. Use of a corticosteroid ointment is contraindicated over an area that is contained within a diaper.
On average, 50% of nursing-home residents are incontinent and at risk of incontinence-associated dermatitis. Unlike a pressure ulcer, dermatitis is not limited to skin over
bony prominences, its areas of damage are usually profuse, and there is usually partial-thickness skin loss.
Skin products should mimic skin pH as closely as possible. Many soaps and cleansers are alkaline, with a pH around 9, and alter the skin’s acid mantle, potentially
increasing susceptibility to breakdown and infection. No-rinse foam cleansers are good alternatives to soap.

Prevention

 Categorize clinical condition:


o ≥10 years of remaining life expectancy
o 5-10 years of remaining life expectancy
o Moderate Dementia: 2‒10 years of remaining life expectancy
o End of Life: <2 years of remaining life expectancy
 Breast CA
o Pros vs. cons of mammo q2yrs (≥ 10 yr life exp.)
o Perform CBE periodically, but no evidence for benefit > 75 y/o
 CRC for those 50 – 75 y/o
o Stool based tests (q 1 – 3 yrs) or C-scope q 10 yrs
o No recommendation for screening ≥ 85 y/o or those w/ life expectancy < 10 yrs
 Cervical CA
o Paps q 3 yrs or 5 yrs if combo w/ HPV testing (Medicare covers pap + pelvic exam q 2 yrs)
o Stop screening @ 65 y/o if adequate prior screening & don’t screen ppl with TH + removal of cervix + no h/o CIN II or III or cervical CA
 Adequate screening = 3 consecutive (-) cytology results or 2 consecutive (-) HPV results w/in 10 yrs before cessation of screening w/
most recent test w/in 5 yrs
 Prostate CA
o USPSTF does not recommend PSA
o ACS and AUA recommend discussion w/ pts ≥ 50 yrs + > 10 – 15 yr life expectancy
 Lung CA
o Annual screening w/ low-dose CT in adults 55 – 74 y/o or 55 – 80 y/o + 30 PY smoking hx + currently smoke + have quit w/in past 15 yrs
o Stop screening when pt has not smoked for 15 yrs or life expectancy has declined

USPSTF recommends against screening an asymptomatic adult for: With these tests:

Asymptomatic bacteriuria Urinalysis


Coronary artery disease in adults with few or no risk factors ECG, exercise treadmill test, or electron-beam CT

Carotid artery stenosis Duplex ultrasonography


Cervical cancer in women ≥65 years old who have had adequate prior Pap smear
screening or among women who have had a hysterectomy with removal
of the cervix for benign disease
Colon cancer in adults ≥86 years old (Screening may be modestly Stool based tests/ sigmoidoscopy/colonoscopy
beneficial in adults 76–85 years old with long remaining life expectancy
and no or few comorbidities.)
COPD Spirometry
Ovarian cancer Transvaginal ultrasonography, CA-125, or pelvic examination
Pancreatic cancer Ultrasonography, abdominal palpation, or serologic markers
Prostate cancer PSA and/or digital rectal examination
Bladder cancer Urinalysis, bladder tumor antigen measurement, NMP22 urinary enzyme immunoassay, or
urine cytology
Breast cancer in women ≥75 years old Mammography
Chronic kidney disease Creatinine-derived estimates of glomerular filtration rate, urine testing for albumin
Oral cancer Systematic clinical examination of the oral cavity
Peripheral arterial disease Ankle-brachial index
40
Skin cancer Whole-body skin examination

Coronary artery disease in adults at intermediate or high risk ECG or exercise treadmill test
Thyroid cancer Palpation
Thyroid disease Thyroid stimulating hormone
Type 2 diabetes in normal weight adults Fasting plasma glucose, 2-hour postload plasma, hemoglobin A1c

Other Screening Tests:


≥10 years remaining 5 to <10 years remaining life Near end of Cost-
Procedure life expectancy expectancy Moderate dementia life effective?
DEXA At least once after age 65, or age 60 if Consider if not done previously NR NR Yes
screening for high risk
osteoporosis
Blood glucose Screen obese or overweight adults up to NR NR NR Uncertain
age 70
Cholesterol Screen those with additional risk factorsf NR NR NR Uncertain
screening
US for AAA Once for men 65−75 years old who ever Consider NR NR Yes
smoked
Thyrotropin Every 2–5 years Every 2–5 years Every 3 years Consider Uncertain
HIV Consider for those at high risk Consider for those at high risk Consider for those at high risk NR Yes
Hepatitis C One time for those born between 1945– One time for those born btwn One time for those born btwn NR Yes
1965 1945–1965 1945–1965
Blood pressure Consider each visit Consider each visit Consider each visit Consider each Uncertain
visit
Height Once a year Once a year Consider Consider Uncertain
Weight Each visit Each visit Each visit Each visit Uncertain
 Lifestyle counseling
o Physical activity (annually)
 Emphasize advantages:  M&M, promotes mobility,  CAD, osteoporosis, many other dz’es, improves psych health, promotes fxnal
independence, & prevents falls
 Rec a program that balances exercise for: Flexibility (eg, stretching), Endurance (eg, walking, cycling), Strength (eg, weight training),
Balance (eg, Tai Chi, dance)
o Nutrition (BMI each visit)
 USPSTF recommends that obese (BMI ≥30) adults be offered intensive counseling and behavioral interventions to promote sustained
weight loss
 Ideal BMI may be higher for older adults than middle-aged adults
 BMIs between 25–29 are a/w the lowest mortality risk for adults ≥70 years old, which may be b/c of benefits from greater
nutritional reserve
 Weight loss, especially when combined with exercise, may improve physical function and ameliorate frailty among obese older adults
 Malnutrition and undernutrition are common yet frequently unidentified problems in the geriatric population; 15% of older outpatients
are malnourished
o EtOH misuse (initially, then if ssx)
 Light to moderate alcohol consumption in middle-aged to older adults has been associated with some health benefits, such as reduced
risk of coronary heart disease
 Moderate drinking is defined as 1 drink or less per day for adults > 65 years old
 Ask all older adults ≥ 65 years old annually about their alcohol use
 Use Alcohol Use Disorders Identification Test (AUDIT) or a single screening question, “How many times in the past year have you had
4 or more drinks in a day?”
o Smoking cessation (q visit)
 Ask all adults about tobacco use
 Emphasize that cessation at any age reduces rates of COPD, many cancers, CAD
 Designate quit date, discuss therapies to aid cessation, and provide close medical f/u (in person or by phone within 3-7 days of the quit
date and monthly for 3 months)
o Sexual dysfxn & STIs (routinely)
 USPSTF recommends routine screening of adults up to age 65, and screening of adults > 65 years old who are at increased risk
 Provide high-intensity behavioral counseling (multiple sessions) to prevent STI for older adults at high risk
 Geri health issues screening
o Assessing for Geriatric Health Issues is an essential component of Medicare’s wellness exams
o Comprehensive Geriatric Assessment has been associated with improvements in general well-being, life satisfaction, instrumental activities of daily
living, and fewer clinic visits
Issue Screening recommendation

Falls Annually

Incontinence Annually

Cognitive status If symptomatic

Depression Annually

41
Vision Annually

Hearing Annually

Nutrition Obtain weight each visit and height annually; calculate BMI

Mistreatment of older
adults Question with clinical suspicion

Check smoke detectors and carbon monoxide detectors, check water


Safety and preventing heater temperature, use sun protection, assess driving skills, wear seat
injury belts, complete advance directives and determine health care proxy

 Immunizations
o Influenza  annually
o TD booster  q 10 yrs
o TDAP  1 dose for everyone
o Herpes zoster  once after 60 y/o
o Pneumococcal  once after 65 (PCV13 and PPSV 23 in series)
 Chemoprophylaxis
o ASA: ppx based on 10-yr risk of CAD, age, life expectancy, no at inc risk of bleeding, willingness to take ASA for 10+ years
 USPSTF recommends
 50 – 59 y/o + >10% 10-yr CVD risk
 60 – 69 y/o + > 10% 10-yr CVD risk  indv decision
o Calcium + VItD
 USPSTF recommends against daily supplementation with < 400 IU of Vit D and < 1000 mg Ca
 High dose VitD is a/w prevention of fx or other dz
 Recommend Vit D 600 IU for adults 50 – 70 y/o and 800 IU for > 70 y/o
 1,200 mg/d of Ca in women > 50 y/o and men > 70 y/o & 1000 mg/d for men 51 – 70 y/o
 Counseling on CA screening
o When discussing cancer or other screening tests with older adults, clinicians should:
 Indicate whether any data suggest that the screening test improves older adults’ quality or quantity of life
 Discuss the risks of screening, including discomfort from undergoing the test itself, anxiety, potential complications from diagnostic
procedures resulting from a false-positive test, false reassurance from a false-negative test, and overdiagnosis/diagnosis of tumors that
are of no threat and that may result in overtreatment
o Older adults should be asked how they view the potential benefits and harms of different screening tests, so that their values and preferences are
considered in screening decisions
o Health maintenance discussions among older adults with limited remaining life expectancies should focus on measures that have benefits likely to
be achieved in a short time frame (eg, counseling on home safety, falls prevention, immunizations)

Cases / Questions
Case #1
• A 70-year-old woman comes in for a routine visit.
• History: degenerative joint disease, hypertension, urinary incontinence, hypercholesterolemia
• Medications: metoprolol, acetaminophen
• She is not sexually active and never had children.
• She has had no surgeries, and never had a Pap test.
• Which one of the following is the most appropriate screening recommendation for cervical cancer for this patient?
A. Screen once.
B. No screening is necessary.
C. Screen twice over the next 5 years.
D. Screen only if she becomes sexually active.

ANSWER: C
This patient has not had adequate screening for cervical cancer. She does not meet the USPSTF criteria to discontinue screening for cervical cancer in women >65 years
old who have had adequate prior screening and are not otherwise at high risk of cervical cancer. The American Cancer Society (ACS) defines adequate prior screening as
3 consecutive negative cytology results or 2 consecutive negative results for HPV within the previous 10 years, with the most recent test undertaken within the previous
5 years. Thus, this patient should be screened twice (2-5 year intervals between screenings).
Although sexual activity, particularly activity with multiple partners, is a RF for cervical CA, it does not alone determine the need for screening. If this patient had
adequate prior screening, with (-) cytology results, and became newly sexually active, she would not need to be screened again. However, given her lack of prior screening,
she needs to be screened, irrespective of whether she engages in sexual activity.
The USPSTF recommends against screening for cervical cancer in women who have had a hysterectomy with removal of the cervix and who do not have a history of
high-grade precancerous lesion (CIN grade 2 or 3) or cervical CA. For women who have had spontaneous regression or appropriate management of a high-grade
precancerous lesion, routine screening should continue for at least ≥20 years after, even if this extends screening past age 65. The ACS further states that screening should
not resume after cessation in women ≥65 years old, even if there is a new sexual partner.
Screening may be indicated in women ≥65 y/o when prior screening cannot be accurately assessed or documented. Women with limited access to care, minority women,
and women from countries where screening is not available are less likely to meet criteria for adequate prior screening. The USPSTF guidelines recognize that certain
considerations may support screening in women ≥65 years old who are at high risk, including women who have had HPV or HIV, have a compromised immune system,
were exposed to diethylstilbestrol in utero, or were previously tx for a high-grade precancerous lesion or cervical CA.
Women who have had a hysterectomy with removal of the cervix and who do not have a history of a high-grade precancerous lesion or cervical cancer are not at risk of
cervical cancer and should not be screened. Women who had their cervix removed during surgery for ovarian or endometrial cancer are not at high risk of cervical cancer
and would not benefit from screening. Among older women particularly, clinicians should confirm that the cervix in fact was removed. It is not unusual for older patients
or their families to be uncertain as to the extent of ovarian or cervical surgery.
The USPSTF cutoff at age 65 years is based on the incidence and prevalence of cervical CA, which peak in the mid reproductive years and begin to decline roughly
between ages 30 - 40 (this general pattern has also been seen in studies of previously unscreened women). Cervical CA in older women is not more aggressive or rapidly

42
progressive, and the rate of high-grade squamous intraepithelial lesions diagnosed by cytology is low among older women who have been previously screened. In modeling
studies of age at which to end screening, varying the age by 5-year intervals (from ages 65–95 years) provides <1 additional life-year beyond age 65 in women who have
been previously screened. In addition, there is inc risk of harm a/w false-positive results and subsequent testing with colposcopy and cervical bx.
By comparison, studies show that screening previously unscreened women would  mortality by 74%. A strategy of screening previously unscreened women q 2–5 years,
ending at age 70–75 yrs, represents a reasonable tradeoff between benefits & harms. Women with a clearly inadequate screening hx are those who have never been
screened or have not been recently screened before age 65 yrs. ~1/2 of invasive cases of cervical CA are dx’ed in women who have never been screened or have not been
screened in the last 5 yrs (another 10% occur in women who did not have appropriate f/u for an abnl Pap). ~ 42% of women ≥65 y/o w/ dx’ed cervical CA had never been
screened. Thus,  the burden of cervical CA mortality can be best achieved by focusing on who have not been adequately screened.
Conversely, overscreening is a/w several risks, including psychologic distress and time and burden (for ps and caregivers), and pain, bleeding, or d/c from colposcopy
and bx consequent to false-positive results.

Case #2
• An 84-year-old woman comes in for routine follow-up.
• History: hypertension, diabetes mellitus, degenerative joint disease, hypothyroidism
• Medications: metoprolol 50 mg/d, metformin 1,000 mg/d, levothyroxine 50 mcg/d, acetaminophen 650 mg three times daily as needed for pain
• Examination
• Blood pressure 150/70 mmHg
• Heart rate 72 bpm, regular with no murmurs
• BMI 34 kg/m2
• No dependent edema
• Laboratory findings:
• Hemoglobin 11.9 mg/dL
• Hematocrit 37.4%
• Glomerular filtration rate >60 mL/min
• Hemoglobin A1c 7.4%
• Thyrotropin 3.4 mIU/L
• Which one of the following would be the most important intervention for this patient?
A. Limiting salt intake to <1,500 mg/d
B. Weight loss and exercise
C. Calcium and vitamin D supplementation
D. Management of hyperglycemia
E. Balance training and stretching exercises

ANSWER: B
The prevalence of obesity in the United States is >30% in adult men and women. Obesity is a/w increased risk of CAD, type 2 DM and insulin resistance, various CA
(esp ones that are hormonally related, eg, large-bowel CA), gallstones, and disability. These comorbid medical conditions are a/w higher use of health care services and
costs. The prevalence of obesity in adults >80 years old is about one-half that of adults 50–59 years old; nonetheless, >15% of older Americans are obese. Several factors
contribute to obesity in older adults, including increased sedentary behavior and hormonal changes that contribute to the accumulation of fat. Aging is a/w a decrease in
growth hormone secretions, reduced responsiveness to thyroid hormone, decline in serum testosterone, and resistance to leptin.
The most important recommendation for this patient would be to focus on changing behavior with regard to diet and physical activity to achieve sustained weight loss.
The USPSTF recommends that persons who have a BMI >30 kg/m2should receive counseling and behavioral interventions related to both diet and exercise (optimally,
aerobic activity).
The USPSTF found adequate evidence that intensive, multicomponent behavioral interventions for obese adults can lead to average weight loss of 4–7 kg (8.8–15.4 lb).
These interventions also improve glucose tolerance and other physiologic risk factors for cardiovascular disease. The USPSTF found inadequate direct evidence about
the effectiveness of these interventions on long-term health outcomes, such as decreasing CVD, hospitalizations, or death rates. There are, however, significant short-
term benefits to weight loss, such as improving blood pressure, diabetes, and physical function. There is adequate evidence that the harms of screening and behavioral
interventions for obesity are small. The USPSTF concluded that the net benefit of screening is moderate.
Current guidelines allow for sodium intake of 1,500 mg/d; limiting intake to <1,500 mg/d is not necessary. Behavioral counseling for management of hyperglycemia &
balance training & stretching exercises, may be useful but are not as important as wt loss in this obese pt. Balance & stretching activities are not likely to influence wt
loss & thus should not be recommended for this purpose.

Case #3
• A 62-year-old white woman comes to the office to establish care.
• History: degenerative joint disease in both knees, breast cancer (diagnosed at age 41)
• Bilateral mastectomy and prophylactic oophorectomy (family history of ovarian cancer)
• After surgery, 6-month course of chemotherapy with doxorubicin and cyclophosphamide, followed by tamoxifen for 2 years (discontinued because
of side effects)
• No recurrent disease or further treatment
• Medication: none
• She does not smoke, and drinks alcohol only socially (never >8 oz of wine at one sitting).
• Physical activity: 40–100 minutes of moderate-level weight-bearing exercise daily (walking and running on a treadmill)
• Vegetarian diet, including animal by-products (1 hardboiled egg and 3 oz of cheese daily)
• Height: 147 cm (4 ft 10 in.) ; Weight: 40.8 kg (90 lb)
• Which one of the following is the most appropriate recommendation regarding bone density testing for this patient?
A. Begin at age 65.
B. Begin testing now.
C. Begin at age 75.
D. Begin testing after a fracture occurs.

ANSWER: B
This patient is at risk of osteoporosis because of her early oophorectomy, subsequent premature and early menopause with no replacement therapy, limited calcium intake,
and small stature. She should undergo screening before age 65. Waiting until she has a fracture would not give her the opportunity to benefit from potentially useful
interventions.
The USPSTF recommends routine bone density screening for all women, irrespective of race or ethnicity, who are ≥65 years old with no history of osteoporotic fracture,
osteoporosis secondary to another condition, or other specific clinical indications for measurement of bone density. Screening is recommended for women <65 years old
who are at high risk of osteoporosis (ie, women who have the same or greater 10-year fracture risk compared with a 65-year-old white woman without additional risk

43
factors). There is no upper age limit for screening in women because the risk of fracture increases with age and treatment harm remains small. Clinicians should take
into account the patient's remaining life span when deciding whether to screen patients with significant illness: In the Fracture Intervention Trial, benefits emerged 18–24
months after initiation of treatment. According to USPSTF, there is insufficient evidence regarding benefits of screening men for osteoporosis, and the balance of benefits
and harms cannot be determined. However, the National Osteoporosis Foundation recommends screening men at age 70 years.
By 2020, approximately 12 million Americans >50 years are expected to have osteoporosis. One-half of all postmenopausal women will have an osteoporosis-related
fracture during their lifetime; in 25% of these women, a vertebral deformity will develop, and 15% will have a hip fracture. Osteoporotic fractures, particularly hip
fractures, are associated with chronic pain and disability, loss of independence, decreased quality of life, and increased mortality. Hip fractures are less common in men
than in women, yet one-third of men who have a hip fracture die within 1 year.
The USPSTF found convincing evidence that bone measurement tests predict short-term risk of osteoporotic fractures in women and men. The most common tests are
dual-energy x-ray absorptiometry (DEXA) of the hip and lumbar spine and quantitative ultrasonography of the calcaneus. There is adequate evidence that clinical risk
assessment instruments have modest predictive value for low bone density or fractures (SOE=B). No controlled studies have evaluated the effect of screening for
osteoporosis on fracture rates or fracture-related morbidity or mortality.
There is no evidence in regard either to optimal intervals for bone density screening or to whether women with normal bone mineral density should repeat screening.
Because of limitations related to test precision, an interval of ≥2 years may be needed to reliably measure a change in bone mineral density.
Although no harm is a/w DEXA testing, there may be harms a/w pharmacologic treatment. Nonetheless, info on changes in bone mineral density may reinforce behavioral
interventions such as diet and exercise.
Drug therapy is available for primary prevention (prevention of osteoporotic fracture in patients with low bone mineral density who have no previous fractures) or
secondary prevention (prevention of osteoporotic fracture in patients who have a known previous osteoporotic fracture). Therapies include bisphosphonates, teriparatide,
raloxifene, estrogen, denosumab, and calcitonin.

Transitions of Care

 Transitions = movement of pt from one set of providers, level of care, or health care setting to another
 1/5 pts are re-hospitalized w/in 30 d
 Suboptimal transitions
o Can result in adverse events, medication errors, inaccurate or incomplete information transfer
o Almost half of all medication errors occur during hospital admission or discharge
o Inaccurate or incomplete information can result in:
 Delayed diagnosis
 Duplicative medical services
 Hospital readmission
 Reduced patient and provider satisfaction
 Risks for suboptimal transitions
o Patient level
 Education less than high school
 No help with ≥1 deficit in an activity of daily living
 Limited self-management ability
 Worse self-rating of health
 Living alone
 Transition to home with home-care services
 Prior hospitalization
 Long hospital length of stay
 Low income or Medicaid-eligible, including homelessness
 Older age
 Five or more comorbidities
 Depression, CVD, diabetes mellitus, cancer, substance abuse
o System level
 Communities with high hospital admission rate
 Lack of discharge education
 Insufficient communication across care settings
 Failure in implementation of care plan (durable medical equip, home health, f/u appts, mediations, tests)
 Common barriers
o Diverse needs depending on illness, social situation, and type of transition
o Lack of provider education and feedback, for example:
 No timely, effective discharge summary
 Lack of knowledge of the capabilities of various post-acute care settings
o Difficulty communicating with colleagues at the previous or next site of care
o Lack of time or financial resources; lack of reimbursement in the current US system
 Components of optimal transitional care
o Accurate and timely transfer of information to the next set of providers
o Empowerment of the older adult to assert his or her preferences
o Comprehensive assessments of older adult and caregiver needs
o Comprehensive medication review and management
o Logistical arrangements related to executing the transition
o Education to prepare older adults and caregivers for what to expect at the next site of care
o Support for self-management of medical conditions
o Coordination among medical and community resources
o Follow-up and support after discharge
 Major interventions
Intervention Goal of Intervention Key Component Interventions Demonstrated Outcome
Improvements

44
Care Transitions Intervention Minimize rehospitalization of older Post-di/c: f/u call; home visit Reduction in readmissions
(www.caretransitions.org) patients with complex care needs Transition: pt-centered d/c instructions Improvement in care transition quality
score

Re-Engineered Discharge Minimize rehospitalization of diverse Pre-d/c: pt education, d/c planning, Rx Reduction in ED and hospital
(“Project RED”) inpatient populations reconciliation utilization within 30 days
(www.bu.edu/fammed/projectre Post-d/c: timely communication w/ PCP,
d/) f/u call
Transition: pt-centered d/c instructions

Transitional Care Model Minimize rehospitalization of older Pre-d/c: pt education, d/c planning Reduced readmissions, increased time
(www.transitionalcare.info/) patients with complex care needs Post-discharge: f/u call, hotline between discharge and readmission,
Transition: transition coach, pt-centered decreased cost of providing health
discharge instructions care

Better Outcomes by Optimizing Minimize rehospitalization of patients Approach not stated; rather, each site Reduction in 30 day readmission
Safe Transitions (BOOST) at high risk of readmission receives a mentor to develop its own
strategies based on local needs
assessment and existing best practices

Guided Care Managing complex older adults in Pre-discharge: pt education, d/c Tended to use less home health
(www.guidedcare.org/) ambulatory care or home care, and planning, Rx reconciliation services
through transitions Post-d/c: timely clinic f/u, timely Reduced costly health care utilization
communication, f/u call, post-d/c in integrated health care systems
hotline, home visit
Transition: coach, pt-centered d/c
instructions, prov continuity

Interventions to Reduce Acute Minimize rehospitalization of Post-discharge/Transition: Reduction in readmissions


Care Transfers (INTERACT) nursing-home residents • Timely staff communication
(http://interact2.net/) of  in clinical status
• Evidence-based clinical care
pathways triggered by
changes in clinical status
• Advance care planning

Discharge of Elderly from the Reducing risk of older adults’ return Pre-discharge: comprehensive geriatric Reduced rate of hospitalization within
Emergency Department (DEED) to the ED assessment the first 30 days
(www.ncbi.nlm.nih.gov/pubmed Post-discharge/Transition: Reduced rate of ED admission for 18-
/10408663) • Timely communication with months after index ED visit
PCP Longer time to the first repeat ED visit
• Home visit Maintained greater degree of physical
• Formulation of care plan by and mental function
IP team
• Transition pt to use
community services

Geriatric Resources for Reducing ED visits, hospital Post-discharge /Transition: Improved patient-centered care
Assessment and Care of Elders admissions & readmissions, & • Home-based care mgt by NP transitions, reduced hospital
(GRACE) nursing-home admissions for frail & social worker readmissions and nursing-home
(http://graceteamcare.indiana.ed older adults with complex needs • Collaborations with PCP and placement
u/case-for-grace.html) geri IP team
• Care protocols for geriatric
conditions

 Discharge destinations
o Home with family support
o Home with home-health care
 Works well for older adults requiring only intermittent skilled services (nursing, PT, or speech therapy)
 Older adults with one of these needs may also receive assistance under Medicare from OT, medical social work, or home-health aides
 Medicare requires that older adults receiving home-health care be homebound
o Custodial care (eg, assisted living or “nursing home”)
o Skilled-nursing facility (SNF)
 Under Medicare, older adults in SNFs must need a skilled service such as IV therapy, artificial nutrition and hydration, complex wound
care, ostomy care, or rehabilitation
 Medicare covers all or part of SNF care for up to 100 days after a qualifying hospital stay
 Coverage stops earlier if treatment goals are met or the older adult no longer demonstrates improvement
o Acute rehabilitation hospital
 For older adults with substantial rehabilitation needs and considerable rehabilitation potential
 Many older adults are ineligible because they cannot participate in 3 hours/day of intense therapy
o Long-term acute care
 For the rare pt who req prolonged hospital-level care such as long-term mechanical ventilation, multiple IV medications, parenteral
nutrition, or complex wound care
o Inpatient hospice
 D/C Rx Regimen

45
o Include:
 An indication for each medication
 Stop dates (eg, antibiotics) or tapering schedules (eg, for systemic corticosteroids), as appropriate
 Clear behavioral triggers for as-needed psychiatric medications
o Rx added during the hospital stay for use prn or for PPX, such as analgesics, PPIs, or laxatives, can be tapered or d/c
o Formally reconcile with the preadmission medication regimen
o Imperative to clearly document which:
 Medications that are new since admission
 Preadmission medications that have been stopped
 Dosages of continued medications that have been changed
 Info for caregivers
o When discharge is directly to home:
 Follow-up appointments
 Warning symptoms or signs to watch for, with instructions on whom to contact
 Clinical disciplines (eg, nursing, PT) contracted to provide services in the home
 Reconciled medication list
o When discharge is to another care setting:
 Nature of the new institution
 Identity of the new attending physician (if known)
 Expected frequency of provider visits
 Info for new clinicians
o Directly communicate about:
 Critical but pending study results
 Nuances of goals of care
 Family dynamics
o Otherwise, a brief, prompt discharge summary suffices:
 Summary of hospital course with care provided and results of important tests
 List of problems and diagnoses
 Functional and cognitive status at baseline and at discharge
 Reconciled medication list
 Allergies
 Test results still outstanding
 Follow-up appointments
 Goals, preferences, and advance directives
 Best contact information for the discharging clinician in case any questions arise
 3 steps to improve transitions
o Set expectations for both the sending and receiving provider teams
 The National Transitions of Care Coalition recommends shifting from the concept of “discharge” to that of “transfer with continuous
management”
 Based on the information available at admission, what needs will this older adult have after transfer?
 What are the patient’s and caregiver’s preferences?
 How will the patient care for himself or herself after transfer?
 What other clinicians need to evaluate the older adult to formulate an effective care plan?
 Do the elderly & caregiver understand the purpose of the transfer & what to expect at the next site of care?
 Has the next site of care received, understood, and clarified discrepancies about the care plan?
o Tailor communication strategies to the type of information being communicated and the type of transition
 Based on this pt’s current epi of illness, what’s the most relevant info to communicate to the next site?
 Should the information also be given directly to the older adult and caregiver?
 How should this information be communicated?
 Electronically ― for notification of admission, discharge, or nonurgent issues
 Verbally ― for situations of urgency or uncertainty, or those with complex social dynamics
 In writing ― for information that must be a part of the medical record or used as a reference by the older adult, caregiver, or
clinician
o Target specific processes or outcomes for improvement, using established QI methods
 Begin by focusing on 1 or 2 measures, then expand once initial goals are achieved
 Examples:
 Communication with PCP before older adult’s transfer
 Medication reconciliation at the time of transfer
 Older adult’s, caregiver’s, or receiving clinician’s satisfaction with the quality of the transition
 Timeliness of arrival of transfer summaries
 Inclusion of various components in transfer summaries (eg, doc of cog & fxnal status)
 Ease of scheduling follow-up appointments
 Frequency of health care usage after transfer

Cases / Questions
Question #1
Which one of the following transitional care interventions is most likely to reduce the rate of hospital readmission?
A. General patient education (self-care instruction, symptom management, and medication counseling)
B. Designated transitional care provider
C. Risk assessment before discharge
D. Facilitated follow-up with patient’s primary care provider

ANSWER: B
The effort to reduce 30-day ED visits and hospital readmission of Medicare pts has yielded a variety of strategies. The reviewers found a significant reduction in 30-day
emergency department use, readmission, or other composite endpoints in trials that had a dedicated transition provider who contacted general medical patients before and

46
after discharge. Included in the review were 3 trials of Care Transitions Intervention (in which a transitions coach emphasizes patient education and self-management
during home visits) that reported significant reductions in 30-day readmissions in diverse hospitals and in integrated and nonintegrated care systems.
Thirty of the trials combined pre- and post-discharge interventions. The review established that, in trials that did not have a dedicated transition provider but used other
hospital-initiated pre- and post-discharge interventions, alone or in combination, there is little evidence that the interventions had an effect on post-discharge adverse
events in general medical patients. The other interventions included Rx reconciliation, risk assessment, improved communications with outpatient providers, facilitated
f/u in ambulatory care, improved pt engagement strategies, individualized lay-language records for patient and caregiver use after d/c, interdisciplinary discharge planning
teams, and outreach to patients through follow-up telephone contact, patient hotlines, and home visits. The most common component was patient engagement, which
ranged from patient education to tailored instructions concerning symptom and medication management. A few studies of clinical pharmacist-led medication safety
interventions show promise.

Which one of the following statements is true about rehospitalization rates after discharge from a postacute nursing facility?
A. Skilled-nursing facilities are not eligible to participate in the Bundled Payment Care Initiative (BPCI) for postacute care.
B. There are no data that assess the effect of interventions on reducing the rate of rehospitalization.
C. More than 20% of pts who are d/c from Medicare-eligible SNF are rehospitalized within 30 days of discharge.
D. The Initiative to Reduce Avoidable Hospitalizations among Nursing Facility Residents provides recommendations to reduce the rate of rehospitalization after
discharge from a skilled-nursing facility.

ANSWER: C
Established by the Centers for Medicare and Medicaid Services, the Bundled Payment Care Initiative (BPCI) aims to align incentives for various providers (hospitals,
postacute care facilities, physicians, and other practitioners), allowing them to work across specialties and settings to yield higher-quality, better-coordinated care at a
lower cost to Medicare. Improved transitions from postacute nursing providers, including SNF, to lower levels of care are expected to be a major factor in the success of
BCPI. In 2012, 23% of Medicare patients discharged from a SNF were rehospitalized within 30 days; there were >2.5 million Medicare d/c from a skilled-nursing facility.
To date, there are few studies on programs that provide transition for patients in a SNF to lower levels of care following an index hospitalization and a short-term stay at
a postacute nursing facility. Two single-site, quasi-experimental studies used pre–post designs to measure the effect of transitional care programs on 30-day
rehospitalization and other outcomes. One pilot study investigated the impact of a 1-time visit, at or just before discharge from a skilled-nursing facility, to a transitional
care clinic at a VA hospital. Outcomes measured were 30-day rehospitalization and ED visits. The transitional care clinic was staffed by an advanced NP who conducted
Rx reconciliation; made arrangements for medical supplies, equipment, and home health care; provided patient and caregiver education; and communicated with outpatient
primary care and other providers through the VA’s computerized pt record system. The 30-day rehospitalization rate was 14% among pts who visited the transitional care
center, whereas the rate was 23% before the center had been established. There were also fewer acute inpatient days in the 30-day period after discharge from the SNF.
A second, similar study was based at a large, urban SNF and included Medicare patients admitted from a number of short-stay acute-care hospitals. The Project Re-
Engineered Discharge (Project RED) checklist approach was adapted for use in the nursing facility’s 50-bed subacute unit to ensure comprehensive transition planning,
patient and family engagement, and connection with community clinicians after discharge from the nursing facility. Before the intervention, the 30-day rehospitalization
rate among patients discharged from the facility was 18.9%; after initiation of Project RED, the rate was 10.2% (P<.05; with inverse probability [propensity] weighting,
P=.045). In sum, results from the 2 single-site quasi-experiments supported the potential of transitional care interventions to reduce 30-day rehospitalizations.
There has been considerable research on inappropriate hospitalization of residents of nursing facilities, many of whom are dually eligible for Medicare and Medicaid and
occupy long-term care beds. Many nursing facilities have SNF covered by Medicare as well as long-term beds or units, and some of the strategies used to reduce
hospitalizations of long-term residents (such as Interventions to Reduce Acute Care Transfers [INTERACT II]) may apply to transitional care between the skilled nursing
facility and the community. However, like INTERACT II, the CMS Initiative to Reduce Avoidable Hospitalizations among Nursing Facility Residents (2013) is not
directed toward improving transitions from postacute care to the community.

Question #3
Which one of following interventions has been found to reduce in-hospital deaths in patients at the end of life?
A. Use of teams comprising specialists from oncology, palliative care, social work, and office-based nursing
B. Increased home nursing care
C. Enhanced access to patient-centered medical home model of primary care
D. Enhanced access to team of specialists in palliative, nursing, and primary care

ANSWER: D
There is evidence that community-based specialist palliative care teams improve symptom management, quality of life, and satisfaction with care. There is less consistent
evidence on how these services affect hospital and ED use at the end of life, and on the proportion of in-home versus hospital deaths. Three of 7 trials found that patients
who had in-home palliative care had significantly fewer end-of-life hospitalizations and emergency department visits than patients who had usual care.
A recent systematic review and meta-analysis identified 9 controlled clinical and randomized controlled trials that examined the impact on home deaths of adding or
increasing home nursing hours to in-home specialist palliative care services. The pooled odds ratio (OR) indicated a significant effect of services on home deaths. However,
in the 2 trials for which quality was rated as high, no effect was. Thus, the reviewers judged the present evidence on this question as inconclusive.
A retrospective cohort study examined the pooled effect of treatment by 11 in-home specialist palliative care teams providing services in different regions of Ontario,
Canada. The teams met several study criteria, including 1) provision of interdisciplinary, home-based palliative care; 2) inclusion of palliative care physicians, family or
primary care physicians, and nurses; and 3) round-the-clock availability. Propensity analysis was used to match patients receiving these services between 2009 and 2011
(N=3,109) with a control group that received usual care (about 80% had a cancer diagnosis, and 80% were receiving end-of-life services). Outcomes measured were
hospitalization or ED visit within the last 2 weeks of life, and dying in hospital. Of the patients who had in-home palliative care, 31.2% were hospitalized and 28.9% had
an ED visit in the last 2 weeks of life; the rates for the control group were 39.3% and 34.5%, respectively. The pooled relative risks were 0.65 and 0.77, respectively.
Fewer patients who received in-home palliative care died in hospital than control pts, with a pooled relative risk of 0.46.

Urinary Incontinence

 Stats
o 15-30% of community dwelling older adults
o 60-70% of residents of long-term institutions
o Prevalence increases w/ age
o Women > men (2:1) until 80 then 1:1
 Urinary incontinence = involuntary leakage of any amount of urine
 RF: obesity, fxnal impairment, dementia, Rx, environmental barriers to toilet access
 Comorbidities that cz or worsen UI: affective/anxiety d/o, alcoholism, AVD, chronic cough, CHF, constipation, DJD, delirium, DM, hyperCa, mobility
impairment, MS, NPH, Parkinson Dz, peripheral venous insufficiency, psychosis, RA, sleep apnea, spinal cord injury, spinal stenosis, stroke, VitB12 def
 Rx that cz or worsen UI: EtOH, -adrenergic agonists, -adrenergic blockers, ACE-I, antichol, antipsych, CCB, cholinesterase inhibitors, PO estrogen,
GABAergic agents, loop diuretics, narcotic analgesics, NSAIDs, sedative hypnotics, thiazolidinediones, TCAs

47
 Age related LUT changes: decreased bladder contractility, increased uninhibited bladder contractions, diurnal urine output shifted later in day, sphincteric
striated mm attenuation, decreased bladder capacity, increased postvoid residual (PVR), decreased urethral closure pressure, increased vaginal mucosal
atrophy, BPH, prostate hypertrophy
 Pathophys
o Urge UI  detrusor overactivity (DO) = uninhibited bladder contractions
 Can be age-related, idiopathic, 2/2 lesion in central inhibitory pathways (stroke, cervical stenosis), d/t bladder outlet obstruction or local
bladder irritation
o Stress UI  impaired urethral sphincter support &/or closure
 Damage to pelvic floor supports (levator ani, connective tissues)  failure to adequately compress urethra
 Sphincter failure  Sx damage or severe atrophy, subsacral spinal cord injury (rare)
 Can cz “stress UI” when cough triggers uninhibited detrusor contraction
 Leakage is usu after cough w/ large volume & difficult to stop
o Mixed UI  both DO & impaired sphincter support/fxn
o UI w/ impaired bladder emptying
 Outlet obstruction
 Cz’es in men: prostate hyperplasia  prostate enlargement
 Cz’es in women: urethral scarring or large cystocele/prolapse that kinks urethra
 Detrusor underactivity results from:
 Intrinsic bladder sm.mm damage (ischemia, scarring fibrosis)
 Peripheral neuropathy (DM, Vit B12 def, alcoholism)
 Damage to spinal cord or spinal bladder efferent nn (disc herniation, spinal stenosis, tumors, deg neuro dz)
 Nocturia
o Nocturnal polyuria (nocturnal output > 33% of total 24-hr urine output)
 Late day/evening fluids, esp w/ caffeine or EtOH
 Pedal edema (Rx, venous stasis, HF)
 HF
 OSA
o Sleep disturbance
 OSA, Rx, cardiac/pulm dz, pain, restless leg syndrome, depression, sleep partner
o Lower urinary tract
 Detrusor overactivity, BPH, impaired bladder emptying

Follow up questions to determine type of UI


 Do you leak urine most often:
o When you are performing some physical activity, such as coughing, sneezing, lifting, or exercising? (stress UI)
o When you have the urge or feeling that you need to empty your bladder but cannot get to the toilet fast enough? (urge UI)
o With both physical activity and a sense of urgency? (mixed UI)
o Without physical activity and without sense of urgency? (other)
 Onset, freq, vol, timing, exacerbating/ameliorating factors, LUT ssx, amt/types of fluid intake, success/failure past Tx, current mgmt
 Red flag ssx: abrupt onset, pelvic pain, hematuria (could herald neurologic disease or cancer—prompt evaluation and referral required)

Testing
 Postvoid Residual (PVR)  high PVR = > 200 mL
o Not routinely necessary and definition of “increased” PVR not standardized
o Done by catheterization or ultrasound
o Consider PVR measurement in those with:
 Prior urinary retention
 Longstanding diabetes
 Recurrent UTIs
 Severe constipation
 Complex neurologic disease
 Higher than routine risk for prostate enlargement (men)
 Marked pelvic organ prolapse or prior UI surgery (women)
 Urodynamic testing
o Routine not necessary or desirable
o Consider if: cz is unclear & knowing would change management, empiric tx has failed & pt would consider invasive Tx / Sx

Management
 Lifestyle changes
o Wt loss reduces stress UI
o Avoid extreme fluid intake, caffeinated drinks, EtOH
o Minimize evening intake
o Quit smoking (decreases cough)
 Behavioral therapy
o Bladder training & pelvic mm exercise (PME)  good for urge, stress, & mixed
 Bladder training for cognitive pts
 Urgency suppression using CNS & pelvic mechs
o Be still, don’t run to bathroom  do several pelvic mm contractions  urgency decreases, pee
 Frequent voiding to keep vol low
o Initial toileting freq  about 2 hrs or use shortest interval
o Increase time by 30 – 60 min btwn until reach a comfortable level
o Bladder training for cognitively impaired pts
 Prompted voiding  urge UI
 Caregiver will…
o Monitors the patient and encourages him or her to report any need to void

48
o Prompts the patient to toilet on a regular schedule during the day (usually every 2–3 hours)
o Leads the patient to the bathroom & gives the patient positive feedback when he or she toilets
o PME req motivated pt & careful instruction
 Perform an isolated pelvic muscle contraction; avoid buttock, abdomen, thigh muscle contraction. Hold 6–8 seconds
 Repeat the contraction 8–12 times (one set); relax the pelvis between each contraction
 Complete 3 sets daily at least 3–4 times a week; continue at least 15–20 weeks
 Drugs
o Antimuscarinic agents
 Mod effective for urge, overactive, and mixed
 CI in pts w/ narrow-angle glaucoma, impaired gastric emptying, or urinary retention
 Works by increasing bladder capacity, but does not ablate uninhibited contractions
 Routine PVR monitoring is unnecessary  only check in worsening UI
 Key SE: dry mouth, caries, constipation, cognitive impairment (class effect)
 DO NOT combine w/ cholinesterase inhibitors
Agent Dosing Considerations

Oxybutynin IR 2.5–5 mg q6–12h

Oxybutynin ER 5–20 mg/day

Oxybutynin patch 3.9 mg/24 h, 2x/week

Oxybutynin gel 3% gel or 10% sachet daily P450 interactions

Tolterodine IR (Deltrol) 1–2 mg q12h

Tolterodine ER 2–4 mg/day P450 interactions; consider dose adjustment in renal impairment

Trospium IR 20 mg q12h or q24h


Renal clearance; give QD in pts w/ renal impairment; should be taken on
Trospium ER 60 mg daily in AM empty stomach

Darifenacin 7.5–15 mg/day P450 interactions

Solifenacin 5–10 mg/day P450 interactions; consider dose adjustment in renal impairment

P450 interactions; prodrug that is metabolized to tolterodine; consider dose


Fesoterodine 4–8 mg/day adjustment in renal impairment
o Mirabegron
 3-agonist  stimulates detrusor relaxation + increases bladder capacity
 Dose: 25 – 50 mg/d
 Mod efficiency + no cognitive adverse effects
 May increase BP
 Drug-drug interactions: digoxin, metoprolol, venlafaxine, desipramine, dextromethorphan
o Duloxetine (off-label) decreases stress UI
o Vaginal estrogen for comfort, may decrease recurrent UTIs
o DO NOT use vasopressin for nocturia  risk of hypoNa
o Insufficient evidence for propantheline, dicyclomine, imipramine, hyoscyamine, CCB, NSAIDs, flavoxate
 Surgery
o MIS
 Sacral nn neuromodulation for urge refractory to Tx & urinary retention
 Posterior tibial nn stimulation
 Intravesical injection of botox for refractory urge UI
 Pessaries for stress or urge UI exacerbated by POP
o Highest cure rate for women  colposuspension (Burch operation), slings (midurethral, bladder neck)
o Periurethral injection for short term (≤ 1 yr)
o Artificial sphincters for refractory stress from sphincter damage
 UI in nursing home residents
o F-tag 315 for eval & management of UI & urinary caths
o Interventions that combine prompted voiding with bedside exercise improve both incontinence and physical function
 Prompted voiding should be tried in all eligible patients
 Continue in those who are able to accept & follow the prompt to toilet at least 75% of the time in first 3 days; “check and change” for
those who do not respond
o Caths w/ caution
 Significant morbidity: polymicrobial bacteriuria, febrile episodes, nephrolithiasis, bladder stones, epididymitis, chronic renal
inflammation, pyelonephritis, meatal damage
 Inappropriate and/or poorly documented indications for catheter use are major focus in F-tag 315
 Reserve indwelling catheters for:
 Short-term decompression of acute urinary retention
 Chronic retention not surgically/medically remediable
 Patients with wounds that must be kept clean of urine
 Very ill patients who cannot tolerate garment changes
 Care for catheters
 Keep long-term-care residents w/ catheters in separate rooms from each other to decrease cross-infection

49
 Bacteriuria is universal in catheterized patients
o Do not treat in absence of clear symptoms of infection
o Do not routinely culture or re-culture after UTI Rx
 Culture symptomatic patients after old catheter is removed and new catheter is placed
 Catheters don’t need to be changed routinely if monitoring is adequate & catheter blockage does not dvlp
o Changing every 30 days is reasonable in patients who cannot be monitored
 RF for catheter blockage: alkaline urine, female gender, poor mobility, calciuria, proteinuria, copious mucin, Proteus
colonization, bladder stones
 Causes of persistent leakage around the catheter: large Foley balloon, detrusor overactivity, bacteriuria, constipation or
impaction, improper catheter positioning

Cases / Questions
Case #1
• A 72-year-old woman has difficulty holding urine when she has the urge to void.
 The difficulty has increased over the past 2 years.
 For 5 years, she has leaked urine when she coughs or sneezes.
 The bladder problems interfere with her part-time job.
 She has no hematuria or dysuria.
• History: hypertension, hypothyroidism, osteoporosis, hypercholesterolemia
 3 pregnancies, each normal vaginal delivery
 Total vaginal hysterectomy 10 years ago for fibroids
• Medications: lisinopril, amlodipine, levothyroxine, pravastatin, alendronate (weekly), vitamin D
 She adheres to the drug regimen, but finds the quantity of pills burdensome.
• Physical examination
 Grade 1 pelvic organ prolapse
 No evidence of atrophic vaginitis
 She can perform isolated contraction of pelvic floor muscles.
 Cough provocation confirms stress incontinence.
• Ultrasonography of bladder: 40 mL post-void residual urine
• Urinalysis
 No leukocyte esterase, nitrates, or hemoglobin
 5–10 WBC/hpf
 No RBCs
 Urine culture: E. coli <10,000 colony-forming units

Which one of the following would be the most appropriate initial treatment?
A. Fesoterodine 4 mg/d
B. Three-day course of ciprofloxacin
C. Exercise-based behavioral therapy of pelvic floor muscle
D. Cystoscopic intradetrusor injection of onabotulinum toxin
E. Stimulation of the posterior tibial nerve

ANSWER: C
This patient has mixed stress and urge urinary incontinence. The positive standing cough test is highly specific for stress incontinence, and her difficulty holding urine
supports a diagnosis of urge incontinence. Behavioral therapy is an appropriate initial strategy for urinary incontinence in older adults. In a Cochrane review that included
only randomized trials, training of pelvic floor muscle improved outcomes for women with stress or mixed urinary incontinence, although outcomes and methods differed
between trials, and many of the trials were small. This patient is a strong candidate for behavioral therapy with pelvic floor muscle training (eg, Kegel exercises). Her
part-time job as a cashier suggests a high level of functioning, and she is able to demonstrate a strong pelvic floor muscle contraction during the examination. In addition
to the exercises, the patient should be taught to contract her pelvic floor muscles just before activities that usually induce stress leakage (eg, coughing, sneezing, lifting),
and to manage urgency by staying very still and repeatedly contracting the pelvic floor muscles (“freeze and squeeze”) until the urgency is gone, only then proceeding to
the bathroom. If the urgency returns on the way to the bathroom, she should again “freeze and squeeze.”
Fesoterodine is used to treat urge ssx and urgency UI but likely would not help stress symptoms. Also, additional drug therapy would further complicate a drug regimen
that the patient finds burdensome. Abx Tx is not appropriate because her urinary ssx have worsened gradually, she has no specific new or acute ssx, and the U/A does not
suggest bacteriuria. Given that the urinalysis is negative, positive culture results would more likely represent a contaminated collection or asymptomatic bacteriuria, and
Abx Tx would likely not improve the incontinence.
Stimulation of the posterior tibial nerve is approved by the U.S. FDA for patients for whom conservative behavioral and drug-based approaches have failed. The procedure
involves neuromodulation for overactive bladder symptoms such as urgency, frequency, and urgency incontinence. Cystoscopic intradetrusor injection of onabotulinum
toxin A is approved by the FDA for refractory symptoms secondary to overactive bladder. Neither of these modalities would address symptoms of stress urinary
incontinence.

Case #2
• A 90-year-old woman has had increasing urinary and fecal incontinence over the past 3 months.
 Her daughter, the primary caregiver, reports that the UI began 2 years ago and was managed with prompted voiding.
 There is now increased urinary frequency and fecal incontinence 2–3 times each week; the latter is particularly burdensome.
• The daughter will be unable to care for her mother at home if the situation persists.
• History: Alzheimer disease, hypothyroidism, osteoarthritis
• Medication changes in the past 6 months
 Simvastatin: discontinued
 Donepezil: starting dose of 5 mg titrated to 10 mg/d
 Levothyroxine: increased to 112 mcg/d
 Acetaminophen: 500 mg twice daily was added

Which one of the following would be the most appropriate strategy to consider?
A. Start extended-release oxybutynin 10 mg/d.
B. Start memantine 5 mg/d and titrate upward by 5 mg/wk.
C. Start loperamide 2 mg up to twice daily as needed.

50
D. Decrease levothyroxine dosage.
E. Decrease donepezil.

ANSWER: E
Incontinence is always a meaningful quality-of-life issue. In this case, UI has recently been compounded by increased frequency and new fecal incontinence. Resolution
is particularly important here because the daughter is approaching caregiver burnout.
Rather than continue a culprit Rx (in this case, donepezil) & add additional Rx to counter its SE (what has been termed the Rx cascade), recommended practice is to ID
the finding might be an adverse effect of drug therapy & d/c the offending agent.
Most pts who are Rx’ed antichol bladder relaxant Tx have no discernible cognitive decline, but some will have precipitous, identifiable cognitive SE. Cognitive SE of
oxybutynin are related to peak Rx levels. Immediate-release Rx w/ the same total dosage could potentially worsen this effect.
The mainstay drug therapy for ED is an AchE-I (procholinergic therapy), whereas urgency ssx & urge UI are Tx’ed w/ antimuscarinic anticholinergic Rx. There are data
that pts with dementia Tx w/ both donepezil & oxybutynin have > rapid fxnal decline than those Tx’ed w/ donepezil alone. The reverse—that bladder & bowel ssx may
dvlp in pts started on AchE-I Rx for dementia.
Starting oxybutynin or loperamide would be an ex of a Rx cascade, in which a 2nd Rx is prescribed to treat SE from a 1st Rx. Starting memantine is unlikely to address
the ssx. Decreasing levothyroxine would likely have no effect on the ssx & might be deleterious and  hypothyroidism.
In one study, nursing-home residents treated with a combination of cholinesterase inhibitors and antimuscarinics for UI experienced a greater physical functional decline
over 6 months than those treated with cholinesterase inhibitors alone.

Case #3
• A 74-year-old man has urge urinary incontinence.
 He is entering nursing home after rehabilitation for left parietal stroke.
 He has residual weakness in right upper and lower extremities.
 He ambulates with a walker and minimal assistance.
• History: diabetes, CAD, benign prostatic enlargement, depression, mild Alzheimer disease
• Medications: tamsulosin, lisinopril, metoprolol, insulin glargine and aspart, sertraline, aspirin, rosuvastatin
• Ultrasonography of bladder: 25 mL post-void residual minutes after an incontinence episode
• Urinalysis: trace protein, no leukocyte esterase, no nitrites, no hemoglobin

Which one of the following is the most appropriate next step in management?
A. Extended-release oxybutynin
B. Trial of prompted voiding
C. Finasteride
D. Urodynamic testing
E. Placement of a condom catheter

ANSWER: B
The most appropriate intervention for this patient is a trial of prompted voiding. From 25%–40% of similar patients respond well to this behavioral protocol, and
responsiveness can generally be determined after 3–5 days. Prompted toileting is more than timed toileting. The pt is taken to the toilet on a schedule (typically every 2–
3 hours), whether he or she feels the need to urinate or not. Pts who improve but have residual incontinence episodes may benefit from the addition of a bladder-relaxant
Rx (oxybutynin or tolterodine). In addition to typical antichol SE, bladder-relaxant drugs may worsen cognitive impairment or precipitate delirium in patients with
dementia. They should therefore be used only in selected pts who have bothersome OAB ssx & who do not adequately respond to prompted toileting alone, & who
demonstrate tolerance + responsiveness to the drug. Additionally, the cognitive SE of oxybutynin have been shown to be related to peak drug levels, which may be more
pronounced with immediate-release formulations. Urodynamic testing is not CI, even in frail nursing-home pts, but the initial approach should be prompted voiding.
If used for at least 1 yr, finasteride, a 5-α-reductase inhibitor, reduces the need for Sx to treat lower UT ssx 2/2 prostatic enlargement. Finasteride therapy would not be
likely to address urgency UI for this resident in the short term. Condom catheters increase the risk of infxn compared w/ no device and should be avoided, unless a pt
prefers them for managing UI in the setting of impaired mobility.

Visual Loss & Eye Conditions

 Impact
o Visual impairment (acuity < 20/40)
 Prevalence increases with age
 Affects 20% to 30% of those aged 75+ years
o Blindness (acuity < 20/200)
 Prevalence: 2% of those aged 75+ years
 50% of blind population is aged 65 and older
 Screen q 1 – 2 yrs

Retinal Detachment Flashes, floaters, decreased vision


Acute Angle-Closure Glaucoma Eye pain, HA, ocular hyperemia, hazy cornea, dilated pupil, vision, N/V
Ischemic Optic Neuropathy Sudden loss of vision (complete or partial) in one eye, swollen optic nn
Central aa Occlusion/Giant Cell Arteritis Sudden painless loss of vision in one eye; if from giant cell arteritis, then review of ssx may reveal jaw
claudication, HA, transient diplopia, etc.
Bacterial Keratitis Decreased vision, eye redness, pain, d/c
Scleritis Eye redness, pain, decreased vision
Posterior uveitits Floaters, decreased vision
Corneal Ulcers Eye redness, pain, decreased vision, corneal infiltrate
Uveitis Photophobia, eye redness, decreased vision
Herpes Zoster Ophthalmicus Eye redness, pain, burning, rash, decreased vision, light sensitivity, characteristic skin lesions

Red Eye
Subconjunctival Hemorrhage Supportive Tx w/ artificial tears
Dry eye Artificial tears, cyclosporine 0.2% eye drops
Blepharitis Lid scrubs, ophthalmic Abx ointment ghs to eyelids, PO doxy
Lid malposition or lid exposure Ocular lubricant, refer for Sx

51
Allergic conjunctivitis Cold compresses, allergen avoidance, topical/systemic antihistamines
Viral conjunctivitis Supportive Tx w/ artificial tears; refer to ophtho if vision significantly affected
Chalazion Warm compresses, may refer for excision
Herpes simplex keratitis Trifluidine eye drops, refer to ophtho
Herpes zoster ophthalmicus Tear drops, refer to ophtho immediately if lesions on tip of nose (Hutchinson’s Sign)
Angle-closure Glaucoma Acetazolamide PO or IV; refer to ophtho immediately

Common complaints
Floaters, flashes Refer to ophtho immediately, may be retinal detachment or vitreous hemorrhage
Sudden decrease in vision Refer to ophtho immediately, may be 2/2 # of vision-threatening problems

Diplopia
Monocular Refractive error, cataract
Binocular Microvascular infarct to cranial nn, giant cell arteritis, compressive tumor

Keratitis Sicca (dry -Tear production decreases w/ age -Artificial tears during day + ointment @ bedtime
eyes) -Redness, foreign body sensation, reflex tearing -Topical cyclosporin A 0.2% in severe cases to Tx underlying
inflammatory
-Tx accompanying blepharitis
Lid abnormalities -Common in elderly Tx: Sx
-Elasticity & tensile strength are lost with age
-Blepharochalasis (drooping of brow) & blepharoptosis (drooping
of eyelid)  cosmetic deformity or impair vision
-Lid ectropion (eversion) or entropion (inversion)
Herpes Zoster -Painful reactivation of varicella zoster -PO acyclovir or famciclovir can shorten course
Ophtalmicus -Dermatomal distribution of weeping vesicles affecting ophthalmic -Post-herpetic neuralgia may be debilitating  Tx: local ointments
branch of trigeminal nn (capsaicin, lidocaine) or systemic Rx (narcotics, TCAs, gabapentin,
-Hutchinson’s Sign = lesions on tip of nose pregabalin)

 Refractive Error
o Leading cause of visual impairment worldwide, along with cataracts
o Treatment: eyeglasses, contact lenses, laser refractive surgery
o Ametropia
 Myopia (nearsightedness)
 Hyperopia (farsightedness)
 Astigmatism (visual distortion)
o Presbyopia ( ability to focus on near objects)
 Begins after age 40
 Caused by gradual hardening of the lens & decreased muscular effectiveness of the ciliary body
 Cataract
o Symptoms include:  glare,  contrast sensitivity,  visual acuity, change in color perception
o Risk factors:  age,  vitamin intake, UVB exposure, smoking, ETOH, long-term corticosteroid use, DM
o Treatment: surgical extraction
 90% of patients achieve vision ≥ 20/40
 3 million surgeries are performed annually in US
 Local or topical anesthesia, small-incision sonographic breakdown and aspiration of the lens, placement of an artificial lens
 Age related macular degeneration
o MCC of irreversible blindness among older adults in developed world
o RF: age, genetics, smoking, hypertension, fair skin
o Diagnosis: presence of drusen (dry form) or of choroidal neovascularization (CNV) (wet form)
o Treatment
 Vitamin C, vitamin E, zinc, lutein, zeaxanthin—to decrease risk of CNV in dry forms
 Intravitreal injections of VEGF inhibitors—to treat CNV in wet form
 Diabetic retinopathy
o Epidemiology: Among people who have had type 2 diabetes at least 10 years:
 70% show retinopathy
 Nearly 10% show proliferative disease
o Most important risk factors: Duration of disease, control of blood sugar and BP
o Prevention: Tight glc control & BP control; however, targets should be individualized for geriatric patients
o Treatment: Laser treatment and intravitreal injections; control of blood glucose and BP
o Types: nonproliferative, preproliferative (severe nonproliferative), proliferative
Nonproliferative -Microaneurysms
-Intraretinal hemorrhages
-Exudates
-Macular edema
Preproliferative -Multiple intraretinal hemorrhages
-Venous caliber changes
-Intraretinal microvascular abnormalities (capillary shunting)
-Capillary nonperfusion or ischemia
Proliferative -Neovascularization of the retina
-Neovascularization of the disc
-Visual loss d/t vitreous hemorrhage or traction retinal detachment

 Glaucoma
o Defined as characteristic optic nerve head damage and visual field loss

52
o Affects >2.25 million Americans 40+ years old
o Second MCC of irreversible blindness worldwide; most common cause among black Americans
o >3 million office visits each year
o Elevated intraocular pressure is a major RF, but many patients with glaucoma have “normal” pressures
 1º open angle glaucoma
o MC form of glaucoma
o Slow aqueous drainage leads to chronically elevated intraocular pressure (IOP)
o Patients are assx and may suffer substantial visual field loss before consulting an ophthalmologist
o Causes are multifactorial and polygenic
 Acute angle-closure glaucoma
o Precipitous increase in IOP
o Redness and pain with acute vision loss and often headache, nausea and vomiting
o Emergent ophthalmologic referral required
 Glaucoma management
o Intraocular pressure–lowering medications (local and systemic)
 Aqueous suppressants
 Aqueous outflow facilitators
o Laser trabeculoplasty
o Filtering surgery  antimetabolite
o Drainage devices
o Ciliary body destructive procedures
 Anterior ischemic optic neuropathy
o Microvascular occlusion of the blood supply to the optic nerve
o Due to atherosclerotic vascular disease or inflammation (temporal arteritis)
o Results in acute vision or field loss
o Anterior ischemic optic neuropathy can  acute vision or field loss. Microvascular occlusion of the blood supply to the optic nn can be attributed
to atherosclerotic vascular dz or inflammation of GCA. The nonarteritic form typically affects pts w/ vasculopathic RF (DM and HTN); the latter,
the arteritic form, tends to occur in elderly with a h/o myalgias, HA, & wt loss. An increased Westergren ESR and a (+) temporal aa bx are
diagnostic. Systemic corticosteroid Tx is crucial to avoid visual loss in the other eye. B/c of the hypotensive effects of phosphodiesterase inhibitors
(eg, sildenafil), it has been speculated that they may contribute to nonarteritic ischemic neuropathy; however, causation has not been proved.
 Charles Bonnett Syndrome
o Visual hallucinations experienced by patients with significant visual impairment
o May be elementary shapes or complex such as children, animals
o Patients have a clear sensorium, are aware that visions are not real
o Hallucinations may be elementary shapes, or MC they are complex and highly organized with pts seeing small children, multiple animals, or a vivid
scene as one would see in a movie. Patients with this syndrome have a clear sensorium and are aware that the visions are not real. It has been
suggested that this syndrome is a concomitant of the phantom limb syndrome. Underlying conditions include age-related macular degeneration,
glaucoma, diabetic retinopathy, & cerebral infarction. If the cz of vision loss is known and there is no homonymous visual field defect present,
neuroimaging is not necessary.
o The best Tx for the Charles Bonnet syndrome is education, reassurance, and support. Pts should be informed that the hallucinations are a sign of
eye dz, not mental illness. An occasional pt has partial insight or loses insight and becomes very distressed by this ssx. When this distress is
significant or leads to dangerous behavior, a cautious trial of low dosages of a 2nd-gen antipsychotic Rx can be considered.
 Low-Vision Rehab
o Available to patients with acuity < 20/60
o Improve lighting; provide reading material w/ bold enlarged fonts & accentuated black-on-white contrast
o Magnification: high-plus spectacles, magnifiers, closed-circuit TV, telescopic devices
o Eccentric viewing for pts with ARMD with central macular pathology: training to use off-center fixation
o Talking devices or Braille for those who have lost vision altogether
o Smartphone apps can provide magnification, money recognition, and dictation functions
 Choosing Wisely
o Do not perform preoperative medical tests for eye surgery without specific indications.
o Most cases of acute conjunctivitis have a viral etiology. Do not treat viral infections with antibiotics; if diagnosis is uncertain, patients may be
followed closely for resolution.
o Do not place temporary or permanent punctal plugs for mild dry eye syndrome before trying other medical Tx.

Cases / Questions
Case #1
• A 68-year-old woman describes vision problems.
 Progressive blurring of distance vision
 Increased difficulty with nighttime driving, which she attributes to worsening glare & haloes around lights
 She no longer needs glasses for reading.
• History: hypertension, hypercholesterolemia
Which one of the following is the most likely cause of the changes in her vision?
A. Uncontrolled hypertension
B. Poorly controlled blood glucose
C. Choroidal neovascularization
D. Cataract progression
E. Increased intraocular pressure

ANSWER: D
The pt’s ssx most likely result from age-related progression of nuclear sclerotic cataracts. The cataracts cause a darkening and hardening of the lens that can change its
refractive index—most commonly, a myopic shift. The induced nearsightedness allows patients to read without their glasses.
Uncontrolled HTN can affect vision. Typically, the changes are more acute & occur in the setting of abrupt + extreme BP increases. Many pts have HA, and their visual
ssx are more likely to entail diffuse image blur caused by scotomas.
Poorly controlled blood glc can result in changes in the lens & retina. This pt has no h/o DM, and findings a/w diabetic retinopathy typically occur years after initial
diagnosis.

53
Choroidal neovascularization occurs in wet age-related macular degeneration. Onset is typically acute & monocular, and pts typically experience a central scotoma.
Increased IOP can  changes in vision. However, MC in glaucoma, the initial loss is in the periphery of the visual field, not in the central vision that is used for reading.

Case #2
• A 72-year-old black man has RT shoulder trauma after he fell in a parking lot. He states that he was walking to his car when he suddenly tripped & fell forward
onto his arm. He learned that he had tripped over a curb.
• History: hypertension, high cholesterol, glaucoma
Which one of the following most likely contributed to his fall?
A. Acute glaucoma
B. Failure to use glaucoma drops that morning
C. Poor peripheral vision
D. Retinal embolus from a carotid plaque

ANSWER: C
Most likely, the patient’s failure to see the curb and his subsequent trip and fall are related to poor peripheral vision due to glaucoma. Patients with glaucoma can maintain
good central visual acuity, allowing them to read and to pass driving vision tests, so that frequently loss of peripheral vision can go unnoticed.
Although acute glaucoma can occur in either open- or closed-angle glaucoma, it is rare in open-angle glaucoma. Patients with acute glaucoma invariably experience
significant eye pain, which is not described here.
Topical eye drops for glaucoma are used on a chronic basis to lower eye pressure and prevent additional damage to the optic nn. Missing a single dose is unlikely to have
a significant, immediate impact on the patient’s vision.
Retinal emboli are typically monocular, allowing for compensation by the other eye, so an embolus is not likely to have caused this patient’s fall.

Case #3
• An 88-year-old woman comes to the office for an urgent visit because, when she awoke this morning, she found that she could no longer see to read. She
depends on her left eye for vision because she lost her right eye in an accident when she was a child.
• On examination, vision is 20/400 in the left eye. She does not perceive light in the right eye. She is referred emergently to an ophthalmologist, who diagnoses
neovascular age-related macular degeneration.
Which one of the following is the most appropriate therapy?
A. Topical prostaglandin analog
B. Phacoemulsification with intraocular lens implantation
C. Intravitreal vascular endothelial growth factor inhibitor
D. Glasses with updated refraction
E. Oral lutein, zeaxanthin, and omega-3 eye nutrients

ANSWER: C
Intravitreal VEGF inhibitors are the mainstay of Tx for neovascular age-related macular degeneration. These agents inhibit choroidal neovascularization & minimize
vision loss a/w wet age-related macular degeneration.
Topical prostaglandin analogs are used to treat glaucoma and are not indicated for age-related macular degeneration. Phacoemulsification is used to break cataracts before
their removal with surgery. A new pair of glasses with an updated refraction is unlikely to help this patient: the problem is at the level of the retina, not with uncorrected
optical distortion.
Although lutein, zeaxanthin, and omega-3 eye nutrients slow vision loss in both neovascular and dry macular degeneration, they do not provide benefit in acute neovascular
disease. These nutrients possibly should be part of the long-term plan for this patient.

54

Potrebbero piacerti anche